OAT - biology

¡Supera tus tareas y exámenes ahora con Quizwiz!

although there is abundant CO2 that is carried in blood, blood does not become acidic, because -it is absorbed by the leukocytes -it combines with water to form H2CO3 which is neutralized by NaCO3 -CO2 transport and blood buffers play an important role in it -it is continuously diffused through the tissues and is not allowed to accumulate -none of the above

-CO2 transport and blood buffers play an important role in it due to the buffering action of blood, CO2 that is carried out by blood does not become acidic.

which one of the following sets of ions is necessary in the chemical events for muscle contraction? -Na+ and K+ -Na+ and Ca2+ -Na+ and Mg2+ -Ca2+ and Mg2+ -Ca2+ and K+

-Ca2+ and Mg2+ Ca2+ and Mg2+ ions help in ATPase activity and muscle contraction. the release of calcium ions from the sarcoplasmic reticulum triggers the muscle contraction process. Mg2+ is required for actomyosin ATPase activity, which is essential for muscle contraction.

in the DNA microarray, the probes are made of -primers -RNA -DNA -antibodies -none of the above

-DNA primers are made of RNA and they cannot be used to probe DNA

according to the Central Dogma of Molecular Biology, what is the sequential flow of hereditary information in the cell? -protein --> tRNA --> mRNA --> DNA -DNA --> mRNA --> tRNA --> DNA -DNA --> mRNA --> protein -mRNA --> tRNA --> protein -protein --> mRNA --> tRNA --> DNA

-DNA --> mRNA --> protein

reverse transcription is the process of synthesis of -RNA from DNA -RNA from RNA -DNA from DNA -DNA from RNA -C & D

-DNA from RNA reverse transcription is the process of making a double stranded DNA molecule from a single stranded RNA template. it is called reverse transcription as it acts in the opposite or reverse direction to transcription.

all of the following statements characterize DNA replication EXCEPT one. which one is this EXCEPTION? -second DNA strand is in form of Okazaki segments -DNA is synthesized in 3' to 5' direction in one strand and 5' to 3' in another -growth of DNA chain is discontinuous -overall growth of DNA chain is bidirectional -none of the above

-DNA is synthesized in 3' to 5' direction in one strand and 5' to 3' in another DNA can only be synthesized in a 5' --> 3' direction

Okazaki fragments are joined in a correct sequence by -DNA polymerase I -DNA ligase -RNA polymerase -primase -none of the above

-DNA ligase an Okazaki fragment is a relatively short fragment of DNA with an RNA primer at the 5' terminus created on the lagging strand during DNA replication. when the lagging strand is being replicated on the original strand, the 5'-3' pattern must be used. thus, a small discontinuity occurs and an Okazaki fragment forms. these fragment are processed by the replication machinery to produce a continuous strand of DNA and hence a complete daughter DNA helix. adjoining fragment are linked together by DNA ligase, using phosphodiester bonds, to create a continuous strand of DNA.

the net equation for cellular respiration in the cell is written as - 6CO2 + 6H2O --> C6H12O6 + 6O2 - C6H12O6 + 6CO2 --> 6H2O + 6O2 - C6H12O6 + 6O2 --> 6CO2 + 6H2O - 6CO2 + 6O2 --> C6H12O6 + 6H2O -none of the above

- C6H12O6 + 6O2 --> 6CO2 + 6H2O

three molecules of DNA are expressed as: (i) TTCAGAGAACTT AAGTCTCTTGAA (ii) CCTGAGAGGTCC CGACTCTCCAGG (iii) TCCAAGTTACCT AGGTTCAATGGA which of the following represent the correct order of their melting points from highest to lowest? -(i) > (ii) > (iii) -(ii) > (iii) > (i) -(i) > (iii) > (ii) -(iii) > (ii) > (i) -(ii) > (i) > (iii)

-(ii) > (iii) > (i) because G - C base pairs have 3 hydrogen bonds whereas A - T base pairs have 2 hydrogen bonds. the higher the G - C content the higher the melting point

cystic fibrosis is a recessive disease in humans. Based on the principles of classical Mendelian genetics, what is the probability that a child will be born with the disease if one parent is a carrier of the trait and the other parent is homozygous dominant? -0% -25% -50% -75% -100%

-0%

in humans, how many sperm(s) are needed to fertilize an egg? -1 -10 -100 -10,000 -more than 10,000

-1 among 200-300 million sperm, only 300-500 can reach ampulla of the fallopian tube where the fertilization occurs. there, only one sperm can fertilize the egg.

in which phase of meiosis do synapses occur? 1. first prophase 2. second prophase 3. zygotene stage of first prophase 4. pachytene stage of first prophase -1 only -1 and 2 -1 and 3 -1 and 4 -2 only

-1 and 3 in prophase I, every chromosome lines up next to its homologue. the pairing go homologous chromosomes, which does not occur in mitosis, is called synapsis. all these occur in the zygotene stage of prophase I.

a recombinant frequency of what percentage is equivalent to 1 genetic map unit -1% -10% -25% -50% -100%

-1% Alfred Sturtevant developed the first genetic map. according to him, the probability of crossing over in the region between genes increase with the distance between linked gene. the unit to measure the distance between genes for one product of meiosis in 100 is recombinant and a recombination frequency of 1%, which is equivalent to 1 map unit.

an ecosystem has three trophic levels. the percentage of energy stored in the third trophic level is -1% -10% -20% -30% -50%

-1% according to Lindeman's ten percent law, during the transfer of organic food from one trophic level to the next, only about ten percent of the energy is transferred. the remaining is lost during transfer or broken down in respiration. thus the percentage of energy stored in the second trophic level is 10% and that stored in the third trophic level is 1%.

the total number of H2O molecules used per aerobic metabolism of a glucose molecule: -8 -10 -12 -16 -none of the above

-10 2 molecules of H2O are used in glycolysis and 8 molecules in Kreb's cycle. for this reaction of glycolysis, 2 molecules of H2O are used. 2-phosphoglycerate --> phosphoenolpyruvate for each of the following reaction of Kreb's cycle 2 molecules of H2O are used. Acetyl CoA --> Citric acid cis aconitic acid --> isocitric acid succinyl CoA --> succinic acid fumaric acid --> malic acid

if an animal has 10 pairs of chromosome, the number of linkage group that animal is -5 -10 -15 -20 -25

-10 linked genes form a linkage group. homologous chromosomes contain the same genes or alleles and so the number of linkage group is half the number of chromosomes. so, 10 pairs of chromosomes form 10 linkage groups.

what percentage of energy is transferred to a snake from a frog when the snake eats the frog? -10 -20 -30 -40 -50

-10 on average 10% of the total energy consumed in one trophic level is incorporated into the organisms in the next.

the average size of a eukaryotic cell is -1 - 10 nm -10 - 100 nm -10 - 100 um -1 - 10 mm -none of the above

-10 - 100 um

a woman with two genes for hemophilia and one gene for colorblindness on one of the 'X' chromosomes marries a normal man, how will the progeny be? -hemophilic and colorblind daughters -all sons and daughters hemophilic and colorblind -50 percent hemophilic colorblind sons and 50 percent normal sons -50 percent hemophilic daughters and 50 percent normal daughters -100 percent hemophilic sons and 50 percent colorblind sons

-100 percent hemophilic sons and 50 percent colorblind sons both hemophilia and colorblindness are sex-linked diseases because both are related to X chromosome in females. when performing test cross, the mother may only donate an X gene for hemophilia and the father may either donate a normal X or a Y, so the daughters will be all carriers, and all the sons will be hemophiliac since the normal father must donate a Y. the sons will be 50% colorblind, but 100% hemophiliac. test cross: for hemophilia: XhXh * XY --> Females: XhX / XhX (both heterozygous, carriers); Male: XhY/XhY (both hemophiliacs). for color blindness: XcX * XY --> Females: XcX/XX (one carrier, one normal); Male: XcY/XY (1 color blind; 1 normal).

hemophilia is a sex-linked disease in humans. if an affected female has children, what is the probability that she will have a daughter who is a carrier for the trait, if her husband is homozygous dominant for the condition -0% -25% -50% -75% -100%

-100%

when a dwarf pea plant was treated with gibberellic acid, it became as tall as a normal tall pea plant. if these pea plants are crossed with pure tall plants then what will be the phenotypic ratio in F1 generation -100% dwarf plants -100% tall plants -50% dwarf and 50% tall plants -75% dwarf and 25% tall plants -25% dwarf and 75% tall plants

-100% tall plants here, one plant is chemically treated tall plant and the other one is genetically tall plant. so when these plants are crossed their F1 generation will produce 100% tall plants.

in the krebs cycle, how many ATP molecules are formed from a molecule of pyruvic acid -12 -15 -22 -24 -30

-12 during oxidation in TCA cycle each molecule of pyruvic acid produces three (NADH + H+), one FADH2 and one GTP (ATP equivalent). considering each (NADH + H+) and FADH2 produce 3 and 2 molecules of ATP respectively during the oxidation through Electron Transport System (ETS), we have the following formula: (3 ATP x 3 NADH) + (2 ATP x 1 FADH2) + (1 ATP equivalent) = 12 ATP produced

which is NOT an eukaryotic r-RNA? -5.8 S -16 S -18 S -28 S -none of the above

-16 S 16 S r-RNA is a small prokaryotic ribosomal RNA. 5.8 S and 28 S are large and 18 S is small eukaryotic r-RNA

in hybridization, Tt * tt gives rise to the progeny in the ratio of -1:1 -1:2 -2:1 -1:2:1 -2:1:2

-1:1

test crossing of an animal with genotype AaBb will produce offspring in the ratio -3:1 -1:2:1 -1:1:1:1 -9:3:3:1 -none of the above

-1:1:1:1

if the F1 generation is crossed with itself in a monohybrid cross, what is the genotypic ratio of the offspring? -1:3 -3:1 -1:2:1 -9:3:3 -1:16

-1:2:1

the number of floating ribs in the human body is -2 pairs -3 pairs -4 pairs -5 pairs -6 pairs

-2 pairs in the human body, there are two pairs of floating ribs - the 11th and 12th pair. these ribs do NOT have a sternal part hence they are called floating ribs.

if twenty-eight percent of the bases in a DNA sequence are adenine, what percentage of the bases are cytosine -22 -28 -44 -56 -none of the above

-22 A-T C-G 28%A = 28%T = 56% 100%-56% = 44% 44% / 2 = 22% 22%

chimpanzees have 48 chromosomes, how many chromosomes are present in a chimpanzee's daughter cells after meiosis? -12 -23 -24 -48 -96

-24

when crossing two similar hybrids, the percentage of recessive is -0 -25 -50 -75 -100

-25

the number of bones in a human child at birth is approximately -106 -206 -270 -33 -600

-270 human babies are born with more bones than there are in adults. many of these bones are un-fused, and as the child develops the number of individual bones decreases. the average number of bones in an adult is 206. there are 33 bones in the vertebral column. the number of skeletal muscles in the body is approximately 600

how many days after fertilization is the morula formed? -3 -7 -12 -14 -21

-3 after fertilization, a zygote begins mitotic cell division known as cleavage. cleavage produces a ball of cells called morula, 3 days after fertilization. the blastocyst develops from the morula about 5 days after fertilization before implantation.

how many kinds of point mutation can occur? -1 -2 -3 -4 -more than 4

-3 missense mutation -results in a different amino acid substitution nonsense mutation - results in a stop codon so that incomplete proteins are produced silent mutation - produces code for the same or a different amino acid but without any functional change in the protein

the circulatory system is divided into how many branches? -1 -2 -3 -4 -6

-3 the coronary circuit supplies blood to the heart. the systemic circuit supplies oxygenated blood to the tissues of the body. the pulmonary circuit carries deoxygenated blood to the heart where it is oxygenated, and returns it back to the systemic circuit.

how many 'stop' codes are there among all the codons? -1 codon -2 different codons -3 different codons -4 different codons -no 'stop' codon

-3 different codons of the 64 possible codons, there are 3 'stop' codons: UGA, UAG, UAA

based on fossil record and technologies such as radiocarbon dating, when did life begin on earth? -150,000 years ago -1.8 million years ago -3.8 billion years ago -5 billion years ago -none of the above

-3.8 billion years ago

in DNA, if the percentage of thymine is 20%, then what is the percentage of guanine? -20 -30 -40 -60 -80

-30 in DNA the number of adenine resides is always equal to the number of thymine (A=T). the number of guanine residues is always equal to the number of cytosine residues (G=C). so, A + G = T + C thus, if the percentage of thymine is 20% then the percentage of guanine will be 30%

the normal ratio of systolic, diastolic and pulse pressure is respectively -1:1:1 -1:2:3 -3:2:1 -1:1:2 -none of the above

-3:2:1 systolic pressure is the blood pressure when ventricles are contracted diastolic pressure is when ventricles are expanded. the difference between these two is the pulse pressure. ratio of these three is 120 mmHg: 80 mmHg: 40 mmHg, which is reduced to a ratio of 3:2:1

inside the cell, the sodium potassium pump pushes ____Na+ molecules out of the cell and carries ______K+ molecules into the cell in a single cycle -1; 3 -3; 1 -2; 3 -3; 2 -bundle of His

-3; 2

which of the following is used to differentiate DNA from RNA molecules -1H -2H -3H -12H -none of these

-3H to differentiate DNA from RNA, radioactively labeled thymidine is used as uracil, usually replaces thyme in RNA. tritium (3H), radioactive isotope of H2 is used to label thymidine as it emits very weak beta particles during radioactive decay which will penetrate only 1 micro meter.

how many molecules of CO2 are formed in the Krebs cycle? -2 -3 -4 -5 -6

-4 when alpha-ketogluterate is formed from oxalosuccinate (decarboxylation process) with the help of the enzyme oxalosuccinate decarboxylase, 2 molecules of CO2 are produced. at the next step, alpha-ketogluterate is oxidized to succinyl CoA (oxidative decarboxylation) with the help of the enzyme dehydrogenase. in this step, also 2 molecules of CO2 are produced.

what are the sizes of the two subunits that form the 80S ribosome in eukaryotes? -40S and 40S -30S and 50S -40S and 60S -10S and 70S -none of the above

-40S and 60S

the genetic code uses ______ nucleotides to code for ______ amino acids -3; 20 -4; 20 -6; 20 -10; 20 -none of the above because the genetic code is different in prokaryotes

-4; 20 adenine, guanine, cytosine, uracil

suppose a CO2 molecule is diffused from the matrix of a mitochondria of an animal cell to the interior of a Golgi apparatus of another cell nearly touching the first cell. it will cross a minimum of -2 membranes -3 membranes -4 membranes -5 membranes -6 membranes

-5 membranes the CO2 molecule has to cross two membranes of the mitochondria, the plasma membrane of the first cell, the plasma membrane of the second cell and the membrane of the Golgi apparatus; therefore, making the correct answer 5 membranes.

the daughter DNA sequence can be formed from this template DNA strand: 3' GACTAACGATGC 5' -3' CGUAGCAAUCAG 5' -3' CTGATTGCTACG 5' -3' CGTAGCAATCAG 5' -5' CTGATTGCTACG 3' -5' GCATCGTTAGTC 3'

-5' CTGATTGCTACG 3'

what is the complementary mRNA sequence for the following DNA sequence: 3' ATCGGGCATG 5' -5' TAGCCCGTAC 3' -5' UAGCCCGUAC 3' -3' AUCGGGCUAG 5' -5' CAUGCCCGAU 3' -3' GATCGGGCTA 5'

-5' UAGCCCGUAC 3' mRNA does not contain any thymine bases, so A and E are eliminated.

during DNA replication, nucleotides are added to the growing DNA strand in what direction -3' to 5' -5' to 3' -2' to 5' -5' to 2' -the strands are identical so they can be added from any direction

-5' to 3'

bile secreted from the gall bladder is -2-3 times concentrated -5-10 times concentrated -10% dilute -15% dilute -20% dilute than the bile secreted from the liver

-5-10 times concentrated inbuilt, the concentration of bile salts, sodium glycoholate and sodium taurocholate is 0.8%. it is then stored in the gall bladder through bile duct. the gall bladder removes water from the bile and concentrates up to 5-10 times. the concentration then becomes about 4%.

if a woman is a carrier for the recessive allele that causes hemophilia, what is the probability that she will have a daughter or son who is affected by the disease if her husband is also heterozygous for the allele -0% -25% -50% -75% -none of the above

-50%

what will be the probability that a boy will be color blind, given the his mother is normal but her mother was color blind? nothing is known about the father's sight -0% -25% -50% -75% -100%

-50%

a woman with two genes for hemophilia and one gene for color blindness on one of the X chromosomes marries a normal man. How will their progeny be? -all sons and daughter will be hemophilic and color blind -50% hemophilic daughters and 50% colorblind daughters -only hemophilic and colorblind daughters -50% hemophilic colorblind sons and 50% normal sons -50% hemophilic sons and 50% colorblind sons

-50% hemophilic colorblind sons and 50% normal sons both hemophilia and color blindness are sex linked diseases because both are related to the X chromosome in females. when a woman with two genes for hemophilia and one gene for colorblindness on the X chromosomes, marries a normal man, their progeny will be 50% hemophilic colorblind sons and 50% normal sons.

which of the following are components of the 70S ribosomes? -50S and 20S -40S and 30S -50S and 30S -40S and 40S -A & B

-50S and 30S the unit of measurement of ribosomal sub unit is the Svedberg unit, a measure of the rate of sedimentation in centrifugation rather the size and so fragment names do not add up. the prokaryotes have 70S ribosome which consists of two subunits - 50S and 30S. the 50S subunit is large. the 30S subunit is smaller and occurs above the 50S subunit.

how many CO2 molecules are produced in aerobic respiration? -2 -3 -4 -5 -6

-6 when pyruvic acid is converted to Acetyl CoA (by process of oxidative decarboxylation) two molecules of CO2 are produced and in Krebs cycle four molecules of CO2 are produced.

for the formation of a 1 glucose molecule, the calvin cycle circulates -1 time -2 times -4 times -6 times -8 times

-6 times in the Calvin cycle, the resynthesis of RuBP takes place which require the cycle to circulate 6 times.

in the genetic code dictionary, how many codons are used to code for all the twenty essential amino acids? -20 -32 -60 -61 -64

-61 the coding dictionary comprises of 64 possible triplet codons. but these 61 codes are used to code for all the 20 essential amino acids and are termed as sense codons. the other three triplets UAA, UAG, AND UGA do not code for amino acid and are called non-sense codons.

the genetic code contains 4 nucleotides and 20 amino acids, which are organized into 3 letter codons that produce a single amino acid. how many codons are in the genetic code? -20 -24 -64 -80 -240

-64 there are 60 (3*20) amino acids producing codons and 4 regulatory codons. these 4 codons include the start codon (AUG) and 3 stop codons (UAA, UGA, UAG).

how many codons are in the genetic code? -4 -8 -16 -32 -64

-64 with four different bases (A, U, G, and C) possible and three bases per codon, there are 4^3 = 64 possible combinations, which encode 20 different amino acids and 3 stop codons (UAA, UAG, UGA)

in mammalian development, implantation occurs about -2 days after fertilization -4 days after fertilization -7 days after fertilization -10 days after fertilization -14 days after fertilization

-7 days after fertilization implantation is the process in which the blastocyst burrows and embeds itself into the lining of the uterus. it occurs about a week after fertilization.

which of the following is the sedimentation coefficient found in ribosomes of mitochondria, bacteria and chloroplasts? -50 S -80 S -70 S -30 S -none of the above

-70 S bacteria have 70S ribosomes, each consisting a small 30S and large 50S subunits. mitochondria and chloroplasts also have 70S ribosomes.

if there are 3 pairs of homologous chromosomes, how many chromosomal combinations will be expected in the gametes due to meiosis? -1 -3 -6 -8 -27

-8 2^3 = 8

to produce 256 daughter cells, mitosis cell division should be repeated for -128 times -64 times -32 times -16 times -8 times

-8 times

which of the following represents the average value of human diastolic blood pressure -40 mm -80 mm -120 mm -120/80 mm -140 mm

-80 mm the blood pressure during complete cardiac diastole is 80 mmHg in an adult. when the heart is relaxed, it is called diastolic blood pressure.

if four percent of the population is affected by Tay-Sachs disease, what is the frequency of the dominant of the allele in the population -2% -4% -20% -80% -96%

-80% p + q = 1 p^2 + 2p1 + q^2 = 1 4% = 0.04 q^2 = 0.04 0.04(square root) = 0.2 frequency of dominant allele is: 1 - 0.2 = 0.8 --> 80%

enzymes are effective biological catalysts because they -lower the activation energy of a reaction -increase the rate of a reaction -increase the activation energy of a reaction -A & B -all of the above

-A & B

exergonic reactions -release energy -have a negative delta G -have a positive delta G -A & B -A & C

-A & B

in the reproductive system, luteinizing hormone -stimulates testosterone production in males -stimulates ovulation in females is produced by the hypothalamus -A & B -all of the above

-A & B

osmosis is -the movement of water through cell membranes -a type of diffusion -a form of active transport -A & B -all of the above

-A & B

the cortical reaction -occurs when a sperm fuses with an egg -occurs in the egg -occurs in the spermatozoa -A & B -all of the above

-A & B

which of the following are classified as primary producers -algae -plants -fungi -A & B -all of the above

-A & B

which of the following enzymes functions in DNA repair -DNA polymerase I -DNA polymerase II -DNA polymerase III -A & B -all of the above

-A & B DNA polymerases are a group of enzymes that facilitate DNA replication by adding new nucleotides to the growing DNA strands. DNA polymerase III is the major enzyme that is involved in DNA replication and it does not have any repair activity. DNA polymerase I is a major repair enzyme and it plays a minor role in replication. DNA polymerase II is specialized for DNA repair.

which of the following terms describes the process of photosynthesis -anabolic -endergonic -exergonic -A & B -A & C

-A & B anabolism is the set of metabolic pathways that construct molecules from smaller units. these reactions require energy, known also as an endergonic process. anabolism is the building-up aspect of metabolism, whereas catabolism is the breaking-down aspect.

beta-oxidation occurs in -peroxisome -mitochondria -ER -A & B -B & C

-A & B at the beta carbon of mitochondria fatty acid is oxidized to form Acetyl CoA with the help of enzymes in the matrix. in every cycle of the beta-oxidation Acetyl CoA is added. peroxisomes can beta-oxidize medium and long chain fatty acids, as well as very long chain fatty acids that are poor substrates for mitochondrial beta-oxidation.

which of the following is a common cofactor of enzymes -Zn2+ -Fe2+ -biotin -A & B -all of the above

-A & B cofactors are inorganic compounds that must be bound to the enzyme in order for it to function properly. metals such as zinc and iron are common enzyme cofactors. coenzymes are organic molecules that enzymes need to function. common organic molecules that function as coenzymes include biotin.

diseases of the thyroid gland occur when it is unable to produce its hormones in the correct concentrations. which of the following hormones is produced by the thyroid gland? -thyroxine -thyroid stimulating hormone -triiodothyronine -A & C -all of the above

-A & C TSH is produced by the anterior pituitary gland. and it controls the function of the thyroid gland.

which of the following produces ATP glucose --> glycolysis --> pyruvic acid --> acetyl coA --> krebs cycle -glycolysis -pyruvic acid --> acetyl coA -krebs cycle -A & C -all of the above

-A & C in glycolysis, ATP is produced from ADP when 1-3-biphosphoglycerate is changed into 3-phosphoglycerate and phosphoenol pyruvate is changed to just pyruvate. when pyruvic acid is changed to Acetyl CoA, CO2 & NADH is formed. in the Krebs cycle, ATP is formed from GTP.

during recombination, crossover occurs between -autosomes -sex chromosomes -sister chromatids -homologous chromosomes -A & D

-A & D

pedigree analysis is used to study genetic diseases in families if the pedigree for a family with affected individuals shows no carriers on the pedigree chart, what type of genetic disease is present in the family? -autosomal dominant -autosomal recessive -x-linked -y-linked -A & D

-A & D

in the DNA of a single species, which of the following correctly represents the amount of the bases? -A = G -G = T -A = G = T = C -A + T = G + C -A + G = C + T

-A + G = C + T

A: a cell membrane shows fluid behavior B: a membrane is a mosaic or composite of diverse lipids and proteins. Which of the following statements is correct about 'A' and 'B' -A is correct and B is explanation to A -A is correct and B is not explanation to A -A is correct and B is wrong -A is wrong but B is correct -both A and B are wrong

-A is correct and B is not explanation to A the fluid mosaic model states that the phospholipid bilayer of a cell membrane behaves like a fluid. the membrane's lipid and protein can move laterally within the bilayer. according to this model both A and B are correct, but B does not explain A.

the hormone released by the pituitary gland that instructs the kidneys to retain water is -oxytocin -ADH -vasopressin -A & B -B & C

-B & C ADH is anti-diuretic hormon. This is also known as vasopressin. it causes decreased water excretion by instructing the kidneys to produce concentrated urine, as opposed to dilute urine. oxytocin is released by the pituitary gland, but it causes uterine contractions during childbirth.

vacuoles are membrane-bound storage sacs that are found in -bacteria -plants -animals -B & C -all of the above

-B & C bacteria does not have any membrane-bound organelles.

bone tissue remodeling is a function of which of the following -osteons -osteoblasts -osteoclasts -B & C -osteoprogenitors

-B & C osteoblasts is the building new bone tissue osteoclasts is the break down of tissue osteons are the functional units of a bone and osteoprogenitors are bone stem cells the give rise to various types of cells in the bone

which of the following can produce three dimensional image of an object? -compound microscope -stereoscope -scanning electron microscope -B & C -A & C

-B & C stereoscope can produce three dimensional images of an object by the combined action of two eyes. scanning electron microscope also can produce three dimensional images. here a beam of electrons cans over a specimen point by point and builds up an image on the fluorescent screen of a cathode ray tube.

following the fertilization of the gametes, the zygote undergoes -meiosis -mitosis -cleavage -A & C -B & C

-B & C the gametes, sperm and egg, were formed by meiosis in each parents. the fertilized egg does not go through meiosis, as this is restricted to gonads in adults who are producing gametes. the zygote goes through a series of mitotic divisions called cleavage. during this cleavage, the number of cells increases, but the size does not.

which of the following immune cells are known to be inherently phagocytic? -monocytes -neutrophils -macrophages -A & C -B & C

-B & C (neutrophils and macrophages) phagocytosis is the process by which white blood cells engulf bacteria or cellular debris. the white blood cells that are capable of this activity are neutrophils and macrophages. although monocytes mature into macrophages they are not phagocytic until they are fully developed

which of the following best characterizes the arthropoda -they have compound eye -they have malpighian tubule as an excretory organ -they have trachea as a respiratory organ -jointed appendages are found in each of the body segments -B and D

-B and D all the arthropods do not have a compound eye. for example, arthropoda under crustacea > maxillopoda > thecostraca > cirripedia do not have a compound eye. malpighian tubule and trachea are found in insects only.

which of the following is responsible for humoral immunity? -B lymphocyte -T lymphocyte -basophil -macrophage -neutrophil

-B lymphocyte in the immunity system, antibodies are produced in the B lymphocyte cells. secreted antibodies bind to antigens on the surfaces of invading microbes like viruses or bacteria, which flags them for destruction. it is called humoral immunity as it involves substances found in the humors, or body fluids.

the cup-shaped structure of the kidney that functions as a filter for salts, water, and organic wastes in the production of urine is called the -Bowman's capsule -nephron -collecting duct -proximal convoluted tubule -distal convoluted tubule

-Bowman's capsule

the capsule surrounding the glomerulus is called the -Bowman's capsule -fibrous capsule -capsid -renal fascia -none of the above

-Bowman's capsule glomerulus is a compact tuft of interconnected capillary loops in each renal corpuscle and it protrudes into a fluid filled capsule called Bowman's capsule.

stomata is open at night and close during the day in -C3 plants -C4 plants -CAM plants -C4 and CAM plants -all of the above plants

-CAM plants in C4 plants, stomata are partially closed during the hottest hours of the day. CAM plants take in CO2 during the night and fix it with variety of organic compounds. during the day, CO2 is released and enter the Calvin cycle. Therefore, CAM plants have an open stomata at night and close during the day. for the other plants the stomata opens during the day and closes at night.

the starting products of photosynthesis are -CO2 and glucose -CO2 and water -O2 and glucose -O2 and water -all of the above

-CO2 and water

which of the following neurotransmitters is active at the neuromuscular junction? -adrenalin -acetylcholine -glutamate -histamine -none of the above

-acetylcholine the neuromuscular junction is the junction between the skeletal muscle fiber and the terminal of a motor neuron. the terminal contains many vesicles filled with acetylcholine. when action potential reaches the terminals, acetylcholine is released and helps Na+ ions diffuse in by opening transmembrane channels in the postsynaptic membrane on the fiber.

the enzyme that helps the sperm to penetrate the zona pellucida is -fertilizin -acrosin -hyaluronidase -pepsin -none of the above

-acrosin during fertilization after penetrating corona radiata the sperm touches zona pellucida of the ovum. at this tine a chemical substance called spermlycin and active enzyme called across formed from sperm digest a limited area of zona pellucida. through this area the sperm enters the ovum and lies at perivitelline space.

which of the following is responsible for cell cleavage in animal cells? -accumulation of fragmoplast -microfibril contraction -desmotubule contraction -actin myosin contractile ring -none of the above

-actin myosin contractile ring during telophase, cytokinesis begins and the cell membrane is pinched inward at the middle of two poles through the action of the actin myosin contractile ring.

the beewolf wasp always circles its nests in an ever-widening circle before flying away to hunt. this is an example of which of the following behaviors: -action patterns -habituation -mating ritual -imprinting -aggression

-action patterns

which of the following is not a method by which cells internalize extracellular DNA -transduction -transfection -conjugation -active transport -transformation

-active transport

which of the following is not a tenet of the cell theory -all cells come from cells -all living things are made of cells -the smallest unit of life is the cell -all cells have membrane-bound organelles -none of the above

-all cells have membrane-bound organelles

amoebae belong to phylum sarcodina. all amoeba -are unicellular -are heterotrophic -use pseudopodia -A & C -all of the above

-all of the above

an organism's phenotype is influenced by its -environment -diet -genotype -A & C -all of the above

-all of the above

if a scientific researcher wishes to induce the formation of erythrocytes from a population of cells, what type of embryonic stem cells can he or she use? -totipotent -multipotent -pluripotent -A & C -all of the above

-all of the above

members of kingdom Protista obtain their food by -absorption -photosynthesis -ingestion -B & C -all of the above

-all of the above

members of which of the following kingdoms contain cell walls -Monera -Fungi -Plantae -A & C -all of the above

-all of the above

which of the following is/are a general characteristic(s) of the phylum chordata? -notochord -pharyngeal slit -dorsal, hollow, tubular nerve chord -post-anal tail -all of the above

-all of the above at some point in the life cycle of chordates, a notochord, a hollow dorsal nerve cord, pharyngeal slits and a post-anal tail are formed.

which of the following is an end codon? -UAA -UAG -UGA -all of the above -none of the above

-all of the above each of three codons specifies the termination of translation and results in the release of the completed polypeptide chain from the ribosome. these codons do not have rRNA molecules that recognize them. they are recognized by protein factors that terminate translation.

fungi reproduce using -mitosis -meiosis -asexual reproduction -A & C -all of the above

-all of the above fungi are heterotrophic organisms that reproduce using multiple processes. new fungi can be formed by mitosis, which produces genetically identical individuals or clones. these clones are produced by asexual reproduction, which is the creation of new offspring by a single parent. although fungi are not classified as male and female, they are capable of sexual reproduction through meiosis. the fungi are classified as (+) and (-) based on their genes. they are able to produce genetically diverse offspring through meiosis.

which of the following enzymes is found in lysosome? -acid phosphatase -triglycerol lipase -neuroaminidase -all of the above -none of the above

-all of the above lysosomes contain all the digestive enzymes that help to digest macromolecules from phagocytosis.

which of the following type(s) of motility is/are found in the small intestine? -segmentation -peristalsis -antiperistalsis -pendular movement -all of the above

-all of the above peristalsis is the rhythmic contraction of smooth muscles that helps in downward movement of the (food) content antiperistalsis is the smilier movement but in the reverse directions. this is found only in the second and third part of the duodenum segmentation is the rhythmic contraction and relaxation of the small intestine pendular movement is a passive motility found in the loops of small intestine

C2 cycle occurs in -chloroplast -peroxisome -mitochondria -all of the above -none of the above

-all of the above photorespiration is called the C2 cycle or glycolate cycle. in this process 2-phosphoglycolate is converted to 2-phosphoglycerate and two carbon glycolate, glyoxalate, and glycin are formed as intermediate compounds. C2 cycle occurs through chloroplast, peroxisome, and mitochondria.

the thee types of neurons that are found in the nervous system are the -sensory -motor -interneuron -A & B -all of the above

-all of the above sensory neuron - the type of neuron that receives information from the internal and external environments of the body interneuron - sensory neurons must deliver their inputs to the neurons that process and relay the message to the appropriate control center that will generate a response. motor neuron - once the message has been interpreted, the interneurons send a message to the motor neuron that will deliver impulses to the appropriate muscle or gland.

the "ink gland" is a feature that is found in which of the following? -squid -octopus -cuttlefish -all of the above -none of the above

-all of the above squid, octopus, cuttle fish have an ink gland at the back of the rectum. it opens by a cut dorsally into the rectum close to the anus. when startled, these animals release a brown or black fluid from the ink gland and make a dark cloud in the water. under this cover they escape from an enemy or approach a prey.

the epiglottis is a small flap that covers the entrance of the larynx. it closes when a person swallows, and remains open at other times to facilitate ventilation. what type of material is the epiglottis made of -muscle -fiber -cartilage -collagen -adipose

-cartilage adipose or fat, would not be able to carry out the function of the epiglottis. neither would muscle.

which of the following mitotic structures is only found in dividing plant cells -nucleus -centrioles -cell plate -plasma membrane -chromosomes

-cell plate

which of the following molecules is found in the plant cell wall? -chitin -peptidoglycans -glycogen -cellulose -silicic acid

-cellulose the cell wall is a protective structure that surrounds the plasma membrane of the plant cell. plant cell walls are made of cellulose. chitin is found in the cell walls of fungi. peptidoglycans are found in the cell walls of bacteria. silicic acid is found in the cell walls of diatoms. diatoms are a type of algae and they are classified as protists. glycogen is not found in any cell walls, rather it is a polymer of glucose molecules that is stored in the liver of animals. animals do not have cell walls.

which of the following structures will NOT be common to mitotic cell of a higher plant? -cell plate -centromere -spindle fiber -centriole -plastid

-centriole centrioles are organelle important in spindle formation during nuclear division. these are commonly found in most animal cells and also in lower plant cells but not in higher plant cells.

the microtubule organizing center of the cell is referred to as the -cytosome -centrosome -cytoskeleton -endoplasmic reticulum -Golgi apparatus

-centrosome microtubules are the largest of the three types of protein filaments that make up the cytoskeleton of the cell. the cytoskeleton gives the cell its shape, and supports the organelles. microtubules are built by the polymerization of a protein that is called tubulin. tubulin is organized into microtubules by the centrosome of the cell. a cytosome or micro body is an organelle in some cells that contains metabolic enzymes. the ER processes proteins after they are made by the ribosome in the cell. the Golgi apparatus organizes or sorts and packages proteins and other substances for export from the cell.

during recombination, crossover occurs at sites on the chromosomes called a -locus -chiasma -centromere -kinetochore -equator

-chiasma

in rare cases, an individual is born with both XX and XY chromosomes in his or her body. this results from the fusion of two separate embryos during early development, this condition is referred to as -hermaphroditism -chimerism -aneuploidy -turner syndrome -down syndrome

-chimerism

which of the following is NOT a limiting factor for photosynthesis under normal conditions -light -CO2 -water in plant cell -chlorophyll content of leaf -none of the above are limiting factors

-chlorophyll content of leaf law of limiting factor was proposed by Blackmann in 1905. he stated that when a process is conditioned as to its rapidity by a number of separate factors, the rate of the process is limited by pace of the slowest factor. but in any case, chlorophyll cannot be the limiting factor for photosynthesis.

bacteria-like organelles that are only found in algae and plants are called -mitochondria -chloroplast -centrioles -A & B -B & C

-chloroplast centrioles are only found in animal cells, in which they produce the spindle apparatus that pulls chromosomes apart during cell division.

which of the following organelles is involved in the storage of energy inside the cell -mitochondria -chloroplasts -vacuole -centrosome -liposome

-chloroplasts in eukaryotic cells there are two organelles that process energy. in plant cells energy is captured from the sun and stored in organelles chloroplasts. although mitochondria produce energy they release it immediately in the form of ATP. in animal cells that do not contain chloroplasts, photosynthesis does not occur and animals store energy in form of lipids or fats. liposomes are synthetic vesicles made from the phospholipids that form the plasma membrane in the cell, and they do not store energy. the centrosome is the organelle that organizes the microtubules that help to form the cytoskeleton. vacuoles are membrane-bound storage sacs that typically store water.

the contraction of gall bladder is due to -secretin -cholecystokinin -gastrin -enterogastrone -none of the above

-cholecystokinin secretin - stimulated by low pH in the duodenum, released from S cells, signaling pancreatic secretion of bicarbonate CCK - stimulated by lipid presence in duodenum, released from signaling release of bile from gallbladder. gastrin - stimulated by protein and various other triggers related to stomach motility and emptying, released from G cells to stimulate parietal cells to release HCl

besides annelid and arthropoda, the metamerism is exhibited by which of the following -cestoda -mollusca -chordata -acanthocephalan -none of the above

-chordata metamerism is exhibited by chordates due to repetition of succession of homologous structures along the anterio-posterior axis.

which one of the following is NOT a muscle cell type? -striated -smooth -skeletal -ciliated -cardiac

-ciliated

which of the following cellular structures is not involved in the transport of proteins out of the cell? -vesicles -Golgi apparatus -endoplasmic reticulum -cilium -plasma membrane

-cilium substances leaving the cells are packaged into vesicles by the golgi apparatus prior to entering the golgi apparatus, proteins are process by the ER. all substances leaving the cell, including proteins, must pass through the plasma membrane. cilia are extracellular protrusions that allow movement and the absorption of nutrients into the cell.

the bacterial chromosome is _______, and the eukaryotic chromosome is __________ -circular; linear -circular; helical -linear; helical -linear; circular -linear; linear

-circular; linear

the functional unit of Golgi complex is -cristal -archoplasm -thylacoid -grana -cisternae

-cisternae cisternae are flattened membrane disks which make up the Golgi complex. the main function is the secretion from a cell of protein materials like enzymes, hormones etc. that are not easily diffusible thought the cell membrane by the process of reverse pinocytosis.

which of the following phases of krebs cycle produces water -cis-aconitate <-> isocitrate -fumarate <-> malate -citrate <-> cis-aconitate -oxaloacetate <-> citrate -malate <-> ocaloacetate

-citrate <-> cis-aconitate in the reaction: citrate <--> cis-aconitate, one molecule of water is removed and is added back to produce isocitrate from cis-aconitate. in the reaction: fumarate to malate, a molecule of water is added. in the other two reactions, water molecules are not produced or added.

the process by which a species splits into two different species is referred to as -biogenesis -anagenesis -cladogenesis -bifurcation -outgrouping

-cladogenesis anagenesis - a species evolves gradually into another species over time. cladogenesis - a species splits into two species bifurcation - the splitting of the main body, such as data, into two parts and it does not describe speciation. outgrouping is the assignment of organisms to a reference group for determining evolutionary relationships among species. biogenesis is the scientific belief that living things only arise from other living things.

the process by which fertilized ovum undergoes rapid mitotic divisions without cell growth is referred to as -capacitation -cleavage -morulation -blastulation -gastrulation

-cleavage after fertilization the ovum goes through a precise set of mitotic cell divisions. although the number of cells increases, the size of the egg does not increase. this process is referred to as cleavage. morulation - is the division of the egg into 32 cells and it precedes blastulation which occurs after cleavage. blastulation - the formation of a hollow ball of spherical cells, and it is followed by gastrulation. gastrulation - three embryonic germ layers form capacitation - the process by which the sperm undergoes further maturation, while it is inside the female reproductive tract prior to reaching the egg.

mitotic cell division of a zygote is called -placentation -gastrulation -cleavage -ovulation -none of the above

-cleavage cleavage is the mitotic cell division of zygote. the zygote undergoes rapid cell cycle and produces a cluster of cells. placentation means formation or arrangement of placenta. gastrulation is the early development phase of animal embryo. ovulation is the process in the menstrual cycle by which ovum is discharged.

if the members of a population are dispersed so that the distance between neighboring individuals is minimized, its distribution is classified as -clumped -random -uniform -regular -none of the above

-clumped in a random distribution, the members of the community are independently dispersed. in a regular distribution, members of the population are equally spaced apart (also called uniform distribution)

which of the following has only one gastrovascular cavity -ctenophora -cnidaria -porifera -protozoa -none of the above

-cnidaria a cnidaria consists of a sac containing a gastrovascular cavity which has a single opening that acts as a mouth or an anus. ctenophores have two body layers. the inner layer, gastroderm, surrounds a cavity acts as a stomach, which is connected to the mouth by a long narrow gullet. porifera or sponges do not have such cavity. protozoa are unicellular.

radial symmetry occurs in which of the following? -porifera and cnidaria -cnidaria and echinodermata -cnidaria and platyhelminthes -arthropoda and mollusca -none of the above

-cnidaria and echinodermata the members of the phylum platyhelminthes arthropoda show bilateral symmetry. the members of phylum echinodermata show radial symmetry in adults but bilateral symmetry in larval stage. cnidaria also shows radial symmetry. the members of the phylum porifera and mollusca don not have a symmetric body.

when dominant and recessive alleles express themselves together, they are called -dominance -co-dominance -pseudodominance -amphidominance -none of the above

-co-dominance in co-dominance, both the genes of an allelomorphic pain express themselves equally in F1 - hybrids. this means that a heterozygote for co-dominant genes exhibits both the characters side by side.

the process in which an animal no longer responds to a stimulus after repeated periods of exposure is called -imprinting -habituation -associative learning -cognition -memory

-habituation imprinting is learned behavior from parents early on in life associative learning (opposite of habituation) the animal learns to connect specific behaviors to specific events or stimuli. cognition is perception or the ability to process what is perceived memory is the ability to recall past events.

a man known to be victim to hemophilia marries a normal woman, whose father was known to be a bleeder. it is expected that -all their children will be bleeders -none of their children will be bleeders -one-third of their children will be bleeders -one-fourth of their children will be bleeders -half of their children will be bleeders

-half of their children will be bleeders 25% daughters and 25% sons will be hemophilic among the children born to the couple 25% daughters and 25% sons will be hemophilic. thus, half of their children will be bleeders.

cells that contain half the number of chromosomes in their nuclei are referred to as -haploid -diploid -sister chromatids -homologous -recombinants

-haploid

acrosome is found in this part of a sperm -head -neck -middle piece -tail -end piece

-head a mature sperm consists of three main regions - head, middle piece, and tail. the head contains the haploid cell nucleus and possesses a cap-like structure called an acrosome.

which of the following carries glucose from the digestive tract to the liver? -pulmonary vein -pulmonary artery -renal portal system -hepatic artery -hepatic portal vein

-hepatic portal vein hepatic portal system is a system of veins leading from the digestive tract to capillaries (sinusoids) in the liver. thus, the absorbed products of digestion (except fat) go straight to liver and not to heart.

a vein that breaks up into capillaries is -pelvic vein -pulmonary vein -renal vein -hepatic portal vein -none of the above

-hepatic portal vein in the circulatory system, a portal vein is formed from capillaries in organs and breaks up into a second set of capillaries in other organs of the body before returning blood to the heart.

xylem and phloem are NOT found in the phylum -pteridophyta -lycophyta -coniferophyta -anthophyta -hepatophyta

-hepatophyta Xylem and phloem are known as vascular tissue. the three phyla bryophyta, hepatophyta and anthocerophyta do not have vascular tissue, true roots, stems or leaves. these are called nonvascular plants. others are vascular plants. they have vascular tissue, true roots, stems or leaves.

in an ecosystem, which is known as 'key-industry' -green plants -herbivorous animals -omnivorous animals -carnivorous animals -bacteria and fungi

-herbivorous animals Charles Elton, one of the founders of ecology, referred to the term 'key-industry' to denote animals that feed upon plants. that is, herbivorous animals which have a very large number of animals that are dependent upon them

in the lac operon, genes that control lactose catabolism are expressed by the presence of lactose in the environment. the lac operon is an example of a(n) _________ system -inducible -suppressive -repressive -allosteric -none of the above

-inducible suppressive system is the opposite. repressive is a term used to describe molecules that block the activity of the promoters for the genes. allosteric compounds are molecules that bind enzymes at locations that are distant to their active sites. they are not directly involved in gene expression.

the process by which embryonic cells begin to differentiate into their final cell types during development is called -maturation -induction -fate -gene expression -localization

-induction

which of the following mechanisms cause the stomata of a plant to open? -efflux of K+ -influx of H+ -influx of Ca2+ -influx of Na+ -influx of K+

-influx of K+ the opening of the stomata is an energy consuming process. it is initiated by the excretion of H+ ions by guard cells and the intake of potassium ions and malic acid

which of the following nucleic acids take part in translation? -only DNA -only mRNA -only tRNA -mRNA and tRNA -DNA and tRNA

-mRNA and tRNA during translation, amino acids are assembled from information encoded in mRNA. as the mRNA codons move through the ribosome, tRNAs add a specific amino acids to the growing polypeptide chain. the process continues until a stop codon is reached and the newly made protein is released.

which of the following bones are not found in the foot? -phalanges -tarsus -calcaneus -metacarpals -talus

-metacarpals the tarsus or tarsal bones form the lower join of the ankle the talus is the second largest of these bones. the calcaneus form the heel bone. the phalanges form the toes. the metacarpals form the middle part of the hand between the phalanges or fingers and the wrist (carpals)

the series of changes from larvae to adult, after embryonic development, is called -growth -aging -regeneration -metamorphosis -none of the above

-metamorphosis metamorphosis is a series of morphological changes from larva to adult after embryonic development.

colchicine blocks which of the following stages of cell division -prophase -anaphase -telophase -metaphase -interphase

-metaphase colchicine specifically bind to tubulin of microtubules. one mole of colchicine binds to each mole of tubulin and as a result of this binding, the microtubules break down. in dividing cells, mitosis is blocked at metaphase because of the breakdown of the spindle microtubules.

in carcinogenesis, the acquired ability of cells to migrate to other tissues and organs of the body is known as -angiogenesis -metastasis -malignance -dysplasia -hyperplasia

-metastasis angiogenesis is the formation of new blood vessels. this is often seen in carcinogenesis. malignance is a term that describes a tumor that is harmful to the body. dysplasia and hyperplasia refer to the increase in the number of abnormal cells and the increase in the number of cells respectively. these are also observed in carcinogenesis.

the stability of microtubules is maintained by which of the following proteins? -spectrin -fimbrin -profilin -microtubule associated proteins -none of the above

-microtubule associated proteins in the assembly of microtubules, 15-20% of different proteins other than tubulin take part. these proteins are called microtubule associated proteins or MAP.

the spindle apparatus is visible during mitosis in the cell. which of the following protein polymers is the material from which the spindle apparatus is formed? -centrioles -microfilaments -intermediate filaments -microtubules -microvilli

-microtubules centrioles are the organelles that produce the spindle apparatus, and they are made up of microtubules. but the questions asked specifically for what material.

the layer found between the cell wall of two cells is called -lysosome -microsome -lomasome -middle lamella -tertiary cell wall

-middle lamella the middle lamella is a thin layer between two adjacent cells and is formed after cell division from the cell plate. it is viscous, colloid, elastic and composed of organic compounds like protein, calcium, and magnesium pectate. it holds the two adjacent cells together.

if we completely remove the decomposers from an ecosystem its functioning will be adversely affected, because -rate of decomposition will be very high -energy flow will be blocked -herbivores will not receive solar energy -mineral movement will be blocked -all of the above

-mineral movement will be blocked the decomposers act on the dead organic matter. they block by breaking them down into simpler compounds and minerals which are available to produce organism i.e. they recycle the minerals

adenosine triphosphate is produced by which organelle of the cell? -nucleus -ribosome -mitochondria -Golgi apparatus -chloroplast

-mitochondria

which of the following organelles contains DNA? -Golgi apparatus -ribosome -lysosome -mitochondria -ER

-mitochondria

which of the following structures has an enzyme that has two major structural parts known as F1 and F0? -chloroplast -mitochondria -dictyosomes -nucleus -ER

-mitochondria each mitochondria is made up of two unit membranes. the outer membrane is smooth and inner membrane has many infoldings, which extends into the matrix. these infoldings are called cristae. the inner surface of the inner membrane is covered by small tennis racket like particles with a head and stalk. these particles have been variously named as elementary particles or F1 particles or electron transport proteins.

enzymes of oxidative phosphorylation occur in -Golgi apparatus -mitochondria -ER -chloroplast -nucleus

-mitochondria oxidative phosphorylation is an ATP generating process which occurs in mitochondria.

which of the following does NOT describe mitochondria -mitochondria have their own DNA -mitochondria generate NADH -mitochondria have a large negative voltage across their inner membrane -mitochondria increase ATP generating in the absence of oxygen -none of the above

-mitochondria increase ATP generating in the absence of oxygen mitochondria cannot generate ATP under anaerobic condition. in this condition, ATP is generated by glycolysis which takes place in cytoplasm.

the rapid deterioration of neurons in the brain leads to nervous system dysfunction and disease. neurons are arrested in the G0 of the cell cycle. what cellular process would researchers like to induce in neurons? -differentiation -mitosis -apoptosis -induction -oncogenesis

-mitosis

gametogenesis involves all of the following except -mitosis -oogenesis -spermatogenesis -oocytogenesis -none of the above

-mitosis gametogenesis is the production of gametes, or egg and sperm cells, in an organism. in males sperm production is caled spermatogenesis. in females, egg production is referred to as oogenesis. oogenesis involves two stages. first is oocytogenesis. this is the development of immature oocytes or egg in the female prior to or shortly after birth. this is followed by ootidogenesis. this is when the oocyte becomes an ootid, an egg that is arrested at prophase I. mitosis is incorrect because gametogenesis occurs by meiosis

the tendon is primarily composed of -adipose tissue -areolar tissue -yellow fibrous connective tissue -modified white fibrous tissue -none of the above

-modified white fibrous tissue tendon is a modified white collagenous fibrous tissue in which fibers occur in thick parallel bundles. tendon connects muscle to the sheath of a bone, so that contraction of the muscle can bring about movement of the bone.

when the developing embryo has divided into 32 cells it is called a -morula -blastocyst -trophoblast -blastula -gastrula

-morula

during cleavage, a ball of 16 cells is formed, the ball of cells is called a(n) -zygote -blastocyst -inner cell mass -morula -embryo

-morula immediately after fertilization, the zygote goes through a series of mitotic divisions called cleavage. it produces a ball of 16 cells called a morula which consists of a group of internal cells surrounded by a larger group of external cells.

Laurent Chabry's experiment in 1887 demonstrated that the blastomeres of developing tunicates were responsible for producing a specific set of larval tissues, and that the blastomeres were developing on their own. The type of development that Chabry uncovered is referred to as -mosaic development -allocation -determination -regulative development -commitment

-mosaic development

ABO blood group is an example of -multiple allele -multiple gene -iso-allele -complimentary gene -none of the above

-multiple allele if more than two alleles occur simultaneously within a species population at a gene locus, then they will be called multiple alleles. an example of multiple allele inheritance is human blood type. blood type exists as four possible phenotypes: A, B , AB, and O. there are 3 alleles for the gene that determines blood type. the alleles are iA, iB, and iO

the axons in the nervous system are insulated by which of the following -adipocytes -insulator fluid -myelin -blubber -hypodermis

-myelin

which of the following is not a component of the cell's cytoskeleton? -microfilament -microtubules -intermediate filaments -myosin -tubulin

-myosin the filaments that make up the cytoskeleton are microfilaments, intermediate filaments, and microtubules. the filaments are made of proteins such as actin and tubulin. myosin does work closely with actin to facilitate muscle contraction, myosin is NOT a component of the cytoskeleton

many species of bacteria have developed resistance to many commonly used antibiotics, this is an example of -fitness -genetic variation -natural selection -genetic linkage -artificial selection

-natural selection

the process by which the bacteria have developed resistance is referred to as -artificial selection -natural selection -gene flow -sexual selection -genetic drift

-natural selection in natural selection, environmental pressures or changes can drive the selection of specific traits or genes in a population. gene flow is the movement of genes in or out of a population by migration. if the distribution of alleles or the genetic composition of a population changes by chance, the process is referred to as genetic drift. in sexual selection, mates are chosen by their phenotypes that are considered advantageous to survival. bacteria are asexual and therefore do not carry out sexual reproduction. artificial selection is also called selection breeding, and it is the practice of intentionally breeding organisms to have specific traits.

which characteristic of peripatus resembles annelids -nephridia present -body is segmented -simple eye -all of the above -none of the above

-nephridia present annelida have coiled paired metamerically arranged nephridia as an excretory organ. in peripatus, nephridia are present in all the legs.

organogenesis in the embryo begins with which of the following organ systems -circulatory -respiratory -integumentary -digestive -nervous

-nervous

the peripheral nervous system is derived from which of the following embryonic structures? -endoderm -neural crest -neural tube -mesenchyme -coelom

-neural crest the neural tube gives rise to the central nervous system. the neural crest gives rise to the peripheral nervous system.

which of the following is regarded as a unit of nerves system? -neurons -axons -dendrites -myelin sheath -ganglia

-neurons neurons are the structural and functional units of the nervous system. a neuron is composed of cell body (cyton), axon, and one or more dendrites where the nerve impulses pass through.

the first compound with two carbon atoms formed in C2 cycle is -glycin -glycolate -glyoxalate -phosphoglycolate -none of the above

-phosphoglycolate in C2 cycle, RuBP is oxidized in presence of the enzyme RuBP - oxygenase to form phosphoglycolate. RuBP + O2 -----RuBP - oxygenase----> 2-phosphoglycolate + 3-PGA + 2H+ + H2O phosphatase

which of the following represents the Michaelis - Menten theory about the activity of enzymes? -E + S <-> P -E + S <-> E + P -E + S <-> ES -> E + P -E + S <-> ES -> P -none of the above

-E + S <-> ES -> E + P according to this theory, at first, enzymes react with substrates and form enzymes-substrate complexes. as a result, the activation energy of the reaction decreases and the reaction occurs easily. in the next stage, the product forms from the intermediate complex and the enzyme becomes free. E = enzyme S = substrate P = product

which organelle connects the cell membrane with the nuclear membrane? -Golgi apparatus -centrosome -mitochondria -ER -microfilament

-ER endoplasmic reticulum is a system of membranous flattened tubes and sacs which are physically connected to a nuclear membrane and go through cytoplasm onto cell membrane.

ribosomes are often found on the outer surfaces of -plasma membrane -nucleus -mitochondria -chloroplast -ER

-ER ribosomes can be free in cytoplasm, but are often found on the outer surface of ER. they are rarely found on nuclear membrane. ribosomes can also be found inside mitochondria and chloroplast.

which of the following is formed in the nucleolus? -DNA -RNA -protein -A & B -A & C

-RNA most cells contain one more more prominent spherical denser part called nucleoli's in their nuclei. a nucleolus has three regions - fibrillar center, a dense fibrillar component and cortical granular components. different stages of formation of ribosomal RNA are completed in these three distinct regions of the nucleolus.

prokaryotes do NOT have -DNA -a ribosome -a plasma membrane -molecular motos -a nucleus

-a nucleus prokaryotes are generally unicellular. they lack a nucleus and other membrane-bound organelles. here, DNA remains in the cytoplasm

during forced expiration, an actively contracting muscle includes the -abdominal muscle -external intercostals -diaphragm -intestinal muscle -C & D

-abdominal muscle expiration is a passive process that does not require the involvement of any muscular contraction. in a forced expiration, the abdominal muscles increase intra-abdominal pressure and pull the rib cage downward and inward.

the behavior of an animal is centered around its -ability to move -intelligence -fitness -egotism -size

-ability to move

the biological fitness of an organism is a measure of its -ability to reproduce -physical strength -survival -intelligence -none of the above

-ability to reproduce

a co-factor (prosthetic group) is a part of a holoenzyme. Which of the following is true of a cofactor? -accessory non-protein substance attached firmly -scattered positive accessory proteinaceous substances -loosely attached organic part -loosely attached inorganic part -none of the above

-accessory non-protein substance attached firmly co-factor is a non-protein substance that helps an enzyme to carry out its activity. it is produced when a catalytically active enzyme forms a complex substance with a co-factor of holoenzyme.

when entering the Kreb's cycle, pyruvic acid is converted into -NADH -glucose -citrate -acetyl coA -ATP

-acetyl coA at the beginning of Kreb's cycle, pyruvate, the ionized form of pyruvic acid, is converted to acetyl CoA and CO2. in glycolysis, glucose is converted to pyruvate. NADH is produced when 3-phosphoglyceraldehyde is converted to 1,3-bisphosphoglycerate. when the Kreb's cycle begins, acetyl CoA and oxaloacetate form citrate. ATP is produced during the formation of pyruvate in glycolysis.

the link between Embden-Meyerhoff-Parnass (EMP) and TCA cycle is -OAA -FAD -acetyl coA -pyruvic acid -none of the above

-acetyl coA in the EMP pathway one molecule of glucose is oxidized to form two molecules of pyruvic acid. before entry in the TCA cycle, pyruvic acid is converted to Acetyl CoA with the release of CO2 through a complicated series of reactions. the TCA cycle, or Kreb's cycle, begins when Acetyl CoA reacts with a 4-C compound, oxaloacetic acid, to form citric acid.

the end product of beta-oxidation is -acyl coA -acetyl coA -fatty acyl coA -beta-keto fatty acyl coA -none of the above

-acetyl coA the metabolic pathway of beta-oxidation: fatty acid --> fatty acyl coA --> alpha-beta unsaturated fatty acyl coA --> beta hydroxy fatty acyl coA --> beta keto fatty acyl coA --> acetyl coA + [fatty acyl coA(shortened by two carbons)]

growth hormone (GH) is produced by which of the following endocrine glands -pituitary -thyroid -adrenal -pancreas -hypothalamus

-pituitary growth hormone (GH) is involved in growth, cellular respiration, and tissue generation. the thyroid released hormones that regulate metabolism. the adrenal gland releases hormones that respond to stress. the pancreas releases hormones that regulate sugar levels in the blood. the hypothalamus produces hormones that affect homeostasis by modifying the blood pressure, heart rate, and body temperature.

the pea-sized gland that is located on a stalk that hangs from the base of the brain is the -adrenal gland -pituitary gland -thyroid gland -thymus -hypothalamus

-pituitary gland

during gestation, the mother nourishes the embryo through -umbilical cord -placenta -chorionic villi -yolk sac -none of the above

-placenta chorionic villi forms placenta through which the mother nourishes the embryo. nutrients, gasses, pathogens etc. pass from the mother to the embryo through the placenta. umbilical cord develops from and contains remnants of the yolk sac and allantois. the embryo is attached to the placenta by the umbilical cord. it contains veins and arteries that carry blood between the embryo and the placenta.

which of the following secretes antibodies? -neutrophils -macrophages -mast cells -plasma cells -none of the above

-plasma cells plasma cells are not distinct cells. they differentiate from B lymphocytes during immune response. synthesis and secretion of antibodies are the major function of plasma cells. main function of neutrophils and macrophages is phagocytosis. mast cells release histamine. all these three also release chemicals involved in inflammation.

a small circular ring of DNA that is used as a vector to introduce foreign DNA into a host cell is referred to as a -plasmid -gene gun -micropipette -SNP (pronounced snip) -primer

-plasmid

which of the following secretes thromboplastin? -monocytes -lymphocytes -erythrocytes -platelets -none of the above

-platelets thromboplastin is required for the clotting of blood. it is released from damaged platelets when skin is cut and blood flows out.

which of the following is considered to be the longest phase of mitosis -prophase -metaphase -anaphase -telophase -A & D

-prophase prophase is the longest phase in mitotic cell division. in this phase chromatin condenses so chromosomes are noticeable. two identical copies of each chromosome, called chromatids, are formed. the nuclear membrane and nucleolus disappear. metaphase takes less time than prophase and the duration of the other two phases are also less than metaphase.

females are born with all of their eggs. these eggs are arrested in which of the following stages of meiosis? -prophase I -prophase II -metaphase I -anaphase I -anaphase II

-prophase I

in mRNA, the exon codes for the production of -lipid -protein -phospholipid -carbohydrate -two of the above

-protein mRNA consists of codons for protein synthesis. exon is the stretch of bases which codes for amino acids.

which of the following pairs are correctly matched? -central dogma - codon -Okazaki fragments - splicing -restriction enzymes - genetic engineering -RNA polymerase - RNA primer -none of the above

-restriction enzymes - genetic engineering restriction enzymes (genetic scissors - for slicing off genes) are used as gene therapy techniques for the treatment of conditions such as diabetes, hemophilia, cancer, and leukemia.

the lymph serves to -return of RBCs to the lymph nodes -return of interstitial fluid to the blood -transport CO2 to the lungs -transort O2 to the brain -none of the above

-return of interstitial fluid to the blood an important function of the lymph is to return interstitial fluid back to the blood. this interstitial fluid is also called extracellular fluid, which is formed from blood due to various factors like hydrostatic pressure, osmotic gradient etc.

which of the following statements is true? -phospholipids have charged hydrophobic head groups -phospholipid have hydrophilic fatty acid tails -phospholipids form lipid bilayer in aqueous environment -phospholipids form droplets within the cytoplasm -none of the above

-phospholipids form lipid bilayer in aqueous environment phospholipids have charged hydrophilic head groups and fatty acid tails that are hydrophobic. phospholipids will not form droplets as this would place the hydrophilic head group in a hydrophobic environment.

an animal's evolutionary history is referred to as its -phenotype -genotype -clade -phylogeny -phylum

-phylogeny

the process in which cells internalize very small molecules or extracellular fluid is referred to as -exocytosis -receptor mediated endocytosis -pinocytosis -phagocytosis -clathrin-mediated endocytosis

-pinocytosis

in receptor-mediated signaling -cell secretes hormones -cell secretes neurotransmitters -two cells bond together -a chemical mediator is used to deliver the signal to a distant cell -none of the above

-two cells bond together there are two types of cell signaling. in chemical signaling, cells secrete a chemical, hormone or neurotransmitter to deliver signals to local or distant cells. in receptor-mediated signaling, a physical contact is needed. in this case, two cells bind together with their receptors or ligands.

in the electron transport system, one molecule of FADH2 produces -one molecule of ATP -two molecules of ATP -three molecules of ATP -four molecules of ATP -five molecules of ATP

-two molecules of ATP oxidation of FADH2 occurs by transfer of their electron through a series of electron carries, and then they finally combine with oxygen to form water and ATP. one molecule of FADH2 produces 2 molecules of ATP.

which of the following enzymes is not involved in deoxyribonucleic acid replication? -helicase -ligase -polymerase -ribonuclease -topoisomerase

-ribonuclease helicase opens up the double stranded DNA strands, which causes it to relax. this makes the base pairs accessible as a template. DNA polymerase adds new nucleotides to the new DNA strands. ligase works as the glue to connect the new strands together. ribonucleases are enzymes that degrade the RNA molecules into smaller components, and they are not involved in DNA replication.

which of the following is not a membrane bound organelle inside the cell? -nucleus -ribosome -mitochondria -Golgi apparatus -vacuole

-ribosome

which of the following is found in both prokaryotic cell and eukaryotic cell? -ribosome -lysosome -centrosome -chloroplast -none of the above

-ribosome both prokaryotic and eukaryotic cell contain ribosomes. for example, each cell of E. coli (prokaryotic cell) contains 10,000 ribosomes and a eukaryotic cell can have 10 million ribosomes. others are not found in eukaryotic cells.

on which of the following cell organelles are new proteins made? -ribosome -nucleolus -smooth ER -vacuole -mitochondria

-ribosome new proteins are made on ribosome. as each tRNA arrives at the ribosome, its amino acid is removed and added to the growing amino-acid chain that will become the protein. this process is terminated when the ribosome arrives at a stop codon in the mRNA which then cause the ribosome to release the newly synthesized protein.

which is NOT enclosed within an envelope consisting of two parallel membranes? -nucleus -mitochondria -chloroplast -ribosome -A and B

-ribosome ribosomes are small, more or less spherical organelles. they are made of proteins and RNA molecules. ribosomes do NOT have a membrane while the other three are developed by two membranes separated by a space.

which of the following organic compounds act as a CO2 receptor of the Calvin Cycle? -ribulose 1, 5 - bisphosphate -ribulose 5 - phosphate -ribulose 1, 5 - diphosphate -ribose 5 - phosphate -any of the above

-ribulose 1, 5 - bisphosphate in the Calvin cycle ribulose 1, 5 - bisphosphate acts as CO2 receptor. CO2 + ribulose 1, 5 - bisphosphate --(RuBisCO)--> diphospho orbital (down arrow) --> 3 PGA

which of the following is the most suitable for the understanding of mitosis in a laboratory -root tip -leaf tip -anther -ovary -none of the above

-root tip root tips have active meristematic zone where cells divide mitotically leading to increase in the length of the roots

post-translational modification of proteins takes place in which of the following organelles? -ribosome -lysosome -rough ER -smooth ER -cytoplasm

-rough ER

secretion of which digestive juice is NOT controlled by any hormone? -bile -saliva -pancreatic juice -A & B -none of the above

-saliva secretion of saliva is not controlled by a hormone. it is entirely controlled by two centers of the nervous system - superior salivary center and inferior salivary center, lying respectively above and below the reticular formation of medulla. two hormones cholecystokinin and secretin control the secretion and flow of bile. they are also responsible for the secretion of pancreatic juice.

members of the kingdom Monera obtain energy using all of the following processes except: -photoautotrophy -chemoautotrophy -saprotrphy -photoheterotrophy -chemoheterotrophy

-saprotrphy

which of the following is not a component of the vertebral column -cervical vertebrae -thoracic vertebrae -coccygeal vertebrae -sartorial vertebrae -lumbar vertebrae

-sartorial vertebrae

the monohybrid genotypic ratio 1:2:1 in a F2 generation indicates -segregation -independent assortment -dominance -incomplete dominance -none of the above

-segregation genotype in monohybrid cross: 1(TT): 2(Tt): 1(tt) this shows that the two genes of a particular character remain uncontaminated while they are inside the organism. during gamete formation, the paired genes segregate and enter different gametes. hence each gamete contains only one of the paired genes which are responsible for a particular character. this is Mendel's first law, Law of Segregation.

globally malaria affects millions of people. individuals who are heterozygous for the sickle-cell anemia trait are immune to malaria infection. an increase in the number of heterozygous individuals in the population due to the conferred protection from the trait is an example of -prevalence -penetrance -resistance -selective pressure -none of the above

-selective pressure

normal cells in the human body divide approximately 70 times before they die. this property of cells is called -senescence -contact inhibition -differentiation -metastasis -immortality

-senescence

in ecological terms, the niche of a population is the -place where the organism lives -geographical area that the organism covers -boundary of area that the organism covers -set of conditions that the organism interacts with -set of conditions and resources the organism uses

-set of conditions and resources the organism uses

a man with a disease marries a normal woman. they have three daughters and five sons. all their daughters have a disease but the sons are normal. the gene of this disease is -autosomal dominant -sex - limited character -sex - linked recessive -sex - linked dominant -none of the above

-sex - linked dominant if the father has the disease and the daughters also have it, then the disease is not sex limited of course, it is sex-linked. in daughters it is expressed even when both XX chromosomes are dominant.

the exchange of gases in the alveoli of the lungs takes place by -simple diffusion -osmosis -active transport -passive transport -none of the above

-simple diffusion in the lungs, the exchange of gases takes place between the air in the alveoli and the blood in the capillaries around the alveoli. this exchange of gases takes place through simple diffusion.

in what step of the Krebs cycle can FAD be reduced but NAD+ cannot be reduced? -succinyl co-A <--> succinate -succinate <--> fumarate -fumarate <--> malate -malate <--> ocaloacetate -none of the above

-succinate <--> fumarate in the presence of the enzyme, succinate dehydrogenase succinate is oxidized to form fumarate. FAD acts as hydrogen receptor. in this reaction the free energy change is not enough and so NAD+ cannot be reduced.

progenitor cells in the skin are classified as which of the following -unipotent -multipotent -pluripotent -totipotent -none of the above

-unipotent progenitor cells are stem cells that give rise to a single type of body cell. the progenitor cells in the skin only give rise to skin cells and are therefore unipotent. totipotent cells give rise to all cell types including embryonic stem cells. pluripotent cells give rise to the thee germ layers. multipoint cells give rise to closely related families of cells.

which base is NOT found in DNA but is found in RNA? -adenine -uracil -thymine -cytosine -guanine

-uracil

in the classification of living systems, viruses are not assigned to a clade because -they do not reproduce -they do not contain nucleic acid -they are obligate intracellular parasites -A & B -all of the above

-they are obligate intracellular parasites viruses are not assigned to clades because they are not considered living organisms. viruses do reproduce and possess nucleic acid in the form of DNA and RNA. however, viruses cannot live freely and require an internal cellular host at all times. therefore they are obligate intracellular parasites.

humans cannot digest cellulose whereas cows and other herbivorous animals can because -they have enzyme cellulase in their stomach -they masticate it well by chewing teeth -they have bacteria in their alimentary canal which digest cellulose -A & B -none of the above

-they have bacteria in their alimentary canal which digest cellulose herbivorous animals do not have an enzyme that digests cellulose. instead, these animals harbor microbes that can digest cellulose. animals like cows have anaerobic bacteria in their digestive tracts which digests cellulose.

species richness is greatness in -tundra -desert -temperate forest -tropical rain forest -temperate grassland

-tropical rain forest a community has more species when it is closer to the equator. therefore, tropical rain forest is the richest in species. climate in tropical regions are more stable than temperate regions, which thus help to increase the number of species. tundra and desert areas are not very suitable for a habitat.

the structural unit of a microtubule is called -tubulin -myosin -myelin -G - actin -none of the above

-tubulin microtubules are formed by a protein unit tubulin. each tubulin molecule consists of two globular polypeptide alpha and beta tubulin. the heterodimers of tubulin molecules are so arranged to form a hollow tube like structure of microtubule. myosin is a protein found in most of the eukaryotic cells and is involved in cell locomotion and muscle contraction G-actin is the subunit of microfilaments myelin is a phospholipid layer surrounding the axon of many neurons

a windy ecosystem that receives low levels of annual precipitation and has frozen soil year-round is referred to as a -desert -grassland -tundra -tropical rainforest -coniferous forest

-tundra

which of the following creates turgor pressure? -mitochondria -vacuole -cell membrane -chromatin -none of the above

-vacuole turgor pressure is the force exerted on the cell wall of a plant cell by the water contained in the vacuole. it is resulted from the osmotic pressure.

in the physiology of the respiratory system, the rate at which gas enters and leaves the lungs is called -internal respiration -external respiration -ventilation -inhalation -exhalation

-ventilation

enzymes are absent in -virus -bacteria -protozoa -fungi -none of the above

-virus viruses do not have enzymes. they only have nucleoprotein. when they enter a living cell, they use the host cell's ribosomes, ATP, enzymes, etc. to reproduce.

primary consumers in the ocean are -phytoplankton -zooplankton -nekton -benthos -none of the above

-zooplankton zooplankton feeds on phytoplankton, the producer, so they are the primary consumers. nekton refers to the collection of actively swimming aquatic organisms in the ocean or lake. benthos' are the organisms which live in the benthic zone of the ocean.

a fertilized egg is referred to as a(n) -oocyte -ovum -zygote -fetus -embryo

-zygote

in human development, totipotent cells are found in which of the following -gametes -zygotes -embryos -fetuses -adults

-zygotes

what is the correct combination of the following terms: a. hyaluronidase b. corpus luteum c. gastrulation d. capacitation 1. acrosomal reaction 2. morphogenetic movements 3. progesterone 4. mammary glands 5. sperm activation

a. 1 b. 3 c. 2 d. 5 the acrosome of sperm secretes hyaluronidase enzyme which neutralizes hyaluronic acid and thus helps the sperm to penetrate the corona radiate and touch the zona pellucid of the ovum. during luteal phase corpus, the luteum begins to secrete large amounts of progesterone and estrogen. gastrulation is the process in which a gastrula develops from a blastula by the inward migration of cells. capacitation is the process through which sperm undergoes physical changes needed to penetrate and fertilize an egg.

which of the following is in correct order from the highest to lowest level of ecological organization? -biosphere -community -ecosystem -population -organism

biosphere > ecosystem > community > population > organism

in which of the following orders does light pass through a light microscope? -objective -specimen -projector -condenser

condenser - specimen - objective - projector light microscopes have a condenser lens to focus light on the specimen. the reflected light from the specimen goes through the objective lens and forms a magnified image which is seen through a projector lens.

the three parts of the small intestine listed in order from the stomach to large intestine are respectively -duodenum -jejunum -ilium

duodenum, jejunum, ilium the part of the gastrointestinal tract between the stomach and the large intestine is the small intestine. it consists of three parts: duodenum, jejunum, ilium respectively from the stomach to the large intestine. the duodenum is the first and shortest part of the small intestine connecting the stomach to the jejunum. the jejunum is the middle of three divisions and lies between the duodenum and ileum. the ileum is the last section of the small intestine.

which of the following represents the correct pathway of glycogenolysis?

glycogen --> (Pi going in) glucose 1-phosphate <------> glucose 6-phosphate --> (Pi going out) glucose glycogenolysis is the process of breaking of glycogen to form glucose, generally in the liver.

the layers of the epidermis, from superficial to deep, are -stratum corneum -stratum granulosum -stratum lucidum -stratum spinosum -stratum basale

stratum corneum -> stratum lucidum -> stratum granulosum -> stratum spinosum -> stratum basale CLGSB crab legs, great stuff bro the epidermis is composed of stratified squamous epithelium. it is stratified into five structural and functional layers. from superficial to deep, these are stratum corneum, stratum lucidum, stratum granulosum, stratum spinosum, and stratum basale. the cells of the first three layers and some portion of stratum spinosum are dead.

which of the following represents the course that semen take to exit the body? -testes -epididymis -vas deferens -urethra

testes -> epididymis -> vas deferens -> urethra sperm move from the seminiferous tubules in the testes to a long coiled tubule attached to the testes, called the epididymis. most sperm remain stored in epididymis and some leave the epididymis and pass through a duct called the vas deferens and exit the body through the urethra.

NADP<sup>+</sup> is hydrogen acceptor of which of the following -photolysis -hexose monophosphate shunt -Krebs cycle -A & B -A & B & C

-A & B during photosynthesis, NADP is reduced to NADPH2 accepting electrons from photosystem-I and H+ released during photolysis of water. NADP+ + 2e- --> NADPe- NADPe- + 2H+ --> NADPH + H+ a single passage of glucose-6-phosphate through the hexose monophosphate shunt oxidizes it to the C5 sugar ribulose-5-phosphate, releasing one molecule of CO2. in this process, two molecules of hydrogen are transferred to NADP+, yielding NADPH.

the transition state complex formed during enzyme catalysis -has a higher energy level than the product -has a higher energy level than the substrate -has a lower energy level than the product -A & B -B & C

-A & B enzymes increase the rate of the reaction, but they do not change the thermodynamics of the chemical reaction. the transition state complex is a high energy intermediate of the substrates (reactants) and products that cannot be physically isolated. the transition state has a higher energy level than both the reactants and products.

the process by which an enzyme is regulated by the binding of the product of its reaction to its active site is referred to as -competitive inhibition -feedback inhibition -allosteric inhibition -A & B -all of the above

-A & B if the product of the metabolic pathway is abundant and binds to the enzyme's active site, this is a form of feedback inhibition. since the product is also competing with the substrate for the enzyme's active site, this is also a form of competitive inhibition. in allosteric inhibition, the inhibitor doesn't bind the enzyme's active site. instead it binds another location on the enzyme and causes an allosteric effect.

which of the following pyramids can be inverted? -pyramid of numbers -pyramid of biomass -pyramid of energy -A & B -A, B, & C

-A & B in an aquatic habitat the pyramid of biomass is inverted. in parasitic food chain, for e.g. an oak tree, the large tree provides food to several herbivorous birds. the birds still support a larger population of ectoparasites leading to the formation of an inverted pyramid of numbers. pyramid of energy is always upright. it is so because at each transfer about 80-90% of the energy available at lower trophic levels is used up to overcome its entropy and to perform metabolic activities. only 10% of the energy is available to next trophic level.

in eukaryotic cells, which of the following introduces cytoplasmic inheritance -plastids -mitochondria -lysosome -A & B -A & B & C

-A & B in eukaryotic cells two cytoplasmic organelles, mitochondria and plastids (in green plants) contain their own genetic materials, i.e. extra nuclear genes or extra chromosomal genes.

photosynthesis occurs in what part(s) of the chloroplasts? -stroma -thylakoid membrane -scleranchyma -A & B -B & C

-A & B photosynthesis occurs in the parenchyma, or living tissues of the plants. the sclerenchyma tissues of the plants are made up of dead cells. photosynthesis is divided into the light-dependent and the dark reactions. the light-dependent reaction takes place the thylakoid membranes and involves the use of sunlight to create chemical energy in the plant cell. the dark reaction which produces stable, energetic compounds such as glucose, takes place in the stroma.

during fertilization of the egg by the sperm, which of the following events takes place to prevent polyspermy -cortical reaction -release of Ca2+ by the egg -release of Ca2+ by the sperm -A & B -A & C

-A & B polyspermy occurs when an egg is fertilized by more than one sperm. inside the egg, changes must be made to prevent polyspermy. this is accomplished by the cortical reaction. during the cortical reaction, enzymes inside the egg are released to break down the receptors on the surface of the egg to which sperm cells bind. during the cortical reaction, Ca2+ is also released by the egg to change the charge of its membrane.

in the developing embryo, the central nervous system is derived from which of the following structures? -ectoderm -neural tube -neural crest -A & B -A & C

-A & B the ectoderm is the germ layer that develops into the nervous system. the neural tube gives rise to the central nervous system. the neural crest gives rise to the peripheral nervous system.

the first 22 pairs of chromosomes in the human genome are referred to as -homologous chromosomes -autosomes -sister chromatids -A & B -B & C

-A & B there are 23 pairs of chromosomes in the human genome. the first 22 pairs are homologous chromosomes. the homologous chromosomes are the same size, carry the same genes and each of a pair was received from one parent. these chromosomes are also called autosomes, because they do not influence the sex of the individual. a chromatid is one of a pair of identical copies of a chromosome. chromatids appear during cell division and the pair is called sister chromatids. the autosomes are not identical

rigidity of cell wall is mainly due to -lignin -suberin -cellulose -pectin -A & B & C

-A & B & C the plant cell wall is composed of a variety of polysaccharides, such as cellulose, hemicelluloses, pectin, lignin, suberin etc., slong with lipids, proteins, and mineral deposits. the cell wall is differentiated into three layers. among these, the secondary cell wall mainly increases wall rigidity. this thicker layer is made up of cellulose and also may contain lignin in xylem cell walls or suberin in cork cell walls. these compounds are rigid and waterproof making the secondary wall stiff.

an individual is born with an extra copy of a chromosome, which raises his or her chromosome number to 47. which of the following describes this condition? -Trisomy -Polyploidy -Aneuploidy -A & B -A & C

-A & C

maternal DNA in the zygote is contributed from which of the following cell structures -nucleus -centriole -mitochondria -A & B -A & C

-A & C

a mother of group O blood type has a child that is also group O blood type. the father could be -O only -AB only -A or B -A or B or O -O or AB

-A or B or O there are three alleles: i, iA, iB for blood groups O, A, and B respectively. iA and iB are dominant over i. so ii people have type O iAi or iAiA people have type A iBi or iBiB people have type B iAiB people have type AB blood group here, the mother has ii type. if the father has iAi, iBi, or ii group, the child may be of ii type. but if the father has iAiB type the child will be of iAi or iBi. so the child cannot have O group.

which of the following structures do protists use to move? -flagella -cilia -pseudopodia -protists are unable to move -A, B, & C

-A, B, & C most protists are able to move at some time during their life cycle some movie with a whip-like structure called the flagella while some others move with cilia. protists like amoeba move by pseudopodia, a temporarily extending structure.

parents with AB blood group can have a child with which of the following types of blood groups -A, B -A, AB -AB -A, B, AB -A, B, AB, O

-A, B, AB

the base pairs in DNA are connected by hydrogen bonds. it takes more energy or higher temperatures to separate the bonds between cytosine and guanine (C-G), than it does to separate the bonds between adenine and thymine (A-T). which of the following is a probable explanation? -A-T pairs have 2 hydrogen bonds and C-G pairs have 3 hydrogen bonds -A-T pairs have 3 hydrogen bonds and C-G pairs have 2 hydrogen bonds -there are more C-G bonds in DNA -the bonds between the A-T pairs are stronger -the bonds between the C-G pairs are weaker

-A-T pairs have 2 hydrogen bonds and C-G pairs have 3 hydrogen bonds

which of the following represents the hormone that is secreted from the posterior pituitary -STH -LH -MSH -GTH -ADH

-ADH ADH and oxytocin are secreted from the posterior lobe MSH from the middle lobe and STH, TSH, ACTH, LH, FSH, GH, and prolactin are secreted from the anterior lobe of the pituitary gland

facultative water reabsorption is controlled by -renin -ADH -ACTH -adrenalin -none of the above

-ADH ADH controls water permeability of principal cells in the collecting and in the last portion of the distal convoluted tubule. when ADH levels are high, a large volume of water is reabsorbed from the tubular fluid into interstitial fluid and the kidneys secrete concentrated urine.

which hormone makes the collecting duct of nephron permeable to water? -ACTH -ADH -FSH -DHEA -renin

-ADH the collecting duct of nephron is normally impermeable to water. but in the presence of ADH, it becomes permeable. it affects the function of aquaporins, the integral membrane protein. as a result, water molecules are reabsorbed as it passes through the collecting duct. aquaporins selectively conduct water molecules while preventing the passage of ions and other solutes.

an mRNA is formed from the DNA chain (TCCAAGTTA) what will be the mRNA strand formed? -AGGUUCAAT -AGGUUCAAU -AGGTTCAAT -AGGUUGAAU -AGGUUGAAT

-AGGUUCAAU

in DNA, when AGCT occurs, their association is like which of the following pairs? -AG - CT -AC - GT -AT - GC -AG - CU -AG - TC

-AT - GC A with T and G with C

the products from the light reaction of photosynthesis are -ATP and H2O -ATP and O2 -ATP and NADPH2 -ATP, H2O and O2 -ATP, NADPH2, and O2

-ATP, NADPH2, and O2

which of the following is not an mRNA stop codon in the genetic code? -AUG -UAG -UGA -UAA -none of the above

-AUG there are three mRNA stop codons in the genetic code and one start codon the three stop codons begin with uracil (U) the start codons begins with adenine (A)

which of the following is a start codon? -UAA -AUG -UGA -UAG -none of the above

-AUG AUG is the start codon which indicated where translation should begin. others are stop codons that indicate where translation should end.

which of the following represents the genotype of an individual, who expresses an autosomal dominant trait that creates a blended phenotype? -AA -aa -Aa -A & C -none of the above

-Aa

inside the cell, the nucleus -controls the cell's activities -has a double membrane -contains chromosomes -A & C -All of the above

-All of the above the nucleus is the largest organelle in the cell, and it controls the cell's activities. DNA is stored inside the nucleus in the form of chromosomes. structurally, the nucleus contains a double membrane.

unicellular organisms exist in all of the below kingdoms EXCEPT which of the following? -Monera -Protista -Animalia -Fungi -none of the above

-Animalia

mitochondria are believed to have evolved from bacteria, and became part of the eukaryotic cell as a result of endosymbiosis, which of the following structures are found in mitochondria that confirm their evolutionary past -nucleus -DNA -ribosomes -A & B -B & C

-B & C

the idea or theory that life can arise from nonliving matter is referred to as -biogenesis -abiogenesis -spontaneous generation -A & C -B & C

-B & C

the network of internal membranes inside the cell is called the -ER -cytoskeleton -Golgi apparatus -microtubules -cytosol

-ER the cell contains a network of internal membranes referred to as the ER. the endoplasmic reticulum sorts and modifies proteins, synthesizes lipids, and transports them to other parts of the cell. the cytoskeleton is a network of filaments that gives the cell its shape, and supports the organelles. the golgi apparatus is made of stacks of flattened vesicles. microtubules are protein components of the cytoskeleton. cytosol is the liquid matriculates inside the cell that holds the organelles in place.

which of the following is needed for erythropoiesis -Mg -Cu2+ -Ca2+ -Na2+ -Fe2+

-Fe2+ formation of RBC is called erythropoiesis. it occurs in the liver and spleen in fetus and in red bone marrow after birth. in the presence of protein and iron, vitamin B12 and folic acid (Fe++) stimulate erythropoiesis.

if mitotic division is restricted in the G1 phase of the cell the condition is known as -G0 phase -G1 phase -G2 phase -S phase -M phase

-G0 phase terminally differentiated somatic cells, such as end cells like neurons and started muscle cells that are no longer divided, are usually arrested in the G1 stage. this type of G1 phase is called G0 phase.

which of the following phases is considered to be the longest in the cell cycle? -S -M -G1 -G2 -both G1 and G2

-G1 G1 phase, the first phase of interphase, is the longest. the growth of the nucleus and cytoplasm, transcription and translation all occur during this phase.

which of the following organelles is often found at the cell center? -nucleus -Golgi apparatus -mitochondria -ER -all of the above

-Golgi apparatus most times the Golgi apparatus is found at the cell center which is adjacent to the nucleus.

the acrosome of a sperm is derived from which of the following cell organelles? -plasma mebrane -ribosome -ER -centrosome -Golgi bodies

-Golgi bodies during testicular maturation, acrosome develops over the anterior half of the sperm's head. it is derived from the Golgi apparatus. in eutherian mammals, it contains many digestive enzymes.

which of the following has the main role in protein synthesis -mitochondria -lysosomes and other vesicles -endoplasmic reticulum -Golgi bodies and ribosomes -plasma membrane

-Golgi bodies and ribosomes proteins are assembled by ribosomes on the rough ER. vesicles transport the proteins to Golgi bodies where they modify protein and put it in new vesicles. these can be thrown out of the cell or they may remain in the cell.

which of the following is NOT an example of an autosomal recessive inheritance -Tay-Sachs disease -sickle cell anemia -phenylketonuria -albinism -Huntington's disease

-Huntington's disease Huntington's Disease is an example of autosomal dominant inheritance. others are due to autosomal recessive disorder carried on the autosomes and are not specific to the sex of the person.

the immunoglobulin that is passed from a mother to her newborn in her breast milk is -IgA -IgB -IgD -IgG -IgE

-IgA remember that IgA is involved with bodily secretions such as tears, breast milk, and saliva (saliva = ahhhhh) IgE involved with allergic reactions (snEeze) IgG involved with invading pathogens (pathoGens) IgD involved with B-cells IgM (not listed) also found on B-cells

the immunoglobulin that functions as a receptor on the surface of B-cells in the immune system is: -IgA -IgB -IgD -IgN -IgE

-IgD B and N are not immunoglobulins

which of the following immunoglobulins is elevated in the body fluids during an allergic reaction -IgA -IgN -IgD -IgM -IgE

-IgE

which of the following antibodies is responsible for causing the allergic reaction? -IgA -IgE -IgM -IgD -IgG

-IgE A: takes part in local protection of mucus membrane E: related to allergic reactions to protect the skin M: forms preliminary protective mechanism D: helps the formation of antibody producing cells G: engulfs virus, bacteria and neutralizes antigen

which of the following antibody is first formed in the body of a new born baby? -IgA -IgD -IgE -IgG -IgM

-IgM

which of the following is used to stain organelle like mitochondria -Azure B -crystal violet -Janus green -Saffranin -none of the above

-Janus green Janus green is a basic dye used in histology and used to stain mitochondria supravitally.

not all eukaryotic organisms contain a single nucleus. the macronucleus is a structure that is observed in members of what kingdom(s): -Protista -Fungi -Monera -A & B -B & C

-Protista members of kingdom Monera do not have a nucleus. members of kingdom Fungi are multinucleated during some of their life stages, but the nuclei are usually the same size. macronucleus is found in ciliates such as paramecium, which belong to kingdom Protista.

the first macromolecule that formed on Earth was -RNA -DNA -oxygen -nitrogen -water

-RNA

the condition in which an individual is born with two female sex chromosomes and one male chromosome (XXY) is referred to as -Turner syndrome -Klinefelter syndrome -Trisomy 21 -Hermaphroditism -Mosaicism

-Klinefelter syndrome nondisjunction is the process in which chromosomes are improperly separated during cell division. this causes chromosomal aberrations in both autosomal and sex chromosomes. if an individual is born with two X chromosomes and one Y chromosome (XXY) he is classified as a male and his condition is termed Klinefelter syndrome. Klinefelter patients develop testes, but the X chromosomes also make them sterile and suppress the expression of testosterone in their tissues. in Turner syndrome, the female is born with only one X chromosome and has developmental complications. Trisomy 21 is Down's syndrome, which is caused by the inheritance of an extra copy of chromosome number 21. hermaphrodites are usually genetically normal. their condition is usually attributed to hormonal disturbances during embryonic development. mosaicism refers to individuals born with two genomes in genetics, and to the process by which embryonic cells undergo pre-patterned development in the developmental embryology.

which is NOT secreted from basophils found in the pituitary gland? -ACTH -LTH -LH -TSH -FSH

-LTH anterior lobe of pituitary consists of two kinds of cells - chromophil and chromophore cells. chromophil cells are of two kinds - acidophils and basophils. all the given hormones secrete from basophils other than LTH, which secretes from acidophils.

the 10% energy transfer law of food chain was given by -Stanley -Lindemann -Weismann -Tansley -none of them

-Lindemann

which of the following is a necessary cofactor of the enzyme hexokinase? -Ni2+ -Mn2+ -Mg2+ -Fe2+ -none of the above

-Mg2+ during glycolysis, hexokinase produces glucose 6-phosphate from glucose by the phosphorylation of glucose molecule. in this reaction Mg2+ acts as a co-factor.

two subunits of ribosome are attached together by -Na+ -K+ -Ca2+ -Mg2+ -Fe2+

-Mg2+ each ribosome is composed of two subunits which remain united with each other due to high concentration of the Mg2+ ions. when concentration of Mg2+ reduces, both ribosomal subunits get separated.

which of the following helps in the movement of cytoplasm? -Cristae -Peroxisome -Ribosome -Microfilament -Golgi bodies

-Microfilament cristae help in ATP production. peroxisomes neutralize oxygen ions, detoxify drugs and break down fatty acids ribosomes build proteins Gogli bodies help in protein synthesis

which of the following phyla has a feature called the radula? -Mollusca -Echinodermata -Annelida -Arthropoda -none of the above

-Mollusca

single-celled organisms that thrive in extreme aquatic environments are classified as members of kingdom -Monera -Protista -Fungi -Plantae -Animalia

-Monera

the presence of a nucleus and cellular division by mitosis is found in all of the kingdoms below except -Monera -Protista -Animalia -Plantae -Fungi

-Monera

in the 1950s, Stanley Miller and Harold Urey performed a landmark experiment, in which they recreated the earth's early atmosphere in a flask. which of the following compounds were not included in their experiment? -H2O -H2 -O2 -CH4 -NH3

-O2

the final electron acceptor in aerobic respiration is -O2 -H2 -CO2 -NADH -H2O

-O2 during aerobic respiration, the hydrogen and electron pass through different electron carriers and finally react with oxygen at the end of the electron transport system (ETS) and form an H2O molecule. thus, the final electron acceptor in respiration is O2.

the end products of photosynthesis are -CO2 and glucose -CO2 and water -O2 and water -O2 and glucose -all of the above

-O2 and glucose

in the female reproductive system eggs are released during which of the following phases of the menstrual cycle? -Granular phase -menstruation -Ovulatory -Luteal -none of the above

-Ovulatory maturation and release of eggs is called ovulation. it is controlled by multiple hormones of the endocrine system. the luteal phase occurs after ovulation, and involves the formation of the corpus luteum which grows on top of the ovary and releases progesterone to prepare the body for pregnancy. during menstruation the lining of the uterus is shed because a fertilized egg wasn't impacted. there is not a granular phase in the female menstrual cycle.

calcitonin is involved in the reduction of Ca2+ in the bloodstream as a part of homeostasis. What is the name of the hormone that works with calcitonin by increasing the amount of Ca2+ in the bloodstream when necessary? -LH -TSH -PTH -oxytocin -insulin

-PTH parathyroid hormone (PTH) is secreted by the parathyroid gland and it increases the amount of calcium in the bloodstream. luteinizing hormone (LH) is produced by the pituitary gland and it regulates ovulation in the female reproductive system. thyroid stimulating hormone (TSH) is also produced by the pituitary gland and regulates the activity of the thyroid gland. oxytocin is another pituitary hormone and it causes uterine contractions during childbirth. insulin is a pancreatic chorine that regulates the amount of glucose in the blood.

a life cycle that includes alternation of generations is found in the members of which of the following kingdoms: -Monera -Plantae -Animalia -A & B -none of the above

-Plantae in the alternation of generations, an organism spends its life in two major stages. the two stages are the gametophyte and the sporophyte. the gametophyte is haploid and contains half the number of chromosomes, while the sporophyte is diploid with the full set of chromosomes. this alternation of generations is observed in plants. bacteria from kingdom Monera have only one body form as do animals.

plasmodium falciparum is the causative agent in malaria. the life stages of P. falciparum include sporozoites, merozoites, and schizonts. P. falciparum belongs to kingdom -Monera -Protista -Animalia -Plantae -Fungi

-Protista

each of the following enzymes is used in creating cDNA libraries EXCEPT one. which one is this EXCEPTION? -DNA ligase -RNA polymerase -reverse transcriptase -DNA polymerase -restriction endonucleases

-RNA polymerase an enzyme, reverse transcriptase uses messenger RNA (mRNA) as a template to copy the sequence of bases in the mRNA into a DNA molecule. this DNA is known as a complimentary DNA (cDNA). DNA polymerase uses the cDNA as a template to synthesize a second strand of DNA. these reactions yield a double-stranded DNA molecule that contains the information (sequence of bases) that was originally in the mRNA. for each gene that produces mRNA, a double stranded cDNA is synthesized. in order to study these cDNA's in more detail, they must be inserted into a plasmid vector and cloned in bacteria. the cDNA molecules are inserted into a plasmid. DNA ligases joins the cDNA with the plasmid DNA. this yields a collection of recombinant DNA molecules. the recombinant DNA plasmids are transferred into bacteria in a process called transformation. the bacteria are grown on an agar medium where they form colonies. each colony of bacteria contains a plasmid with a unique cDNA molecule.

in Hans Driesch's famous experiment, he performed isolation experiments using blastomere cells at the 2, 4, and 8 cell stage from a sea urchin. when cultivated, the 2 cell blastomeres developed into compete larvae, as did some of the later stage cells. Driesch's experiment was the first documented demonstration of which of the following -Mosaic development -allocation -commitment -Regulative development determination

-Regulative development

the kidney is covered by a tough connective tissue capsule called -Malpighian corpuscle -Renal capsule -Glisson's capsule -Bowman's capsule -Pleura

-Renal capsule the kidney is enclosed in a thin, tough, and fibrous whitish capsule called the renal capsule. it is the outermost layer made up of connective tissue.

in which phase does DNA replication occur? -S phase -G1 phase -G2 phase -anaphase -metaphase

-S phase DNA synthesis or replication occurs at the synthesis phase or S phase, the period between G1 and G2 phase in the cell cycle. at the beginning of this stage each chromosome is composed of one coiled DNA called a chromatid and at the end of each chromosome is composed of two sister chromatids.

in which phase of cell division does DNA duplication take place? -S phase -G1 phase -G2 phase -metaphase I -anaphase II

-S phase during the S phase or synthetic phase of interphase, replication of DNA and synthesis of histone proteins occur. at the end of this phase, each chromosome has two DNA molecules and a duplicate set of genes.

histone proteins are synthesized in which of the following phases? -S phase -G1 phase -G2 phase -prophase -A & B & C

-S phase histone proteins are strongly basic and have a low molecular weight since they are soluble in water but insoluble in dilute ammonia. they combine with nucleic acid to form nucleoproteins thus they are synthesized in the S phase.

synthesis of histone occurs in -G1 phase -G2 phase -S phase -prophase -none of the above

-S phase the cell division cycle consists of 4 distinct phases: G1, S, G2 and M phase. during the G1 phase, biological activities that have been slowed down in the M phase start again. synthesis of various enzymes needed for DNA replication takes place in this stage as well. in the S phase, DNA synthesis begins. while rates of RNA transcription and protein synthesis are very low, histone production is high in this phase. in the G2 phase protein synthesis occurs and the phase lasts until the cell enters mitosis.

the pacemaker is -AV - node -SV - node -VA - node -SA - node -none of the above

-SA - node the pacemaker is a small area of specialized cardiac muscle fibers in the wall of the right atrium in the heart of mammals. its spontaneous rhythmical electrical activity activates and maintains contraction of the heart. it is also called SA - node.

the nerve fibers of the peripheral nervous system are covered by myelin sheath producing cells called -dendrocytes -oligodendrocytes -Schwann cells -microglia -none of the above

-Schwann cells in the CNS (brain and spinal cord) cells called oligodendrocytes wrap their branch-like extensions around axons to create a myelin sheath. in the nerves outside of the spinal cord, Schwann cells produce myelin.

the cell-mediated component of the immune system is regulated by which of the following? -B cells -T cells -macrophages -erythrocytes -monocytes

-T cells the cell-mediated branch of the immune system is part of the adaptive immune system, which produces "memory" or immunity towards a pathogen. the adaptive immune system is divided into the humoral and cell-mediated responses. the humoral response is controlled by B cells by their production of antibodies. the cell-mediated response is governed by T cells, which guide and interact with all of the other cell types in the immune system. macrophage are large white blood cells that engulf bacteria and other foreign particles that enter the body. monocytes are immature macrophage. erythrocytes are red blood cells, and they are not components of the immune system.

an animal with genotype TtRr can produce the gamete(s) -T, t, R, r -TR, tr, -Tr, tR -Tt, Rr -TR, Tr, tR, tr

-TR, Tr, tR, tr

which one of the following triplet codes is correctly matched with its specificity for an amino acid in protein synthesis or as a 'start' or 'stop' codon? -UUU - leucine -UAC - tyrosine -UGG - stop codon -AUU - start codon -GGG - alanine

-UAC - tyrosine

map distance of two genes a and b is 10 map units. percentages of crossover gametes are -ab - 5% -ab - 10% -a - 10%, b - 10% -a - 5%, b - 5% -a - 10%, b - 5% or a - 5%, b - 10%

-a - 5%, b - 5%

lichens represent a symbiotic association between -a fungus and alga -a fungus and mold -a fungus and a plant -a fungus and a rhizoid -a plant and a rhizoid

-a fungus and alga

in eukaryotic cells, mRNA processing involves all of the following except: -addition of a 5' cap -addition of a 3' poly-T tail -intron removal -alternative splicing -none of the above

-addition of a 3' poly-T tail in eukaryotic cells the mRNA is processed before it leaves the nucleus. first non-coding regions of DNA called introns are removed. depending on the gene that needs to be expressed, alternative splicing might occur to ensure that the correct gene is produced. splicing is the removal of some eons or coding regions in an mRNA transcript, to create alternate version of the gene. a 5' cap of a single nucleotide is added to the 5' end of the mRNA, to single that it is ready for export from the nucleus. the addition to the 3' end of the mRNA transcript is a poly-A tail, not a poly-T tail.

cellular metabolism is dependent on endergonic and exergonic reactions. what is the name of the molecule that acts as an energy carrier between endergonic and exergonic reactions in the cell? -adenosine monophosphate -adenosine diphosphate -adenosine triphosphate -glucose -glucose 6-phosphate

-adenosine triphosphate

all of the following glands have endocrine and exocrine portions EXCEPT which of these exception? -adrenal -testis -pancreas -ovary -none of the above

-adrenal the adrenal gland does not act as an exocrine gland. others are mixed glands which act as both endocrine and exocrine gland.

which of the following endocrine glands does not release hormones that are involved in digestion -adrenal -pancreas -thyroid -parathyroid -none of the above

-adrenal the pancreas produces insulin and glucagon which regulate the amount of glucose in the blood. the thyroid gland secretes hormones that are involved in metabolism and calcium concentration in the blood. for example, calcitonin is produced by the thyroid, and it decreases the amount of calcium in the blood. the parathyroid gland produces parathyroid hormone (PTH) which increases the amount of calcium in the blood. the adrenal gland produces many hormones but they are not involved in digestion.

the process of sperm attachment on the wall of ovum is called agglutination -agglutination -spermeation -penetration -implantation -none of the above

-agglutination when the sperm gets close to the ovum, antifertilizin is secreted from the sperm and fertilizin is secreted from the ovum. as a result, the sperm attached to the ovum wall. this is called agglutination.

in human females, the eggs are -macrolecithal -mesolecithal -alecithal -microlecithal -none of the above

-alecithal alecithal eggs have no yolk or have an extremely small amount of yolk. human eggs, rabbit, and elephant eggs, are of this kind.

thallophyta includes -fungi and bacteria -algae and fungi -algae, fungi and lichens -algae, fungi, bacteria and lichens -only fungi

-algae, fungi, bacteria and lichens thallophyta are plant-like but not true plants. their body is not differentiated into roots, stems, and leaves but is composed of mass of cells called thallus. thallphyta include algae, fungi, lichens and occasionally bacteria.

which of the following does the endoderm form -duct of gland -alimentary canal -muscular tissue -nerve tissue -blood cell

-alimentary canal as a result of gastrulation, the embryo contains three germ layers: the ectoderm the endoderm the mesoderm the endoderm is on the inside of the embryo. it forms the lining of the digestive tube and the respiratory system.

find the correct pair: 1. diencephalon - third ventricle 2. spinal cord - cauda equina 3. foramen of Luschka - fourth ventricle -1 and 2 are correct, 3 is wrong -2 and 3 are correct, 1 is wrong -1 and 3 are correct, 2 is wrong -1 is correct, 2 and 3 are wrong -all are correct

-all are correct the ventricular system of brain comprises of four ventricles. the right and left ventricles, third ventricle and fourth ventricle. developmentally the third ventricle represents the cavity of the diencephalon. lateral aperture of fourth ventricles is called foramen of Luschka. the cauda equina is a structure within the lower end of the spinal column of most vertebrates that consists of nerve roots and rootlets from above.

which of the following enzymes is produced by the salivary glands -lysozyme -amylase -lipase -A & B -all of the above

-all of the above the salivary glands produce enzymes that destroy bacteria and digest macromolecules. lysozyme is produced by the salivary glands and it destroys the cell walls of bacteria. amylase partially digests starches that enter the mouth. starch digestion is completed in the small intestine which also contains amylase that was produced in the pancreas. lipases which are enzymes that digest lipids or fats are also produced by the salivary glands as well as the pancreas, which sends it to the small intestine.

in a cross of pure white-flowered (recessive) and pure purple-flowered (dominant) plants what will be the color of the flowers of F1 generation? -all purple -all white -50% white and 50% purple -25% white and 75% purple -75% white and 25% purple

-all purple suppose the genotype of dominant purple is RR and that of recessive white is rr. in F1 generation the genotype of all flowers would be Rr. since the purple is dominant character, all flowers will be purple but hybrid.

enzymes are a polymer of -monosaccharide -nucleotide -amino acid -fatty acid -none of the above

-amino acid enzymes are globular proteins consisting of amino acid molecules held together by peptide bonds.

in an egg, the type of cleavage is determined by the -number of egg membranes -amount and distribution of yolk -shape and size of the sperm -size and location of the nucleus -all of the above

-amount and distribution of yolk the amount of yolk and its distribution determine the type and depth of cleavage. cleavage is affected due to yolk resulting in different types of segmentation or cleavage.

holoblastic cleavage occurs in the embryo of -birds -reptiles -amphibians -fish -none of the above

-amphibians in holoblastic cleavage, blastomeres are completely separated and the whole egg participates in cell division. it is typically seen in smaller eggs containing moderate to sparse yolk. cleavage of the embryo of amphibians are radially symmetrical and holoblastic. this kind of cleavage also occurs in the embryo of mammals, echinoderms, annelids, mollusks, etc. the zygotes of birds, reptile, or fish contains large amount of yolk and hence incomplete cleavage or meroblastic cleavage occurs.

which of the following animals has a notochord throughout its entire life? -amphioxus -snake -bird -fish -mammal

-amphioxus in an amphioxus, the notochord extends into the head beyond the brain, and so it is called a cephalochordate. all three important chordate characters exists throughout its life.

fertilization occurs generally in the -uterus -vagina -ampulla -isthmus -none of the above

-ampulla there are four regions of the fallopian tube from the ovary to the uterus. the second region is the ampulla which is an intermediate dilated portion that curves over the ovary. it is the usual site of human fertilization.

polysaccharide is NOT included in the diet chart of a new born baby because -chewing is difficult due to the absence of teeth -concentration of HCl is very low in stomach -amylase is absent in pancreatic juice -absorption of glucose does not occur due to high density of bile salt -none of the above

-amylase is absent in pancreatic juice amylase divides polysaccharides into glucose and maltose. pancreatic juice in adults contains amylase but pancreatic juice of a new born baby does not and results in the indigestion of polysaccharides.

cellulose, the most important constituent of the plant cell wall, is made up of -an unbranched chain of glucose molecules linked by alpha - 1, 4 glycosidic bond -an unbranched chain of glucose molecules linked by beta - 1, 4 glycosidic bond -a branched chain of glucose molecules linked by alpha - 1, 6 glycosidic bond at the site -a branched chain of glucose molecules linked by beta - 1, 4 glycosidic bond in straight chain -branched chain of glucose molecules linked by alpha - 1, 4 glycosidic bond

-an unbranched chain of glucose molecules linked by beta - 1, 4 glycosidic bond cellulose is a polysaccharide, forming the framework of the cell walls of all plants. the individual units of cellulose are a beta-1, 4 linked d-glucose molecule, having a glycosidic bond.

in which of the following phases of mitosis do the chromosomes look like the letters 'V', 'L' or 'J' -prophase -metaphase -anaphase -telophase -interphase

-anaphase during anaphase, the duplicated centromeres of each pair of sister chromatids separate. they begin to move toward opposite poles of the cell. depending on where the centromere is located along the chromosome, a characteristic shape like 'V' (metacentric), 'L' (sub-metacentric), 'J' (acrocentric) appears.

in meiosis, homologous chromosomes separate during -prophase I -metaphase I -anaphase I -anaphase II -telophase I

-anaphase I during prophase I, homologous chromosomes join laterally to form bivalents and rains attached to each other. in this last stage of prophase I bivalents are completely separated. during metaphase I, homologous chromosomes become arranged at the equatorial plate. during anaphase I, homologous chromosomes are separated from one another. during telophase I, chromosomes reach the poles. during anaphase II, the sister chromatids of each chromosome are separated from each other.

In which phase of meiosis do tetrads form? -prophase I -metaphase I -anaphase I -telophase I -none of the above

-anaphase I in this phase, each pair of homologous chromosomes move to opposite poles though the sister chromatids remain together. thus, the number of chromosomes decreases.

in meiosis, homologous chromosomes are separated in which of the following stages? -metaphase I -anaphase I -anaphase II -telophase -diakinesis

-anaphase I separation of homologous chromosomes takes place at anaphase I. during this phase the movement of chromosomes of a bivalent from equatorial plate towards opposite poles takes place.

at which stage of meiosis do the centromeres separate -telophase I -anaphase I -anaphase II -metaphase II -prophase II

-anaphase II in anaphase II, the centromeres split into two chromosomes. two chromatids of each chromosome start moving away from each other and finally reach the poles of the spindle.

which of the following divisions of plants includes land plants that produce flowers? -mosses -bryophytes -gymnosperms -angiosperms -pterophytes

-angiosperms the first plants were bryophytes that did not contain any vascular structures or true roots. bryophytes such as mosses reproduced by sperm cells that had to swim in water to reach the eggs. the next level of plants that evolved were the pterophytes that include ferns. they have a vascular system and true roots. after the pteriphytes, the gymnosperms or seed producing plants evolved. these are the large land plants that include the conifers that bear seeds as cones. flowering plants evolved last, and they are capable of internal fertilization due to the presence of the flower that contains male and female parts. these plants are called angiosperms.

multicellular organisms in which the cell wall is absent include which of the following -bacteria -algae -plants -animals -B & D

-animals cell walls are found in all cell types, except for cells of organisms in kingdom Animalia. bacteria is not multicellular. algae are multicellular but contain a cell wall made of polysaccharides and glycoproteins. plants are multicellular but contain a cell wall made of cellulose.

which of the following hormones makes the collecting ducts in the kidneys more permeable to water -erythropoietin -antidiuretic hormone -aldosterone -cortisol -estradiol

-antidiuretic hormone erythropoietin is produced by the kidneys but it is not involved in excretion. it stimulates the production of red blood cells. if the collecting ducts in the kidneys increase their permeability to water, the urine is more concentrated or less dilute. this process is triggered by antidiuretic hormone (ADH). aldosterone triggers the absorption of sodium in the kidneys, and raises the blood pressure. cortisol is a stress hormone that promotes the release of sugar into the blood stream. estradiol is the form of estrogen that is present in the body during the reproductive years.

in the embryogenesis of vertebrates, the blastopore becomes the -mouth -anus -nostril -vanishes in due course -none of the above

-anus the blastula develops a pore or hole at one end and is called a blastopore. the blastopore becomes the "anus" in the developed animal, those animals will be known as deuterostome, which include vertebrates. the blastopore becomes the "mouth" in animals which are known as protostomes, which include insects, worms, mollusks, etc.

during the development of the embryo, many cells die naturally as the tissues and organs are formed. Research oncologists would like to use the mechanism that drives the removal of these cells to develop remedies and treatments for patients. what is the name of this mechanism? -oncogenesis -differentiation -apoptosis -necrosis -determination

-apoptosis

cardiac muscle fibers -are multinucleated -contain fast and slow twitch fibers -are found in most organs in the body -are striated -none of the above

-are striated there are three types of muscle cells that form the muscle fibers. smooth muscle fibers are found in mist organs in the body, while cardiac muscle is only found in the heart. each cardiac muscle cell has a single nucleus. fast and slow twitch fibers are found in skeletal muscles that can function under anaerobic and aerobic conditions. the parallel arrangement of the fibers i both cardiac and skeletal muscle creates striations or lines that are visible in the muscle.

in a plant cell, tonoplast are found at which of the following sites? -in the nucleus -around the matrix of a plastid -in the perimitochondrial space -over the membrane of ER -around the large central vacuole

-around the large central vacuole tonoplast is the membraneous covering found in mature plant cells around the large central vacuole. proteins found in the tonoplast control the flow of water into and out of the vacuole through active transport.

the blood pressure of the cardiovascular system is measured in which of the following types of blood vessels? -arterioles -arteries -veins -B & C -all of the above

-arteries

in the energy pyramid for trophic levels in an ecosystem, the greatest amount of energy is found -at the top of the pyramid -in the middle of the pyramid -throughout the pyramid -at the base of the pyramid -it depends on the ecosystem

-at the base of the pyramid the four trophic levels are: -producers -primary consumers -secondary consumers -tertiary consumers producers make their own food and therefore harness the greatest amount of energy from the sun. these are at the bottom of the pyramid. primary consumers feed on producers, secondary consumers feed on primary consumers, tertiary consumers feed on secondary consumers.

a cross between hybrid and its parent is a -test cross -monohybrid cross -dihybrid cross -back cross -reciprocal cross

-back cross in Mendelian inheritance, the F2 offsprings are obtained by self - pollination in the F1 hybrids. but the F1 hybrids can be crossed with either of the two parents. such crosses between offspring (hybrid) and parents are known as back cross.

the joint of the femur with the pelvic girdle is of which type -hinge -pivotal -ball and socket -saddle -none of the above

-ball and socket here the femur forms a ball-like head that fits into a socket in the pelvic girdle. the bone with a head (here it is the femur) can move nearly in all directions.

which of the following types of white blood cells generates histamine? -monocytes -natural killer cells -lymphocyte -basophil -neutrophil

-basophil histamine is involved in local immune response and in regulation of some physiological functions. most histamines in the body are generated in granules in mast cells or in basophils.

John was born with blue eyes, but both of his parents have brown eyes. What is John's genotype for the alleles that control eye color -BB -bb -Bb -A or B -none of the above

-bb

phylogenetic trees of life are generally based on all of the following except: -the fossil record -behavior -morphology -embryonic development -DNA

-behavior

the ocean bottom is known as -photic zone -aphotic zone -benthic zone -neritic zone -pelagic zone

-benthic zone the zone in the ocean that receives sun light is known as the photic zone the zone in which does not receive sunlight is known as the aphotic zone the neretic zone is the zone over the continental shelf lastly, the open ocean is known as the pelagic zone, and the ocean bottom is known to be the benthic zone.

insulin is produced in which region of the pancreas -alpha cells -beta cells -delta cells -cortex -medulla

-beta cells insulin is produced by the Islets of Langerhans that contain the beta cells. the beta cells secrete insulin. insulin promotes the uptake of glucose from the blood. the alpha cells secrete glucagon, which works against insulin to promote the release of glucose into the bloodstream. the delta cells produce somatostatin, which is a hormone that inhibits both insulin and glucagon. the pancreas is not divided into a cortex and medulla. organs such as the kidneys have such regions.

the tip of the ecological pyramid is occupied by -planktons -carnivores -producers -decomposers -herbivores

-carnivores in the ecological pyramid, the first level, or producer level, constitutes the base for successive consumer levels. its apex constitutes the top consumer which are carnivores.

the complete oxidation of fatty acid occurs through metabolic pathway of -beta oxidation and TCA cycle -beta oxidation and ETS -beta oxidation and glycolysis -beta oxidation, TCA cycle and ETS -beta oxidation, TCA cycle, ETS and glycolysis

-beta oxidation, TCA cycle and ETS fatty acid produces through Acetyl CoA by the process of beta-oxidation which is then completely oxidized in TCA cycle forming CO2. NADH2 and FADH2 are produced in beta oxidation and TCA cycle. they are oxidized through ETS and form H2O. thus, the oxidation of fatty acid is completed.

Zea mays is the scientific name for corn, the scientific name is also called the -epithet -binomial name -order -clade -A & B

-binomial name

when the ovum reaches the uterus in human fertilization, which of the following phases is the embryo in? -just after fertilization -morula -blastocyst -during cleavage -none of the above

-blastocyst after fertilization, mitotic cell division, known as cleavage, takes place and produces a ball of cells, called morula. cells of the morula divide and release a fluid, resulting in a blastocyst, and during this time it reaches the uterus.

the correct order of developmental stages for the young zygote is -organogenesis, blastulation, gastrulation -blastulation, gastrulation, organogenesis -gastrulation, blastulation, organogenesis -organogenesis, morulation, gastrulation -gastrulation, morulation, organogenesis

-blastulation, gastrulation, organogenesis morulation is the formation of a 32 cell mass from the cleavage of the fertilized egg. at this stage, the size of the egg has not increased, although the number of cells has increased. the first recognizable stage of the zygote occurs after blastulation, during which a fluid filled cavity of cells is formed. next, gastrulation occurs, when the blastula changes into a structure with three germ layers. this is followed by organogenesis in which the germ layers begin their differentiation into specialized tissues and structures.

fresh water bony fishes maintain water balance by -excreting salt across their gills -excreting in form of uric acid -excreting a hypotonic urine -excreting excess water via gills -both C and D

-both C and D osmotic regulation is important to all animals. most of them have to balance their water and salt intake and their exertion to maintain the normal concentration in body fluids. fresh water fishes never drink water. they take in salts by their gills and pass a hypotonic urine.

the genetic identity of a human male is determined by -autosome -X - chromosome -Y - chromosome -both X and Y chromosome -nucleolus

-both X and Y chromosome there are two types of sex chromosomes - the X and Y chromosomes. the constitution of XY denotes males, where as XX denotes females among human beings.

which of the following is not a component of the upper respiratory tract? -nose -bronchioles -pharynx -larynx -epiglottis

-bronchioles upper respiratory tract includes the nose, mouth, pharynx, larynx, epiglottis, and the trachea. the lower respiratory tract includes the bronchi, the bronchioles, and the lungs. the bronchi and bronchioles form the bronchial tree, which leads to the lungs. the bronchioles are smaller branches of the tree.

the bundle of muscle fibers which carries the cardiac impulse from the right auricle to the ventricle is called -S - A node -A - V node -bundle of His -fibers of Purkinje -junctional fibers

-bundle of His the Bundle of His carries the cardiac impulse from right auricle to ventricle. the S - A node sets off cardiac impulses, causing both auricles to contract simultaneously. junctional fibers then carry this impulse to the A - V node and it comes to ventricles through Bundle of His. it then passes through Purkinje fibers causing the contraction of vesicles.

old and abnormal RBC are destroyed -by ornithine cycle in liver -by the reticulo-endothelial cell in the spleen -by the filtration in the glomerulus of the kidney -all of the above -none of the above

-by the reticulo-endothelial cell in the spleen the aging erythrocyte undergoes changes in its plasma membrane, making it susceptible to recognition by phagocytic cells (mainly monocytes and macrophages) of reticulo-endothelial system of the spleen. ornithine cycle or urea cycle is the cycle of biochemical reaction occurred in the liver and produces urea from ammonia. glomerulus filters water and solutes from the blood.

the middle lamella which is found in plants consists of -lignin -suberin -calcium pectate -cellulose -all of the above

-calcium pectate the middle lamella is made up of a layer of calcium pectate (pectin) which runs between the adjoining primary cell walls. it is a thin layer of material that binds adjacent plant cells walls together. (pectin)

the blood vessels through which food and oxygen are released to the cells in the body are the -capillaries -venules -arteries -arterioles -veins

-capillaries there are two types of blood vessels, those that carry blood away from the heart and those that return blood to the heart. arteries carry blood away from the heart. the arteries branch into the arterioles, which are smaller. the arterioles branch into the capillaries, which are the smaller of the three. the capillaries release oxygen and nutrients into the tissues. after releasing the oxygen and nutrients through the capillaries, the blood enters the venules that are also very small. the deoxygenated blood enters the veins that return it to the heart.

in which of the following muscle types can an intercalated disc be found in? -smooth muscle -cardiac muscle -red muscle -white muscle -none of the above

-cardiac muscle the adjacent cardiac muscle fibers in the heart wall are linked by the intercalated disc, an undulating double membrane that appear as thick dark lines within cardiac muscle fibers when they are viewed through a compound microscope.

which of the following does NOT describe a co-enzyme? -it is a non-protein -it is required by an enzyme for its activity -co-enzymes are often vitamins or made from vitamins -co-enzymes are recycled as part of metabolism and remain unchanged after the reaction -co-enzymes are destroyed at 50C-60C

-co-enzymes are destroyed at 50C-60C co-enzymes are non-protein molecules. these are substrates for enzymes and do not form a permanent part of the enzyme's structure. vitamins work as co-enzymes to increase the enzyme activity. in metabolism, co-enzymes are involved in group transfer and redox reaction and are recycled continuously in the process. unlike enzymes, co-enzymes can cope with temperature more than 50C-60C.

enzymes remain in the cell as -crystal -gas -colloid -solution -none of the above

-colloid an enzyme is a protein that speeds up metabolic reactions without being permanently changed or destroyed. it remains in the cell as colloid.

a collection of animal and plants that live in a specific habitat is referred to as a -ecosystem -population -community -niche -food web

-community ecosystem - the collection of living organisms and nonliving entities in a specific biological environment. population - a collection of members of a species that live in the same ecosystem. niche - a species' specific role in the ecosystem. food web - describes the feeding connections between the organisms in an ecosystem.

the scientific discipline that studies the mechanisms that create, maintain, and determine the fate of plant and animal populations that live in a specific habitat is called -population ecology -community ecology -speciation -biodiversity -evolution

-community ecology

in ________, the activity of the inhibitor is reduced by increasing the concentration of the substrate -competitive inhibition -non-competitive inhibition -allosteric inhibition -uncompetitive inhibition -none of the above

-competitive inhibition

which of the following describes double-stranded DNA? -similar and parallel -similar but anti-parallel -similar and can be parallel or anti-parallel -complementary and parallel -complementary and anti-parallel

-complementary and anti-parallel DNA is made of two long strands of repeating subunits called nucleotides. these two parallel strands run in opposite directions to each other and so they are anti-parallel. each type of base on one strand forms a bond with just one type of base on the other strand. for example, cytosine on one strand pairs would pair with guanine on the opposite strand and adenine pairs with thymine. this is called complementary base pairing.

the cell wall shows -complete permeability -semi-permeability -differential permeability -impermeability -varies from cell to cell

-complete permeability cell wall is the non-living, permeable, external, rigid boundary to the plasma membrane in plant cell. due to its completely permeable nature, water can easily diffuse in and out through it.

bone is classified as which of the major types of tissues in the body -epithelial tissue -connective tissue -endoderm tissue -mesoderm tissue -none of the above

-connective tissue four major types of tissues in the body: -epithelial -connective -muscle -nervous epithelial tissues line the surface and cavities of the body. connective tissues like bone support and protect the organs and other tissues. endoderm and mesoderm are two of the three germ layers found in the developing embryo.

the main function of Henle's loop is -the formation of urine -the passage of urine -hormonal secretion -conservation of water -filtration of blood

-conservation of water Henle's loop is a U-shaped segment of nephron coated in the renal medulla. Henle's loop plays an important role in the conservation of water during a counter-current mechanism.

which of the following is NOT character of "fast twitch" muscle type? -striations are more prominent with prominent H zone -contains less capillary network -well-developed T system -myosin ATPase activity is more -contains more myoglobin

-contains more myoglobin according to the rate of response, the skeletal muscle is of two types - a fast twitch and slow twitch muscle. contraction of fast twitch muscle is fast and short lasting. it has less myoglobin and so less oxygen is stored.

the transfer of heat by fluids from one place to another is referred to as -evaporation -radiation -conduction -convection -dissipation

-convection

which of the following is NOT true? -neritic zone extends up to the end of the continental shelf -photosynthesis cannot occur in aphotic zone -water in neritic zone is rich in plankton -fishes in aphotic zone have large jaws and teeth -coral reefs form in aphotic zone

-coral reefs form in aphotic zone neritic zone extends up to the end of the continental shelf. in tropical areas coral reefs are formed in this zone. photic zones extend up to a few hundred meters below sea level. in this zone sunlight can penetrate and so photosynthesis occurs. below this zone is the aphotic where light cannot penetrate and so photosynthesis cannot occur. neritic zone is photic and therefore rich in plankton and lots of small organisms. due to near freezing temperatures and enormous pressure, the fish in the photic zone have large jaws and teeth.

after ovulation, Graafian follicle regresses into the -corpus luteum -corpus artesia -corpus albicans -corpus callosum -none of the above

-corpus luteum the corpus luteum is the yellowish body that forms in the cavity of a Graafian follicle in the ovary of a mammal after the release of the egg cell (ovulation).

a reaction in which the energy that is released by an exergonic reaction powers an endergonic reaction is referred to as a(n) -anabolic reaction -coupled reaction -catabolic reaction -metabolic reaction -A & C

-coupled reaction inside the cell, energetic reactions are also called metabolic reactions. in anabolic reactions, the cell uses energy to create larger molecules. anabolic reactions are therefore endergonic. in catabolic reaction, energy is released into the cellular environment when larger molecules are broken down. these two processes cumulatively called metabolic reactions. if a reaction occurs in which the energy released is used immediately to power an anergy-needing reaction, the reaction is couples. most coupled reactions in the cell are driven by the ATP molecule.

the photosynthetic organisms that converted the earth's early atmosphere from a reducing one to an oxidizing one were the -red algae -brown algae -green algae -bryophytes -cyanobacteria

-cyanobacteria reducing atmosphere means it contained very little oxygen. only prokaryotic cells were alive at this time. red algae, brown algae, green algae are made up of eukaryotic cells. bryophytes are nonvascular plants, and these organisms also contain eukaryotic cells.

for which of the following enzymes does iron acts as a cofactor -cytochrome oxidase -dinitrogenase -urease -carbonic anhydrase -none of the above

-cytochrome oxidase cytochrome enzymes are needed in aerobic respiration. in the oxidized state, this enzyme contains a Fe3+ ion and is called ferricytochrome. in the reduced state, it contains a Fe2+ ion and then it is called ferrocytochrome.

somatostatin is secreted from which cell of islets of the Langerhans -alpha cells -beta cells -delta cells -epsilon cells -PP cells

-delta cells five different types of cells of islets of Langerhans produce different hormones. alpha cells - glucagon beta cells - insulin and amylin delta cells - somatostatin epsilon cells - ghrelin PP cells - pancreatic polypeptide

all of the following are steps in DNA transcription except -initiation -denaturation -elongation -termination -none of the above

-denaturation

which of the following pairs is correct? -ammonification - rhizobium -denitrification - thiobacillus -nitrification - bacillus mycoides -nitrogen fixation - nitrobacter -none of the above

-denitrification - thiobacillus some microbes like micrococcus, bacillus mycoides convert organic nitrogen to ammonium by the process of ammonification. some bacteria like nitrosomonas, nitrobacter can oxidize ammonium to nitrite and then to nitrate by the processionals called nitrification. denitrification occurs when bacteria, like thiobacillus, reduce nitrate or nitrite to nitrogen gas. bacteria, like rhizobium, take part in nitrogen fixation. they can convert the atmosphere's nitrogen gas to ammonia. ammonification - bacillus mycoides nitrification - nitrobacter nitrogen fixation - rhizobium

if the growth rate of a population relies on the birth and death rates of its members, its growth is classified as -density dependent -density independent -exponential -Malthusian -logistic

-density dependent

during DNA replication, covalent bonds form between -deoxyribose of one nucleotide and phosphate group of the next nucleotide -the phosphate groups of adjacent nucleotides -the nitrogen bases of adjacent nucleotides -deoxyribose of one nucleotide and nitrogen base of the next nucleotide -phosphate group of one nucleotide and nitrogen base of the next nucleotide

-deoxyribose of one nucleotide and phosphate group of the next nucleotide when two DNA strands are separated (forming a replication fork) the enzyme DNA polymerase adds complementary nucleotides to each of the originals strands and covalent bonds form between deoxyribose sugar of one nucleotide and phosphate group of the next nucleotide on the growing strand.

if both phenotypic extremes are favored in a population, this is an example of -stabilizing selection -directional selection -sexual selection -diploidy -disruptive selection

-disruptive selection stabilizing selection is the opposite. the extreme phenotypes are elected against and the intermediate phenotypes are selected for. directional selection occurs when a single phenotype is favored in the population. sexual selection is the selection of mates in a population based on phenotypes they possess that are considered desirable. diploidy describes cells that contain 2N or a complete set of chromosomes.

fibroblasts, mast cells, and macrophage are found in which layer of the skin -dermis -hypodermis -epidermis -stratum corneum -stratum basale

-dermis the skin contains three basic layers: epidermis, dermis, and hypodermic. the epidermis is the outermost layer of the skin. it is divided into individual layers and the skin grows from the bottom layer of the epidermis and cell division takes place in the stratum basale. the stratum corneum is the layer of dead cells on the surfaces of the skin. the dermis is the layer of the skin below the epidermis that nourishes it with blood and nutrients. since the dermis contain the blood supply this is where the blood cells such as macrophage and mast cells are found. in addition, fibroblasts, which are fiber secreting cells, are locared in the dermis where they secrete proteins such as collagen that give the skin its strength. the dermis is the true skin. the hypodermis is a fatty layer of tissue below the dermis that connects the skin to the connective tissues below it.

the adjacent epithelial cells are held together by -liposomes -desmosomes -microsomes -glycoxysomes -none of the above

-desmosomes desmosomes are the connection between animal adjacent epithelial cells and smooth muscle cells. they provide a means of attachment and are a mechanism for distributing mechanical stress through tissues

which of the following food chains is NOT started with green plants? -grazing food chain -predator food chain -detritus food chain -parasitic food chain -none of the above

-detritus food chain in detritus food chain, there is no green plants because here producers are absent. here decomposers get their energy from dead organic substances. the energy then flows to the carnivores. other food chains are as follows: producers --> herbivores --> carnivores producers (green plants) --> herbivores (primary consumers) --> carnivores (secondary or tertiary consumers)

in a multicellular organism, different types of cells carry out different functions, this is possible because -each type of cell has its own genome -different genes are expressed by different cells -cells change their genome based on their environment -all of the above -none of the above

-different genes are expressed by different cells every cell in an organism's body contains the same genome meaning they contain the same genes.

the activation of the sympathetic division of the nervous system causes all of the following except: -dilation of the bronchi -dilation of the pupils -digestion -faster heart-rate -release of adrenaline

-digestion the sympathetic division of the nervous system prepares the body for the fight or flight response. in other words, it prepares the body to react to stress. during the stress response, the bronchi or airways to the lungs dilate, the pupils dilate, the hear beats faster, and adrenaline or epinephrine is released by the adrenal glands. digestion is a slow process, during which the body slows down its metabolism. this is not part of the stress response.

which of the following is NOT function of saliva? -digestion of carbohydrate -digestion of protein -digestion of fat -bacteriolytic action -lubrication

-digestion of protein there is no proteolytic enzyme in saliva and hence digestion of protein is not possible. saliva helps in early metabolism of carbohydrate due to alpha-amylase action. salivary glands also secrete salivary lipase to start gat digestion. lysozyme and thiocyanates, present in saliva, can destroy mouth bacteria. saliva helps in moistening and lubricating the mouth and food.

rough ER is found in the cells of - -digestive glands -testes -heart muscle -kidney -ovaries

-digestive glands rough ER is covered with ribosome and produces proteins and phospholipids. it is found in protein producing cells in digestive glands. in contrast, smooth ER does not have ribosomes and it produces lipid; hormones like estrogen and testosterone. it is found in the cells of testes and ovaries, and it is also found in the cells of heart muscles where it helps contraction of the heart by releasing calcium. smooth ER in kidney cells help to detoxify drugs and poisons.

in his 'natural selection theory', Darwin did not consider the role of -survival of the fittest -struggle for existence -discontinuous variation -parasites as natural enemies -none of the above

-discontinuous variation

which of the following is NOT correct about the circulation of O2 in the arteries -every hemoglobin molecule is attached to 4 O2 molecules in saturated condition -dissociation of oxyhemoglobin decreases with the increase of partial pressure of CO2 -as an inorganic solution, a deciliter of blood can dissolve 0.3 ml of O2 -100 ml of arterial blood contains 19 ml of O2 -none of the above

-dissociation of oxyhemoglobin decreases with the increase of partial pressure of CO2 with the increase of partial pressure of CO2, the oxygen-hemoglobin dissociation graph moves to the right. therefore, the dissociation of oxyhemoglobin increases.

eutrophication, the rapid increase in algae or plant growth in an aquatic system, results in the reduction of -dissolved nitrate -dissolved oxygen -mineral salts -parasitic protozoa -none of the above

-dissolved oxygen eutrophication is the rapid increase in algae or plant growth in an aquatic system occurs due to influx of a limiting nutrient that was in short supply previously. as a result dissolved oxygen is depleted.

facultative water reabsorption occurs in -ascending limb of Henle's loop -descending limb of Henle's loop -proximal convoluted tubule -distal convoluted tubule -none of the above

-distal convoluted tubule 20% of the filtered water by glomerulus is reabsorbed based on need in distal convoluted tubule and collecting duct. this is called facultative water reabsorption. it is regulated by the hormone ADH.

substances such as alcohol and coffee that increase the loss of water by the kidneys are called -antidiuretics -stimulants -inhibitors -diuretics -laxatives

-diuretics

of the following lists of taxonomic ranks, which is ordered from the least related to the most closely related -species, genus, order, class -species, genus, family, order -kingdom, domain, family, species -domain, phylum, class, family -phylum, order, class, genus

-domain, phylum, class, family domain kingdom phylum class order family genus species dope king phillip came over for good spaghetti

which of the following is not a difference between process of mitosis and meiosis in the cell? -the number of chromosomes is reduced by half -homologous crossover occurs -two divisions take place -the daughter cells are identical to the parents -during anaphase the sister chromatids do not separate

-during anaphase the sister chromatids do not separate

which of the following is NOT characteristic of fungi? -during mitosis, the nuclear membrane disappears very quickly -most fungi have a cell wall made of nitrogen contain polysaccharide -a typical reproductive unit is a spore by which both sexual and asexual reproduction is possible -fungi imperfecti reproduce only asexually -fungi are classified on the basis of their life cycle and the way spores are produced

-during mitosis, the nuclear membrane disappears very quickly fungi, which are eukaryotes, have all the cell organelles EXCEPT chloroplasts and their nucleus is small. during mitosis, the nuclear membrane does not disappear and all the events occur within the nuclear envelope.

in which of the following phyla is a water vascular system found? -ctenophora -poriphera -cnidaria -amnelida -echinodermata

-echinodermata water vascular system, consisting of a central ring, the hydrocoel, and radial ambulacra stretching along each limb, is the characteristic feature of echinodermata. it is a network of fluid-filled canals which mainly help in gas exchange and feeding. it also helps in locomotion.

tube foot is found in -echinoderms -porifera -arthropods -annelids -none of the above

-echinoderms echinoderms have many small movable extension of water-vascular system, called tube feet. tube feet aid in movement, feeding, respiration, and excretion.

the change in species in an ecosystem over time, as each new set of species modifies the environment in a predictable manner to allow the establishment of other species is referred to as -disturbance -invasion -competition -a trophic cascade -ecological succession

-ecological succession

the structural and functional unit of a biosphere is -ecosystem -ecotone -community -ecophene -none of the above

-ecosystem biosphere is composed of ecosystems which is formed by all living organisms, inorganic and organic substances and sunlight, all in a particular place. community consists of only the interacting organisms. ecotone is the transition area between two adjacent ecosystems ecophene is the variety of phenotypes form a single genotype that is observed in a population

which of the three primary germ cell layers in the embryo is responsible for forming the nervous system -endoderm -mesoderm -ectoderm -epiderm -none of the above

-ectoderm epiderm is not one of the three endoderm gives rise to the epithelial linings of the body. mesoderm gives rise to the connective tissues such as the muscles and bone. the nervous system and the skin are formed from the ectoderm

the embryo sac of a typical angiosperm consists of an -egg, synergids and antipodals -egg, synergids and secondary wall -egg, synergids, central cell and polar nuclei -egg, synergids, polar nuclei -egg, synergids, polar nuclei and antipodals

-egg, synergids, polar nuclei and antipodals a typical embryo sac consists of an egg apparatus at the micropylar end, three antipodal cells at the chalazal end and two polar nuclei in the center. the egg apparatus consists of an egg and two synergids. there is however a considerable variation in the number of synergids, polar nuclei and antipodals in different types of embryo sacs.

the emergence of the body plan of the developing organism is visible during the ________ period. -fetal -embryonic -cleavage -induction -none of the above

-embryonic at this point, the organism is less than 8 weeks old and the body plan and organs begin to appear. during the fetal period which lasts from 8 weeks until birth, the organs become larger and more complex. cleavage is the set of early divisions of the fertilized egg that increases the number of cells without increasing the size of the egg. induction is the beginning of any developmental process.

chemical reactions that have a positive delta G and require an input of energy from the surroundings to proceed are called ________ reactions -endergonic -exergonic -endothermic -exothermic -none of the above

-endergonic

EcoRI is commonly used to digest DNA sequences for analysis in restriction mapping. EcoRI cuts DNA molecule at palindromic sequences within the DNA strand. EcoRI is an example of a(n)_______ -exonuclease -endonuclease -ribonuclease -A & C -none of the above

-endonuclease endocucleases are enzymes that break bonds within a DNA strand exonucleases break the bonds at the end of the DNA strand ribonucleases digest or break down TNA molecules.

the translation of amino acids into proteins occurs on the surface of what organelle in the cell -nucleus -lysosome -endoplasmic reticulum -golgi apparatus -mitochondria

-endoplasmic reticulum

chloroplasts and mitochondria were once free-living organisms. today, they are organelles that produce energy in eukaryotic cells. the process by which chloroplasts and mitochondria became components of the eukaryotic cell is referred to as -endocytosis -biogenesis -phagocytosis -endosymbiosis -cohabitation

-endosymbiosis scientists believe that chloroplasts and mitochondria were prokaryotic organisms that were engulfed by eukaryotic cells. instead of being digested, the prokaryotic cells were maintained, became endosymbionts, and were passed on to future generations during cell division. this is endosymbiosis. biogenesis is the formation of a living system from another living system. phagocytosis is the process by which white blood cells engulf invaders in the body for destruction. cohabitation refers to the sharing of a habitat by two species. endocytosis is the process by which cells bring substances into the cell from the external environment.

an ecological pyramid of biomass is the representation of the ecosystem's -population in each food web -tissue organization at each trophic level -total number of organisms -energy flow through each trophic level -any of the following

-energy flow through each trophic level pyramid of biomass is the dry weight of tissues at different trophic levels. in grasslands and forest ecosystem, generally there is a gradual decrease in biomass of organism at successive levels from the producers to the top carnivores.

which of the following statements is NOT correct? -most genes require enhancers for their transcription -enhancers can activate only cis-linked promoters i.e., promoters on the same chromosome -enhancer is a DNA sequence -enhancer should be on the 5' side of the gene -none of the above

-enhancer should be on the 5' side of the gene an enhancer is a DNA sequence that can activate the utilization of a promoter, controlling the efficiency and rate of transcription from that particular promotor. enhancers do not need to be on the 5' side of the gene. they can also be at the 3' end, in the introns, or even on the complementary DNA strand.

in somatic cell nuclear transfer, the ______ ovum receives a ______ from the donor cell -fertilized; nucleus -enucleated; nucleus -differentiated egg; chromosome -enucleated; chromosome -none of the above

-enucleated; nucleus

pleura is a double membrane sac which -envelops the kidney -envelops the brain -envelops the lungs -envelops the heart -line the nasal passage

-envelops the lungs the lungs are surrounded by a thin transparent double layered membrane called the pleura.

the sequential development of an organism which involves the differentiation of cells into specialized structures is referred to as -epigenesis -preformation -imprinting -induction -allometric growth

-epigenesis

the connective tissue that covers the belly of a muscle is referred to as the -endomysium -epimysium -perimysium -fascicle -tendon

-epimysium the belly of the muscle is its thicker middle region within the belly bunches of muscle cells are gathered into fascicles. the fascicles are surrounded by perimysium. each cell of the fascicle is covered by a thin layer of connective tissue called endomysium. tendons are a type of connective tissue that connect muscles to bones.

in the Bombay phenotype, individuals inherit the A and B alleles of the ABO blood group system, but have type O blood. this phenotype occurs because these individuals are unable to produce the H antigen, which affects the production of the A and B antigens. this is an example of -codominance -pleiotropy -co-adaptation -epistasis -incomplete dominance

-epistasis epistasis is when the expression of one gene is modified by another gene. the H antigen is modifying the behavior of the AB genes.

a gene pair hides the effect of another gene. the phenomenon is called -mutation -segregation -epistasis -dominance -recessiveness

-epistasis in epistatic interaction, one gene masks the effect of other gene. in other words, id does not allow the other gene to express itself.

which of the following describes the main purpose of a "genetic map" -shows the stages during cell division -established various stages in gene evolution -establishes sites of the genes on a chromosome -shows the distribution of various species in a region -all of the above

-establishes sites of the genes on a chromosome the use of a genetic map is to clone genes. it establishes the site of genes on a chromosome and is also called positional cloning or map-based cloning.

the study of animal behavior is referred to as -ethology -entomology -etiology -zoology -ethnology

-ethology entomology - the study of insects zoology - the study of animals in general etiology - the study of causation ethnology - the branch of anthropology that investigates the origin and characteristics of the different ethnic groups in human societies.

the glands in the body that transport their chemical messengers to target tissues through ducts are classified as -endocrine glands -endoplasmic glands -pineal glands -exocrine glands -secretory glands

-exocrine glands chemical messengers are called hormones, and the glands that transport hormones through ducts are called exocrine glands. glands that release their hormones directly into the blood stream are called endocrine glands. there are no endoplasmic glands, rather there are intermembranes inside the cell that are called endoplasmic reticuli. the pineal gland is a specific gland that produces melatonin, which regulates the sleep cycle. all glands release substances and are therefore secretory.

the process in which vesicles fuse with the plasma membrane to release their contents to the extracellular environment is called -exocytosis -pinocytosis -facilitated diffusion -endocytosis -bulk transport

-exocytosis the movement of substances into the plasma membrane is called endocytosis. a type of endocytosis is pinocytosis which is the movement of fluids into the cell. facilitated diffusion substances move through channel proteins in the plasma membrane, from where they are in a higher concentration to areas of lower concentration. in bulk transport, large molecules such as amino acids are moved through the plasma membrane in either direction.

which of the following is NOT a form of homeostasis -in the winter, skin contracting to conserve the body heat -adjustment of the eye to dim light after coming from a bright light -heart beating faster during a run -urinary system regulating the content of water and other substances in the blood -eyes closing when something is going to fall in it

-eyes closing when something is going to fall in it

small charged ions such as Cl- and Ca2+ are transported through the plasma membrane by -osmosis -diffusion -transport proteins -bulk transport -facilitated diffusion

-facilitated diffusion small, non polar molecules such as oxygen can move through the membrane freely by diffusion. osmosis is the diffusion of water molecules across the plasma membrane. transport proteins, in the plasma membrane, carry larger molecules such as glucose and charged ions through the membrane. this is referred to as facilitated diffusion. Ca2+ and Cl- enter and leave the cell through these protein channels. bulk transport is the movement of large macromolecules across the plasma membrane.

which of the following does NOT occur during second trimester of a human baby? -heartbeat is heard -fat deposits under the skin -a layer of soft hair grows over the skin -skeleton begins to form -none of the above

-fat deposits under the skin during the last half of the third trimester, the fetus develops fat deposits under the skin. these fat deposits make the fetus look rounded and less wrinkled. during the second trimester, the fetus' heartbeat can be heard and its skeleton begins to form. also, a layer of soft hair called lanugo grows over its skin.

end product inhibition is called -substrate regulation -irreversible inhibition -reversible competitive inhibition -feedback regulation -non-competitive inhibition

-feedback regulation the rates of enzyme catalyzed reactions in biological systems are altered by activators and inhibitors. in the control of a metabolic pathway, when an enzyme, early in the pathway, is inhibited by an end product of the metabolic pathway in which it is involved, it is called feedback inhibition. it often takes place at the committed step in the pathway to prevent the build up of intermediates and the unnecessary use of metabolites and energy.

which of the following represents the proper sequence of gastrulation? -fertilization -> zygote -> cleavage -> blastula -> morula -> gastrula -fertilization -> morula -> zygote -> cleavage -> blastula -> gastrula -fertilization -> cleavage -> zygote -> morula -> blastula -> gastrula -fertilization -> zygote -> cleavage -> morula -> blastula -> gastrula -fertilization -> zygote - > cleavage -> blastula -> gastrula -> morula

-fertilization -> zygote -> cleavage -> morula -> blastula -> gastrula during fertilization, two haploid cells merge into a single diploid cell called a zygote. the zygote undergoes a series of rapid divisions called cleavage. during this time, a solid ball of 12-16 cells is formed, which is called morula. the morula becomes a hollow ball, defined as the blastula and a hole/pore is formed at one end. the blastula then changes to a more differentiated structure, called the gastrula.

the membrane that prevents the entrance of the second sperm into the ovum after the entrance of the first sperm is -zona pellucida -perivitelline membrane -hyaluronic acid -fertilization membrane -none of the above

-fertilization membrane as soon as the head of the sperm enters into the ooplasm, a fertilization membrane is formed on the inner surface of the vitelline membrane of the ovum. this membrane prevents the entry of other sperms into the ovum.

placenta is the region where the -fetus is provided with the mother's blood -festus is attached to the mother by the umbilical cord -fetus is covered by membranes -fetus receives nourishment from the mother's blood -none of the above

-fetus receives nourishment from the mother's blood the outer cell mass (trophoblast) of the morula is differentiated into two layers. the outer layer gives rise to placenta. the embryo is connected to the placenta by the umbilical cord which contains arteries and veins that carry blood between the embryo and the placenta. within the uterus, the embryo develops by collecting nutrition from the mother's blood through the placenta.

serum contains -one type of antibody -two types of antibody -three types of antibody -four types of antibody -five types of antibody

-five types of antibody there are five isotypes or classes of antibodies or immunoglobulins. all are present in the blood. these are: IgA IgM IgD IgG IgE

the group of epithelial cells that surround a developing egg inside the female reproductive system is called the -follicle -ovary -oviduct -fallopian tube -vas deferens

-follicle follicles are epithelial cells. the ovary is the structure that produces the eggs, and the follicles are inside the ovary. the oviduct or fallopian tube carries the eggs to the uterus after is has been fertilized. the vas deferens are the tubes in the male reproductive system, through which sperm cells travel when they are ready to leave the body.

which of the following describes the reducing phase in the Calvin cycle? -formation of 3PGA from RuBP -formation of DHAP from 3PGAID -formation of FMP from FBP -formation of 3PGAID from 3PGA -none of the above

-formation of 3PGAID from 3PGA PGA reacts with ATP to form BPGA which is reduced to PGAID by NADPH + H+.

which of the following is NOT a function of ER? -steroid metabolism -storage of calcium ion -formation of acrosome -synthesis of protein -none of the above

-formation of acrosome the main function of smooth ER is storage and sudden release of calcium ions. other functions are synthesis of lipids and steroids, metabolism of carbohydrates, drug detoxification, steroid metabolism and so forth. the main function of rough ER is protein synthesis. acrosome is derived from Golgi apparatus

which of the following is NOT the main function of lymph glands -forming RBC -forming WBC -forming antibodies -destroying bacteria -none of the above

-forming RBC lymph glands or lymph nodes are composed of lymphatic tissue and give rise to special white blood cells and antibodies. they act as a defense against the spread of infection by engulfing bacteria and other foreign materials from the lymph. they are not concerned with the formation of RBC.

which of the following sugars is absorbed from the small intestine by facilitated diffusion -glucose -sucrose -fructose -lactose -galactose

-fructose the duodenum and jejunum are main sites for carbohydrate absorption in humans. facilitated diffusion is the major transport route for fructose. a secondary sodium dependent active transporter absorbs glucose and galactose.

cell walls made of chitin are found in members of which of the following kingdoms: -prokaryotes -monera -animalia -plantae -fungi

-fungi

asexual reproduction through the process of budding is primarily carried out by organisms in which of the following kingdoms? -monera -protista -animalia -fungi -plantae

-fungi in Monera, bacterial cells divide by binary fission. Protists also divide by fission. Plantae and Animalia divide by mitosis. Fungi divide by budding.

the hormone that stimulates the stomach to secrete gastric juice is the -gastrin -renin -enterokinase -eneterogastrone -insulin

-gastrin gastrin is a polypeptide hormone secreted by the stomach which stimulates the secretion of gastric acid and pepsin in the stomach.

the process by which embryo forms ectoderm, endoderm, and mesoderm during development is -blastulation -gastrulation -segmentation -differentiation -none of the above

-gastrulation in the gastrula stage, the cell layers are differentiated into outer ectoderm, inner endoderm, and in the middle mesoderm. these are the three germinal layers which give rise to different tissues and organs in the body.

which of the following is NOT an example of fixed action pattern? -the male stickleback fish turns bright red color during the breeding season -infants grasp strongly with their hands as a response to tactic stimulus -brood parasitism -a mating dance -geese and chickens follow their mother nearly from the first day after they hatch

-geese and chickens follow their mother nearly from the first day after they hatch this is called imprinting. being able to discriminate the members of its own species is of fundamental importance for the reproductive success. such discrimination can be based on a number of factors in many species including birds, though this type of learning only takes place in a very limited period of time.

total genetic information in a population is termed as -allele frequency -phenotype frequency -gene pool -distribution of traits -none of the above

-gene pool allele frequency is determined by dividing the number of a certain allele by the total number of allele of all types in the population phenotype frequency is equal to the number of individuals with a particular phenotype divided by the total numbers of individuals in the population the term gene pool is used to describe the total genetic information in a population.

the insertion of normal genes into human cells to correct genetic disorders is referred to as -DNA fingerprinting -gene therapy -transgenics -replacement cloning -RNA silencing

-gene therapy DNA fingerprinting is the technique used to identify an individual based on his or her genomic patterns. in transgenics a new gene is incorporated into an organism's genome from another organism or species. replacement cloning involves the use of clones to replace cells in a population. RNA silencing is a technique in which the expression of genes in a cell is inhibited by inserting foreign RNA into the cells.

which of the following characteristics of the gene does not align with the principles of classical genetics? -genes are independent particles -genes can undergo mutations -genes are linear on the chromosome -genes are transcribed into RNA and then protein -genes undergo recombination as discrete units on the chromosome

-genes are transcribed into RNA and then protein genes were viewed as linear entities on a chromosome that behave independently, while being shuffled around to increase genetic diversity during recombination. genes were also believed to be susceptible to mutations. before molecular biology, many scientists believed that hereditary information was actually stored in protein molecules. with the advent of molecular biology, it was demonstrated that genes carry the hereditary information, and that genes are transcribed into RNA to form proteins.

the process by which the genetic composition of a population changes by chance is referred to as -gene flow -natural selection -non-random mating -mutation -genetic drift

-genetic drift

the technique in which DNA sequences are extracted from one organism, cut into small fragments, and inserted into a host organism of the same or a different species is referred to as -SCNT -transformation -restriction mapping -cloning -genetic engineering

-genetic engineering SCNT is somatic cell nuclear transfer or cloning. transformation is the natural process by which bacterial cells incorporate foreign DNA into their genomes from the environment. restriction mapping is a technique in which DNA molecules are digested with enzymes in order to find the sites in the DNA that can be cut with the specific enzymes.

the tendency of closely linked alleles on a chromosome to be inherited together is referred to as -translocation -linkage disequilibrium -genetic linkage -recombination -gene transfer

-genetic linkage

the process of glucose synthesis from a non-carbohydrate is called -glycogenesis -glycolysis -glycogenolysis -gluconeogenesis -none of the above

-gluconeogenesis by the metabolic process gluconeogenesis, glucose is formed from non-carbohydrate carbon substrates such as pyruvate, lactate, glycerol, and glycogenic amino acids. glycogenesis is the process of glycogen synthesis in glycolysis, glucose is converted to pyruvate glycogenolysis is the breaking of glycogen to produce glucose-1-phosphate

which of the following molecules contains the most chemical energy -AMP -ADP -ATP -glucose -adenine triphosphate

-glucose

which of the following molecules does not move freely across the plasma membrane by diffusion -water -CO2 -NH3 -O2 -glucose

-glucose glucose molecules are larger, and the cell has special receptors and channels on its surface that detect and regulate the intake and release of glucose molecules.

during photosynthesis -glucose is produced during a dark reaction and ATP during a light reaction -both glucose and ATP are produced during a light reaction -glucose is produced during a light reaction and ATP during a dark reaction -both glucose and ATP are produced during both a light and dark reaction -both glucose and ATP are reduced during both a light and dark reaction

-glucose is produced during a dark reaction and ATP during a light reaction photosynthesis consists of two steps - a dark and light reaction. in the dark reaction, reducing capacity of NADPH and energy of ATP are utilized in conversion of CO2 to glucose. in a light reaction solar energy is trapped by a chlorophyll and stored in the form of chemical energy of ATP and as a reducing power in NADPH.

fungi store their food as -starch -glucose -glycogen -paramylum starch -all of the above

-glycogen glycogen is present in fungi cells as a reserve food, in addition to glycogen, mannitol, oil-globules and protein grains.

which combination is NOT correctly matched? -glycolysis - grana -krebs cycle - mitochondria matrix -calvin cycle - stroma -EMP path - cytoplasm -all of the above

-glycolysis - grana glycolysis takes place in the cytoplasm of a cell. grana are stacked thylakoids in the matrix of a chloroplast and grana take part in light reaction of photosynthesis

carbohydrates are a prosthetic group in -glycoproteins -mucleoproteins -lypoproteins -chromo proteins -none of the above

-glycoproteins carbohydrates are a prosthetic group in glycoproteins. they contain carbohydrates which are bound to protein molecules. the carbohydrate usually is one of the mucopolysaccharides.

which of the following plasma membrane proteins is involved cell identification -enzymes -channels -glycoproteins -receptors -transporters

-glycoproteins there are multiple types of proteins that are embedded in the plasma membrane. enzymes in the membrane catalyze chemical reactions. channels or transporters carry molecules through the membrane. glycoproteins in the plasma membrane give each cell type its identity receptors recognize molecules called ligands, that approach the plasma membrane.

all of the following structures are present in or on bacterial cells except: -ribosomes -flagella -pili -glycosome -A & C

-glycosome bacterial cells belong to kingdom Monera and are simpler in structure than eukaryotic cells. all cells contain ribosomes. flagella are present on many species of bacteria, and allow them to swim about freely. pili are protrusions from the bacterial cell wall, that allow bacterial conjugation or mating. the glycosome is a structure found in protozoans, in which glycolysis occurs. this structure is absent in bacteria.

which of the following ecosystem has the highest gross primary productivity -coral reef -grass land -mangroves -rain forest -arctic

-grass land gross primary productivity is the rate at which organic molecules are formed among autotrophs of the ecosystem. the grassland ecosystem has the highest gross primary productivity.

which of the following process does not inhibit the cell cycle -telomeres -cell to cell contact -apoptosis -growth factors -tumor suppressor genes

-growth factors

the first plants that developed the ability to reproduce using naked seeds are categorized as -angiosperms -mosses -bryophytes -gymnosperms -ferns

-gymnosperms first plants were bryophytes that did not contain any vascular structures or true roots. bryophytes such as mosses reproduce using sperm cells that swim in water to reach the eggs. the next level of plants that evolved were the pterophytes. this includes ferns. ferns have a vascular system and true roots, they produce spores not seeds. after the ferns, gymnosperms or seed producing plants evolved. these are the large land plants that include the conifers that bear seeds as cones. the word gymnosperm translates to "naked seed". flowering plants evolved last and they are capable of internal fertilization due to the presence of the flower that contains male and female parts. these are called angiosperms

which of the following glands is NOT controlled by the pituitary gland? -adrenal -thyroid -pancreas -ovaries -none of the above

-pancreas pituitary secretes the hormones ACTH, TSH, FSH, & LH which controls the growth and secretion of adrenal gland, thyroid gland and ovaries respectively.

the process by which unvaccinated members of a community develop immunity towards a pathogen by interacting with vaccinated members of the community is called -passive immunity -herd immunity -adaptive immunity -innate immunity -artificial immunity

-herd immunity the unvaccinated individuals do not have immunity towards the pathogen. rather, they are being protected from infection because the vaccinated individuals prevent the pathogen from spreading. this is herd immunity. innate immunity is present at birth, and it responds the same way to all pathogens. adaptive immunity is the memory component of the immune system that remembers specific pathogens and responds to them on an individual basis. passive immunity is developed by the reception of antibodies from one person to another. artificial immunity is induced by vaccination.

the vertebrae in birds are mostly -amphicoelous -opisthocoelous -procoelous -heterocoelous -none of the above

-heterocoelous (saddle shaped) the centrum of the vertebrae has one dace convex and the other concave

meiosis is responsible for the formation of gametes. if the gametes are different sizes, this condition is referred to as -monogamy -homogamy -heterogamy -schizogamy -isogamy

-heterogamy if the gametes (egg and sperm) are the same size, the condition is referred to as isogamy. if the gametes are different sizes, the condition is termed heterogamy. genetic monogamy describes organisms who only have offspring with each other. homogamy is a biological term for inbreeding. schizogamy is not an actual biological term, but the roots of the term suggest that it would mean the splitting of gametes into two.

which protein helps in packaging of DNA in chromatin? -DNA ligase -DNA polymerase I -histone -RNA -topoisomerase I

-histone DNA binds to histone and nonhistone proteins to form chromatin. it is wrapped around histones to form a nucleosome structure. then it is folded many times and thus the DNA molecule is compressed to a size that fits into the cell.

the maintenance of internal favorable conditions by a self regulated mechanisms, in spite of the fact that there are changes in the environment, is known as -entropy -steady state -homeostasis -adaptation -apoptosis

-homeostasis homeostasis is the regulation of the chemical composition of blood, body fluids, and other aspects of internal environment by an organism so that physiological processes can proceed at optimum rates.

mitochondria are NOT found in which of the following -human red blood cell -human nerve cell -human liver cell -frog inner cell -human white blood cell

-human red blood cell mitochondria are the second largest structures in cells after the nucleus. they are the universal occurrence in plant and animal cells but they are absent in bacteria and RBC of multi-cellular animals.

which of the following has a convex survivorship curve? -insect -fish -birds -humans -none of the above

-humans survivorship curves show the probability that a member of a population will survive to a certain age. insects and fish have a concave and birds have a linear

which of the following is not a component of urine? -urea -water -creatinine -hyaluronic acid -sodium

-hyaluronic acid urine, waste product produced by the kidney when it filters blood, consists of 95 percent water, ions such as sodium, and nitrogen-containing compounds such as urea, uric acid, and creatinine. hyaluronic acid is not a component of urine, rather it is a long un-branched polysaccharide that is found throughout the body in connective, epithelial, and neural tissues.

test cross is a cross between -hybrid X dominant parent (Tt x TT) -hybrid X recessive parent (Tt x tt) -dominant X recessive (TT x tt) -hybrid X hybrid (Tt x Tt) -A & B

-hybrid X recessive parent (Tt x tt) the cross of one F1 hybrid with a homozygous recessive parent is called a test cross. it is used to test whether an individual is homozygous (pure) or heterozygous (hybrid)

plants that are created by crossing two purebred organisms with different alleles of a trait, to ensure that hey are genetically superior are referred to as -transgenics -hybrids -recombinants -mutants -clones

-hybrids

lysosomes are rich in -hormones -hydrolytic enzymes -nucleic acids -carbohydrates -fat molecules

-hydrolytic enzymes lysosome contains hydrolytic enzymes that can demolish the cell itself. duet to the process of destruction of a cell by its own lysosomal enzymes i.e. autolysis of a cell, this organelle is called a 'suicide bag'.

when red blood cells are placed into a _______ solution, the cells shrink because water moves out of the cell and into the solution -hypotonic -hypertonic -isotonic -isometric -none of the above

-hypertonic in a hypotonic environment there is more water outside than inside the red blood cells and water moves into the cell causing it to swell. when concentration of solutes inside the cell and outside the cell are equal. an isotonic solution exists. isometric is used to describe muscle movement in which the joint and muscle length do not change during contraction.

many fungi are filamentous, the main body of these fungi is made up of thin, thread-like filaments that are called -hyphae -mycelium -fruiting bodies -fronds -sporangia

-hyphae in filamentous fungi, the main body of the fungus is made up of thread-like filaments called hyphae. the branches of the hyphae form the mycelium which is the vegetative or asexually reproductive part of the fungus. sporangia are structures, usually attached to the hyphae that produce asexual spores. these spores are diploid and can form new fungi. fruiting bodies are spore producing structures that are found in some species of fungi. fronds are large, divided leaves found on ferns.

which part of the brain controls body temperature -medulla oblongata -cerbellum -thalamus -hypothalamus -none of the above

-hypothalamus one of the most important roles of the hypothalamus is controlling body temperature. the decrease in core body temperature leads to shivering. during shivering, the efferent motor nerves to the skeletal muscles are influenced by descending pathways under the primary control of the hypothalamus. when the body temperature is increased, the hypothalamus is increased, the hypothalamus takes the role to increase heat loss by regulating sweating by controlling the secretion of the hormone epinephrine.

the zero law of thermodynamics states that -energy can neither be created nor destroyed -if two system are in equilibrium with a third system, then they are also in equilibrium with each other -all systems have a tendency to move towards disorder -it is impossible to cool as system to absolute zero -none of the above

-if two system are in equilibrium with a third system, then they are also in equilibrium with each other

Peyer's Patches are usually found in the -duodenum -jejunum -ileum -stomach -bile duct

-ileum Peyer's Patches are secondary lymphoid tissue found in the lowest portion of the small intestine, the ileum.

during reproduction, the adherence of the embryo to the uterine wall is referred to as __________ -implantation -conception -fertilization -ovulation -tubal ligation

-implantation the attachment of the embryo to the walls of the uterus enables it to receive oxygen and nutrients from its mother.

the Radula found in apple snail helps -in chewing food -in respiration -in excretion -in circulation -none of the above

-in chewing food radula is a flexible tongue-like strip of tissue covered with tough abrasive teeth that point backward. apple snails use it to scrape up plants.

which of the following sites is where the adrenal gland can be found? -at both side of the trachea -above trachea -in retroperitoneum on the top of both kidneys -at the bottom of third ventricle of the brain -none of the above

-in retroperitoneum on the top of both kidneys both adrenal glands are located in retroperitoneum, the anatomical space behind (retro) the abdominal cavity, located on top of both kidneys.

independently, the genes for hair texture in humans produce straight and curly hair. wavy hair in humans in an example of: -incomplete dominance -codominance -allelic association -polygenic inheritance -pleiotropy

-incomplete dominance

some flowers are pink because they express a combined phenotype genetically these flowers are heterozygous for the allele that produces the pigments this is an example of -incomplete dominance -codominance -sex-linked inheritance -polygenic inheritance -penetrance

-incomplete dominance when multiple genes control a single trait is is polygenic inheritance. penetrance is the number of individuals in a population who are carrying a specific gene.

the crossing of red snapdragon (WW) with white snapdragon (ww) produces pink (Ww) flower. this is an example of -hybridization -incomplete dominance / co-dominance -mutation -recessivness -none of the above

-incomplete dominance / co-dominance

P53 is a tumor suppression protein in cells. mutations in the TP53 gene that produces P53 can lead to all of the following except -mutations in cellular DNA -increased apoptosis in the cell -oncogenesis -metastasis -increased angiogenesis in the cell

-increased apoptosis in the cell

which of the following factors does not increase the speed of an enzyme -increased substrate concentration -increased temp -increased pH -activators -none of the above

-increased pH as proteins, enzymes have a specific pH at which they work optimally. lowering or increasing the pH can disrupt the configurations of the molecules in the enzyme.

genetic recombination -occurs during mitosis -increases genetic variation -occurs in somatic cells -A & B -A & C

-increases genetic variation genetic recombination is the process in which chromosomes are rearranged to increase the genetic variation in the parents' offspring. this occurs during meiosis in the gametes. meiosis does not occur in body or somatic cells, rather it is present only in the gametes, which include the egg and sperm.

the model of enzyme activation in which the enzyme modifies its conformation to fit the substrate is referred to as the _____ model -lock and key -induced fit -strain distortion -michaelis-menten -B & D

-induced fit there are two currently accepted models of enzyme substrate binding. the first model that was proposed was the lock and key fit. in this model the enzyme doesn't change its shape or conformation. the second model is the induced fit hypothesis. this states that the enzyme changes its conformation to accommodate the substrate. strain distortion is the idea that an enzyme modifies the substrate and forces it to fit the active site. in enzymology, Michaelis-Menten usually refers to a model for describing enzyme kinetics and not structural or physical changes.

the complement cascade is a component of -innate immunity -adaptive immunity -acquired immunity -acquired passive immunity -specific immunity

-innate immunity the complement cascade is a system of small proteins that help the immune system to clear pathogens from the body. the proteins assemble to form a membrane attack complex, which forms pores in bacterial cells that lead to their rupture. this cascade is a non-specific response and responds the same to all pathogens. therefore, the complement cascade is a component of the innate immune system or innate immunity. adaptive immunity or specific immunity is the memory component of the immune system that remembers specific pathogens, and responds to them on an individual basis. acquired immunity is achieved from prior exposure to a pathogen, and is a component of the adaptive immune response. immunity can be acquired and passive. acquired passive immunity is achieved by the reception of antibodies from one person to another.

which portion of cell membrane actively transports molecules into the cell? -phospholipid bilayer -cholesterol -carbohydrate -integral proteins -peripheral proteins

-integral proteins plasma membrane consists of phospholipid bilayer where integral proteins are embedded within the lipid layer. this binds to a molecule outside the cell and transports it through the membrane. carbohydrates thus act as labels on cell surfaces. cholesterol molecules are embedded in the bilayer and help in forming vesicles from plasma membrane, while peripheral proteins help to maintain cell shape and motility.

random genetic drift in a population results from -interbreeding within a population -constant low mutation rate -large population size -highly genetically variable individuals -none of the above

-interbreeding within a population random changes in gene frequency occurring by chance and not under the control of natural selection is called genetic drift. the random genetic drift in a population probably results from interbreeding within a small population.

the protein that cell produce when they are infected by viruses to prevent the spread of the virus to other tissues is called -interferon -histamine -IgA -interleukin 1 -interleukin 2

-interferon interferon is produced by infected cells to interfere with or prevent the spread of the virus in the tissues. histamine is not a protein; rather it is a chemical produced by the mast cells of the immune system that causes vasodilation. vasodilation increases the flow of blood to the infected or injured tissues. immunoglobulin A is an antibody found in the fluids of the body that binds to bacteria and other foreign invaders of the body. interleukins are the chemical messengers of the immune system. interleukin 1 creates the fever response and interleukin 2 stimulates the production of T cells.

the sum of the total amount of energy in a thermodynamic system is referred to as -entropy -enthalpy -potential energy -internal energy -calories

-internal energy

in a chromosomal __________, a segment of the chromosome is broken off and reattached in the opposite direction -duplication -deletion -translocation -insertion -inversion

-inversion

bile is -produced by the gallbladder -stored in the liver -involved in fat digestion -A & C -all of the above

-involved in fat digestion it's stored and released from the gallbladder, but produced by the liver

new species, also referred to as speciation, develops due to -isolation and mutation -isolation and variation -competition and variation -competition and mutation -none of the above

-isolation and mutation speciation is influenced by mutation, hybridization, recombination, polyploidy, natural selection, genetic drift and isolation

which of the following is correct about periosteum? -it produces RBC -it produces WBC -it contains bone marrow -responsible for the increase in the length of the bone -it contains blood vessels that supply nutrients

-it contains blood vessels that supply nutrients the surface of long bones is covered by a tough membrane, called periosteum, which contains a network of blood vessels that supply nutrients.

which of the following is NOT correct regarding respiration? -it occurs only in living cells -it may or may not require free oxygen but always produces pyruvic acid -it produces heat energy -it does not require emzymes -it occurs in cytoplasm and mitochondria

-it does not require emzymes respiration occurs in all living cells. in aerobic respiration free oxygen is needed but anaerobic respiration occurs in the absence of free oxygen. however, in both cases oxygen oxidizes pyretic acid which is produced from glucose. glycolysis occurs in cytoplasm and Krebs cycle occurs in mitochondria. aerobic respiration produces 673Kcal of heat and anaerobic respiration produces 25-28 Kcal of heat from one glucose molecule. respiration requires many enzymes and co-enzymes, for example acetyl CoA, citrate synthetase, isocitric dehydrogenase, fumerase etc. therefore it DOES require many enzymes.

which of the following is a characteristic of bile juice? -it has enterogastrone -it has no enzyme -it has trypsin -it has tripophnomide -none of the above

-it has no enzyme bile is the secretion of the liver and it is stored in the gall bladder and is poured into the duodenum via the bile duct. it has a bile pigment and carries no enzymes. bile helps in the digestion of fat.

which of the following about the plasma membrane is NOT true? -it contains phospholipids -it contains protein -it is permeable to charged ions due to simple diffusion -it is permeable to small uncharged solute molecules -it helps in phagocytosis and pinocytosis

-it is permeable to charged ions due to simple diffusion plasma membrane is made of a phospholipid bilayer which has a polar, hydrophilic phosphate head portion and non-polar hydrophobic fatty acid tail portion. charged ions cannot dissolve into hydrophobic regions and so they cannot cross the plasma membrane by simple diffusion. on the other hand, uncharged molecules can pass through it. plasma membrane contains protein embedded within it. phagocytosis and pinocytosis are cellular processes of engulfing solid or liquid particles by plasma membrane.

which of the following is true about cleavage in the fertilized egg among humans? -it starts while the egg is in fallopian tube -it starts when the egg reaches uterus -it is identical to the normal mitosis -it is meroblastic -none of the above

-it starts while the egg is in fallopian tube in human females, the fertilization takes place in fallopian tube, where the cleavage starts and fertilized ova travels down and gets attached to the uterus at blastocyst stage.

the outermost later of the skin is composed of cells referred to as -tactile corpuscles -keratinocytes -melanocytes -Merkel cells -Langerhan cells

-keratinocytes tactile corpuscles and Merkel cells are touch receptors in the skin. melanocytes are the melanin producing cells in the skin. Langerhan cells are immune cells that are spread throughout the lower layers of the skin.

spindle chromosomes have -centrioles -chromocenters -chromomeres -kinetochores -none of the above

-kinetochores microtubule spindle fibers can be seen attached to the kinetochore. the kinetochore of premetaphase and telophase appears to be composed of three zones.

although animal cells are generally spherical, they can adopt a variety of different shapes because they -are very small -lack a cell wall -have flexible cytoskeletons -have a dynamic phospholipid bilayer -contain mostly water

-lack a cell wall

all of the following describe members of kingdom Monera except: -unicellular -prokaryotic -lack of organelles -circular chromosome -reproduce by binary fission

-lack of organelles

lentic ecosystems are the ecosystems of -estuaries -bays and lagoons -wet lands -lakes and ponds -rivers and streams

-lakes and ponds it's kind of a fresh water ecosystem. Lentic refers to standing or still water. it includes the ecosystems of still or slow-moving water bodies like pools, ponds, lakes

the region of the pharynx that serves as a passageway for air and food is the -nasopharynx -trachea -laryngopharynx -bronchi -epiglottis

-laryngopharynx the pharynx or throat is divided into three parts. the nasopharynx is the uppermost region and it connects the throat to the nasal cavity. it remains open at all times to allow the passage of air. the oropharynx is located behind the mouth and the laryngopharynx is below it directly above the esophagus. both the oropharynx and laryngopharynx function as passageways for air and food. the trachea or windpipe is below the pharynx and branches into brooch that deliver air to the lungs. the epiglottis is a small flap of cartilage that covers the entrance of the larynx. it closes to prevent food from entering the trachea and going to the lungs.

Foramen of Monro in the human skull is an aperture found between the -third and fourth ventricle -lateral and third ventricle -rhinocoel and diacoel -diacoel and metacoel -none of the above

-lateral and third ventricle there are two lateral ventricles, one in each cerebral hemisphere. these two lateral ventricles are connected by two narrow nerve channels called Foramen of Monroe with the third ventricle above the midbrain.

which of the following sense organs are unique to fish -olfactory organ -optic organ -lateral line sense organs -muscle system -none of the above

-lateral line sense organs lateral line sense organs are unique in fishes. this system can perceive vibration of very low frequency and detect disturbances in water which is caused by the movement of the fish and predators.

according to the __________, the alleles for different traits separate randomly of each other during meiosis -law of segregation -hardy-weinberg principle -law of probability -law of independent assortment -law of linkage equilibrium

-law of independent assortment

the mitral valve allows blood to flow from -superior vena cava to right atrium -right atrium to right ventricle -left atrium to pulmonary artery -left atrium to left ventricle -left ventricle to aorta

-left atrium to left ventricle the mitral valve, or bicuspid valve, comprising of two membranous flaps, lies between the left atrium and left ventricle. during left ventricular diastole, the mitral valve opens and blood flows from the left atrium to the left ventricle.

the thickest musculature of the heart is found in -left auricle -right auricle -left ventricle -right ventricle -A & C

-left ventricle the muscle walls of the two ventricles are thicker and stronger than that of the auricles. also, the muscle wall surrounding the left ventricle is thicker than the wall surrounding the right ventricle due to the higher force needed to pump the blood through the systemic circulation.

which of the following enzymes is used to join pieces of DNA? -ligase -endonuclease -primase -DNA polymerase -none of the above

-ligase ligase is an enzyme that catalyses the bond formation between two substrates. this catalytic reaction will only take place at the expense of the breakdown of ATP or other nucleotide (DNA) triphosphate.

which of the following structures has a single membrane? -lysosomes -chloroplast -cell wall -mitochondria -two of the above

-lysosomes the lysosomes are bound by a single unit membrane of 70 A thickness. these are small bag-like structures filled with different digestive enzymes.

trypsinogen is secreted from which of the following parts of the gastrointestinal tract? -duodenum -liver -pancreas -stomach -none of the above

-pancreas trypsinogen is the precursor form of the pancreatic enzyme trypsin. it secretes from the pancreatic acini, a small berrylike clusters of dark staining cells.

which of the following is NOT true of Mendel's Law? -law of segregation -law of independent assortment -dominance -linkage -all of the above

-linkage Mendel's Laws include the Law of Dominance, the Law of Segregation, and the Law of Independent Assortment. the Law of Dominance reflects the relationship between different allelic forms, in which the phenotype of a genotypic heterozygous individual is expressed as one form. the Law of Segregation describes the separation of the two alleles for a given gene during meiosis into separate gametes (i.e. Parent cell: Aa -> Gametes: A and a) the Law of Independent Assortment describes the relationship of two different genes on separate chromosomes, and their tendency to assort independently into a gamete. genetic linkage occurs when particular genetic loci or alleles for genes are jointly inherited. genetic loci on the same chromosome are physically connected and tend to segregate together during meiosis, and are thus genetically linked.

G0 phase is found in the cell of -muscle -liver -zygote -mature RBC of mammals -none of the above

-liver in some animal cells G0 phase is found. during this phase cells do NOT divide. cells either enter the S phase from a cell cycle checkpoint in the G1 phase e.g. restriction point in animal cells, or enter G0 phase if the external conditions are not suitable. parenchymal cells of the liver and kidney generally enter into G0 phase.

Glisson's capsules are present in the -lung -kidney -stomach -liver -large intestine

-liver liver is characterized by the presence of Glisson's capsule. they are made up of connective tissues and they separate each lobule of the liver. Glisson's capsule of the liver is formed by the lier cells which are hexagonal.

the liquid part of the blood is called plasma, where in the body is plasma produced? -inside the red blood cell -urinary system -kidney -bone marrow -liver

-liver plasm is the liquid portion of the blood, and it makes up approximately 50 percent of the blood's volume. plasma carries the blood cells through the body, and it is produced by the liver. the bone marrow produces all of the cells that are found in the blood. the kidney releases a hormone that triggers the production of red blood cells. the primary function of the red blood cell is to transport oxygen through the body. the urinary system filters the blood to produce urine.

in renal physiology, the structure of the kidney in which urine is concentrated is called the -loop of Henle -proximal convoluted tubule -distal convoluted tubule -collecting duct -Bowman's capsule

-loop of Henle at the head of the nephron, or functional unit of the kidney, is a cup-shaped structure called the Bowman's capsule, where the filtration of blood to form urine begins. next, the filtrate enters the proximal convoluted tubule, which adds hydrogen ions to the filtrate before passing it to the loop of Henle. in the loop of Henle, the urine is concentrated and then passed to the distal convoluted tubules which regulates salt uptake and release. the filtrate then enters the collecting ducts where final reabsorption of solutes take places before the urine is passed to the ureter.

in chemical reactions inside the cell, oxidation refers to the -loss of electrons -gain of electrons -gain of hydrogen atoms -A & C -B & C

-loss of electrons oxidation is loss of electrons and it is usually accompanied by the loss of hydrogen atoms. reduction is the gain of electrons and it is usually accompanied by a gain of hydrogen atoms.

calcitonin is a thyroid hormone which -lowers calcium levels in the blood -has no effect on calcium -elevates calcium levels in the blood -regulates water balance in the blood -lowers sugar levels in the blood

-lowers calcium levels in the blood para cells of stroma in the thyroid gland secret calcitonin or thyrocalcitonin. it lowers calcium levels in the blood and inhibits the release of calcium.

secretion of progesterone by the corpus luteum is initiated by -luteinizing hormone (LH) -MSH -testosterone -thyroxine -fertilizin

-luteinizing hormone (LH) progesterone is secreted by the corpus luteum. the secretion of progesterone is stimulated by the Luteinizing Hormone (LH) from the anterior pituitary.

which of the following organelles is associated with phagocytosis and pinocytosis? -ribosome -lysosome -Golgi apparatus -mitochondria -ER

-lysosome both phagocytosis and pinocytosis are endocytosis of solid and liquid particles respectively. when some solid or liquid particle enters into the cell primary lysosomes fuse with the vacuoles covering the particles, thus forming secondary lysosomes. then it digests the particles by secreting different enzymes.

which organelle contains enzymes responsible for the breakdown of macromolecules? -mitochondria -ER -lysosome -Golgi apparatus -ribosome

-lysosome lysosomes are single membrane bound, vesicular cell organelles containing 40 types of hydrolytic enzymes like proteases, nucleases, ligases, glycosidases, etc. these enzymes can break down large molecules, such as proteins, nucleic acids, carbohydrates, and phospholipids.

which of the following cell organelles show polymorphism -Golgi apparatus -mesosome -ER -mitochondria -lysosome

-lysosome lysosomes show polymorphism depending on their size, shape, and internal structure. they are of four types: primary lysosomes, secondary lysosomes or phagosome, residual or post lysosomes, autophagosomes or autophagic vacuoles. also, different forms of lysosome are engaged in different functions.

which of the following organelles is covered with a single membrane? -mitochondria -lysosome -ribosome -chloroplast -none of the above

-lysosome lysosomes have a single membrane made with lipoprotein mitochondria have an inner and outer membrane ribosomes do not have a membrane chloroplasts are surrounded by a double membrane

the feature that cannot distinguish mammalian cleavage from other cleavage types is -cleavages in mammalian eggs are a very slow process -mammalian embryo increases exponentially -the cleavage is rotational holoblastic -mammalian genome is activated during early cleavage -none of the above

-mammalian embryo increases exponentially mammals have a slow rate of division that is between 12 and 24 hours. mammalian blastomeres do not all divide at the same time and so embryos do not increase exponentially from 2- to 4- to 8-cell stages, but frequently contain odd numbers of cells. rotational holoblastic cleavage occurs only in mammals. unlike almost all other animal genomes, the mammalian genome is activated during early cleavage and produces the proteins necessary for cleavage to occur.

which part of the mitochondria produces the enzymes for Krebs cycle -inner membrane -outer membrane -matrix -perimitochondrial space -none of the above

-matrix inner membrane, matrix and perimitochondrial space of mitochondria produce enzymes. only the matrix produces enzymes for Kreb's cycle like pyruvate dehydrogenase complex, citrate synthase, isocitrate dehydrogenase, and alpha-ketogluterate dehydrogenase complex.

in human fertilization, fertilizin is a chemical substance produced by -polar bodies -mature eggs -acrosome -middle piece of sperm -C & D

-mature eggs fertilizin is a gelatinous glycoprotein secreted by the matured egg (ovum) which plays an important role in agglutination of sperms on the egg surface.

the nerve center that controls saliva secretion is the -cerebellum -spinal chord -thalamus -hypothalamus -medulla

-medulla the medulla is the lower part of the hind brain. it contains an inspiratory and expiratory center, vasomotor center, vomitting center, inferior salivary nucleus, and dorsal nucleus for different functions. it helps in the secretion of saliva and other digestive juices.

the respiratory centers which control inspiration and expiration, are located in the -medulla oblongata -cerebellum -spinal cord -diencephalon -none of the above

-medulla oblongata medulla oblongata is the region of hind brain, which is concerned with the functioning of visceral organs like stomach, heart and lungs. it is the grey matter of medulla oblongata which is the center of controlling respiratory rhythm.

which of the following is not a type of white blood cell that is found in the immune system -natural killer cells -eosinophils -macrophage -megakaryocytes -lymphocytes

-megakaryocytes the immune system is composed of a fleet of white blood cells that carry out many of its protective functions. natural killer cells - a type of white blood cell that kill infected and tumor forming cells in the body. they are grouped with lymphocytes. lymphocytes - are white blood cells that are found throughout the lymphatic system. they include B-cells that produce antibodies, and T-cells that regulate the immune response. eosinophils - a type of white blood cell that is increased in number during allergic and parasitic infections. macrophage - are large white blood cells that digest bacteria and cellular debris though a process called phagocytosis. megakaryocytes - a type of bone marrow cell that produces thrombocytes or platelets

during fertilization, the chromosomes from the male and female gametes combine in the same nuclear membrane. what cellular process is responsible for maintaining the organism's chromosome number during this process? -cleavage -semi-conservative DNA replication -meiosis -cytokinesis -recombination

-meiosis

the kidneys, gonads, heart, and blood vessels are derived from which of the following primary, embryonic germ layers? -mesoderm -endoderm -epiderm -ectoderm -hypoderm

-mesoderm endoderm gives rise to the epithelial linings of the cavities and glands of the body. mesoderm gives rise to the organs and connective tissues of the body. ectoderm gives rise to the skin and nervous system. epiderm and hypoderm are not embryonic germ layers. epiderm is outermost layer of the skin and hypoderm is the innermost layer of the skin.

in C4 plants, carbon dioxide fixation is done by -mesophyll cells -guard cells -sclerenchyma -chlorenchyma and hypodermis -none of the above

-mesophyll cells mesophyll cells make up superficial layer of the leaf in C4 plants, where carbon fixation occurs via PEP carboxylase storing the CO2 carbons as oxaloacetate to prevent photorespiration. guard cells form the stomata of plants, and will swell or relax to close or open a given stoma for gas exchange sclerenchyma refers to "hard" supporting tissue of plants and is made of dead cells with thick cell walls chlorenchyma is a type of parenchyma with chloroplasts occurring in the mesophyll layer

the sum of the energetic reactions that take place in the cell are referred to as -anabolism -catabolism -metabolism -mechanical energy -Gibbs free energy

-metabolism anabolism is the sum of reactions in the cell that use energy to create larger molecules. catabolism is the sum of reactions in the cell that release energy when larger molecules are broken down. the sum of these two is metabolism. mechanical energy is not the sum of reaction; rather it is the sum of the potential and kinetic energies in a system. the Gibbs free energy is the amount of energy that is available at the end of a reaction.

during development, the process by which the spinal cord and brain form is referred to as -neurulation -parthenogenesis -neurogenesis -gastrulation -differentiation

-neurulation neurulation is the process in development that forms the central nervous system (brain and spinal cord). the formation of the peripheral nervous occurs though a different mechanism and involves the neural crest. neurogenesis is the formation of nerve cells or neurons from stem cells. gastrulation is the formation of the three germ layers in the embryo. differentiation is the process by which cells are assigned a specific fate during development. parthenogenesis, or virgin birth, is the process by which female eggs auto-fertilize without sperm and form new offspring.

the primary succession refers to the development of communities on a -freshly cleared crop field -pond, freshly filled with water after a dry phase -forest cleared after devastating fire -newly-exposed habitat with no record of earlier vegetation -all of the above

-newly-exposed habitat with no record of earlier vegetation primary succession means the development of communities only on a newly exposed habitat with no record of earlier vegetation. it occurs in barren uninhabited region which has no plant life and lacks soil and organic matter

co-enzyme NAD+ is produced from -vitamin B1 -vitamin B2 -folic acid -niacin -none of the above

-niacin niacin or vitamin B3 is converted to nicotinamide and then to the co-enzyme NAD+ and NADP+

a way of life that is specific to a particular animal that includes its interaction with the ecosystem is referred to as its -habitat -patterns -niche -biome -territory

-niche -habitat is the physical location where an animal dwells in its ecosystem -biome is another term for ecosystem -territory is its marked habitat

the abundance of a species within its habitat is called -niche density -relative density -absolute density -regional density -none of the above

-niche density niche is a functional role of organisms in an ecosystem which may be filled with different species in different geographical areas. thus, the abundance of a species in a population, within its habitat, is called niche density.

turgor pressure becomes equal to the wall pressure when -water enters the cell -water leaves the cell -solute goes from the cell into the water -no exchange of water takes place -none of the above

-no exchange of water takes place the pressure that develops in a cell from time to time, due to osmotic diffusion of water molecules into a cell that causes stretching of the cell wall, is called turgor pressure. due to turgor pressure, rigid cell wall offers resistance. this resistance works in an opposite direction to turgor pressure but it is equal to turgor pressure and is called wall pressure. in this condition, no exchange of water takes place.

in which phase of photosynthesis is O2 released? -cyclic photophosphorylation -non-cyclic photophosphorylation -dark phase -kelvin cycle -A & B

-non-cyclic photophosphorylation in non-cyclic photophosphorylation, O2 is produced (Hill reaction). H20 --> H+ + OH- 4OH- --> 4OH 4OH --> 2H2O + O2

in a chromosomal translocation, the segments of __________ chromosomes are rearranged -homologous -identical -non-homologous -sister chromosomes -sister chromatids

-non-homologous

cells can move using all of the following structures except -cilia -flagella -pseudopodia -microfilaments -none of the above

-none of the above

in biological processes, enzymes -increase the free energy of chemical reactions -decrease the free energy of chemical reactions -increase the activation energy of a reaction -A & C -none of the above

-none of the above

pedigrees are used to map ancestral traits among family members. which of the following cannot be observed using pedigree chart analysis -x-linked traits -y-linked traits -autosomal dominant traits -autosomal recessive traits -none of the above

-none of the above

ribosomes are found in all of the following structures except: -bacteria -mitochondria -chloroplasts -protists -none of the above

-none of the above

which of the following is not a scientifically accepted law of thermodynamics? -energy can neither be created nor destroyed -if two systems are in equilibrium with a third system, then they are also in equilibrium with each other -all systems have a tendency to move towards disorder -it is impossible to cool a system to absolute zero -none of the above

-none of the above

which of the following is not structural feature of a neuron -soma -axon -dendrites -axon hillock -none of the above

-none of the above

which of the following statements is NOT correct? -genetic factors on the Y chromosome determine maleness -females have two morphologically identical X chromosomes -all chromosomes exclusive of the sex chromosomes are called autosomes -sex chromosomes in the males are heteromorphic -none of the above

-none of the above

the region inside of a prokaryotic cell where DNA is heavily concentrated is referred to as the -nucleus -plasmid -nucleoid -nucleosome -nucleolus

-nucleoid prokaryotic cells do not have nuclei. the nucleosome is a eukaryotic structure found in the nucleus and contains DNA tightly bound to histone proteins. the nucleolus is a section of the nucleus in eukaryotic cells, in which ribosomes are produced. plasmids are small, circular fragments of bacterial DNA that are found throughout the bacterial cell. the nucleoid is the region of the bacterial cell in which the bacterial chromosome is found

which of the following statements is/are incorrect? (i) skin color is controlled by three to six genes (ii) a carrier has one copy of a recessive allele (iii) ABO blood groups are controlled by three alleles (iv) a pedigree can be used to trace genetic diseases -(ii) -(iii) -(i) and (iv) -(ii) and (iv) -none of the above

-none of the above all the statements are correct skin color is a polygenic character. it results from the additive effect of three to six genes. these genes control the amount of brownish-black pigment melanin in the skin. individuals in the pedigree are carriers. they have one copy of the recessive allele but they do not have the disease. they do not express the recessive allele but they can pass it to their offspring. ABO blood group is a multiple-allele character. it is controlled by three alleles - iA, iB, and i. a pedigree is a diagram which shows the inheritance of a trait over several generations. so it can be used to trace genetic diseases.

which of the following is NOT a function of bile -lipid emulsification -absorption of vitamin A, D, E, K -lipase activation -enterohepatic transport of drugs -none of the above

-none of the above bile salts emulsify dietary lipids. the negatively charged bile salts surround the lipids, creating small lipid droplets in the intestinal lumen. the negative charges on the bile salts repel each other, so that the droplets disperse, rather than coalesce, thereby increasing the surface area for digestive enzymes. bile increases the absorption of fats and thus helps in absorption of fat soluble vitamins A, D, E, and K. in the duodenum, pancreatic lipase develops its activity on triglycerides by binding to the bile-emulsified oil droplets in the presence of its protein cofactor pancreatic coliapse. the liver filters toxins (bacteria, viruses, drugs) and removes them via the bile.

which of the following movements is NOT involved in gastrulation -invagination -involution -delamination -epiboly -none of the above

-none of the above gastrulation involves all of the above movements. invagination means infolding of cell sheet into embryo. involution is the inturning of cell sheet over the basal surface of an outer layer. delamination is the splitting of one sheet into two sheets. epiboly is the expansion of one cell sheet over other cells.

pyruvate is formed at the end of glycolysis. cells can convert pyruvate into all of the following except -glucose -acetyl Co-A -ethanol -lactic acid -none of the above

-none of the above if the cell is aerobic and enters the citric acid cycle then pyruvate is converted to acetyl Co-A. some bacteria are anaerobic and instead convert pyruvate into lactic acid or ethanol. in some cases the cell can convert pyruvate back to glucose, if it doesn't have any current energy needs.

semen is a mixture of fluids produced by the following structures except the: -prostate gland -seminal vesicles -bulbourethral gland -testes -none of the above

-none of the above sperm cells (spermatozoa) is produced in the testes then stored in nearby in the epididymis until time for release from the body. the sperm is transported by the vas deferens to the urethra for ejaculation. at this time the sperm is combined with secretions from the seminal vesicles, prostate gland, and the bulbourethral glands. all shown are involved in semen production.

which of the following statements is NOT true? -rice, wheat, bamboo, coconut are monocot plants -monocots have parallel venation -monocots have scattered vascular bundles -flower parts of monocots usually occur in multiples of three -none of the above

-none of the above the flowering plants are divided into monocots and dicots. monocots have one cotyledon in the embryo. examples of monocots are rice, wheat, bamboo, coconut, lilies, palms, grasses, bananas, etc. mature monocot leaves have several main veins running parallel to each other. in monocots, vascular bundles appear scattered through the stem, with more of the bundles located toward the stem periphery than in the center. floral parts, like petals, stamens etc., in monocots, generally have a number of parts that is divisible by three, usually three or six.

the classical cell-shapes of members of kingdom Monera include all of the following except: -coccus -bacillus -spirillum -vibrio -none of the above

-none of the above these are all included

in a ______ mutation, an mRNA codon is changed into a stop codon. -point -silent -frameshift -nonsense -missense

-nonsense there are multiple types of mutations that occur in the DNA sequences inside an organism's genome. in a point mutation, a change occurs in a single nucleotide in the DNA sequence. this can be silent, if the change results in the production of the same amino acid. if the change results in the conversion of an amino acid codon to a stop codon, a nonsense mutation occurs. if the change results in the production of another amino acid, a missense mutation has occurred. in a frameshift mutation, nucleotides arcaded to or removed from the DNA sequence, and this disrupts the reading frame. the reading frame is the order in which the letter in the DNA sequence are transcribed. in a frameshift mutation, the message is changed because the starting point is shifted either to the left or the right.

during early embryonic development, the structure that gives rise to the spinal column in vertebrates is the -neural tube -neural crest -notochord -neural groove -coelem

-notochord

the extracellular matrix (ECM) of the cell is made up of all of the following except -collagen -proteins -polysaccharides -nucleic acids -none of the above

-nucleic acids

in the final step of glycolysis, the pyruvate is formed from which of the following molecules -pyruvate phosphate -phosphoenolpyruvate -pyruvate kinase -glucose -glycogen

-phosphoenolpyruvate

inside the nucleus, DNA is found tightly bound to proteins in a structure called the -nucleolus -nucleosome -nucleoid -microchromosome -autosome

-nucleosome inside the cell, DNA is tightly bound to proteins called histones in structures called nucleosomes. structurally the appearance of the nucleosome is similar to a spool thread, because DNA is wound around these proteins. the nucleolus is the region of the nucleus that produces ribosomes. the nucleoid is the structure in prokaryotic cells where the DNA is found. micro chromosomes are extremely small chromosomes and they are absent in mammals. an autosome is a chromosome that doesn't carry genes that determine an individual's sex.

the transcription of DNA into mRNA takes places in which organelle of the cell -ER -cytoplasm -ribosome -nucleus -none of the above

-nucleus

antigens are present -inside the nucleus -inside the nuclear membrane -inside the cytoplasm -inside the Golgi apparatus -on cell surface

-on cell surface antigens are foreign proteins which stimulate specific immune response (antibody) against itself when introduced into the body. they are present on the surface of the cell wall.

in ecosystems, energy flows in how many directions -one -two -the flow of energy is cyclical -the flow of energy is immeasurable because all living things have energy -none of the above

-one

which of the following statements is correct? (i) nitrogen fixation is the process of converting nitrogen gas to nitrate (ii) nitrification is the process of converting ammonium to nitrogen gas (iii) ammonification is the process of converting organic substance to containing nitrogen to ammonia and then to ammonium -only (i) is true -only (i) and (ii) are true -only (i) and (iii) are true -only (ii) and (iii) are true -only (iii) is true

-only (i) and (iii) are true only (ii) is false. Nitrification is the process of converting ammonium to nitrites and nitrates by soil bacteria

which of the following characterizes alcohol fermentation? 1. oxygen is the electron acceptor 2. there is not electron acceptor 3. triose phosphate is an electron donor, while pyruvic acid is the electron acceptor 4. triose phosphate is an electron donor, while acetyl aldehyde is an electron acceptor -only 1 -only 2 -only 3 -only 4 -2 but sometimes 3

-only 4 in alcohol fermentation, pyruvic acid is first decarboxylated to acetaldehyde in the presence of carboxylase. in this process, thiamine pyrophosphate (TPP) is required as a coenzyme. this acetaldehyde is then reduced to ethyl alcohol by an enzyme alcoholic dehydrogenase. thus, triose phosphate is the electron donor, while acetaldehyde is the electron acceptor.

which of the following statements is NOT true? -all populations can be described in terms of density and dispersion -only birth and death cause change in the size of all populations -a population is at carrying capacity when the birth rate is equal to the death rate -quadrats are used to sample a given population for the organisms that tend to stay in a spot all their lives -none of the above

-only birth and death cause change in the size of all populations the size of the population depends not only on birth and death but also on immigration (the movement of individuals into a population) and emigration (the movement of individuals out of a population). these four basic processes cause change in the size of all populations.

eggs are formed in the female reproductive system during which of the following processes? -oogenesis -folliculogenesis -oncogenesis -menstruation -spermatogenesis

-oogenesis

a section of DNA that contains an operator, promoter, regulatory region, and DNA sequences that produce mRNA for multiple proteins is referred to as a(n) -gene -operon -exon -intron -regulon

-operon if a region of DNA includes an operator promoter, regulatory region, and sequences that code for proteins, it is collectively referred to as an operon. DNA sequences that produce proteins are called genes. an exon is a region of DNA that codes for gene or a protein. introns are non-coding regions of DNA. a regulon is a group of genes or operons that are controlled by the same regulatory protein.

the region of a DNA sequence where replication begins is called the -replication fork -Okazaki fragment -origin of replication -primer -chiasma

-origin of replication

the function of the contractile vacuole in protozoa is -locomotion -osmoregulation -food digestion -reproduction -respiration

-osmoregulation the function of contractile vacuole is the removal of excretory substances, CO2 etc. it is very essential to regulate water content and it occurs by the process of osmoregulation.

the fertilization of the egg and sperm occurs in which of the following structures -uterus -oviduct -vas deferens -epididymis -cervix

-oviduct

in the mitochondria, the cristae acts as site for -protein synthesis -breakdown of macromolecules -oxidation - reduction reaction -phosphorylation pf flavoproteins -none of the above

-oxidation - reduction reaction cristae of mitochondria acts as sites for oxidation-reduction reaction. it releases energy through ATP and ADP reactions. the cristae greatly increases the inner surface area of the mitochondria to hold a variety of enzymes.

in the conversion of pyruvic acid to acetyl coA, the pyruvic acid is -oxidized -reduced -isomerised -broken into one-carbon fragment -none of the above

-oxidized pyruvic acid is converted to Acetyl CoA and CO2 by pyruvate dehydrogenase complex. this is an oxidation process in which the carboxyl group is removed from pyruvate, as a molecule of CO2 and the two remaining carbons become the acetyl group of Acetyl CoA.

which of the following was not abundant in the Earth's early atmosphere? -oxygen -methane -hydrocarbons -hydrogen -water

-oxygen

the first reaction in photorespiration is -carboxylation -oxygenation -darcarboxylation -phosphorylation -none of the above

-oxygenation the term photorespiration refers to the oxidation of glycolate produced during glycolate metabolism of photosynthesis. it uses O2 and produces CO2.

which of the following is NOT produced by the process of gluconeogenesis? -palmitate -lactate -alanine -glycerol -none of the above

-palmitate palmitate is a fatty acid and glucose cannot be produced from a fatty acid. glycerol is a sugar alcohol. alanine is an amino acid. lactate is the salt of lactic acid (milk acid)

the branch of the autonomic nervous system that calms and relaxes the body when necessary is the _________ division -sympathetic -motor -parasympathetic -sensory -afferent

-parasympathetic the autonomic nervous system regulates the involuntary functions of the body and it is divided into multiple branches. the parasympathetic branch contains both afferent (sensory) and efferent (motor) components that it uses to calm and relax the body when necessary. the sympathetic division prepares the body to deal with stress. the gathering of sensory information from the external world, through the senses and specialized receptors of the body, is handled by the sensory or the afferent division of the nervous system. the responses from then nervous system to the glands and tissues is handled by the efferent or motor division.

the sarcomere is the -part between two Z-lines -part between two A-lines -part between two I-lines -part between two H-lines -none of the above

-part between two Z-lines the sarcomere is a contractile element in a striated muscle fibril (myofibril). each sarcomere is joined to the next one by Krause's membrane (Z-lines). thick filaments of the protein myosin form a dark central A-band.

the gastric juice that is secreted by the stomach during digestion, contains which of the following? -amylase -chymotrypsin -bile -lipase -pepsin

-pepsin the gastric juice that is secreted by the stomach is very acidic and it contains hydrochloric acid (HCl) and pepsin. pepsin is an enzyme that digests proteins. amylase is an enzyme that digests starches and it is produced by the salivary glands and the pancreas. lipase is an enzyme that digests fats, and it is produced by the pancreas. bile is a substance that is also inlaid in fat digestion and it is produced by the liver and stored in the gall bladder. chymotrypsin digests proteins and it also produced by the pancreas.

which of the following substances is not secreted by the pancreas during digestion -amylase -glucagon -pepsin -insulin -trypsinogen

-pepsin the pancreas secretes hormones that regulate the level of sugar in the blood, as well as enzymes that digest food. glucagon and insulin are pancreatic hormones that control the amount of sugar in the blood. glucagon promotes the release of sugar into the blood. insulin promotes the uptake of sugar from the blood. amylase is part of the digestive juices released by pancreas into the small intestine, and it breaks down carbohydrate molecules. trypsinogen is a part of the pancreatic juices, and it breaks down proteins. pepsin is produced by the stomach and aids in protein digestion in the stomach.

a bundle of muscle cells is called a fascicle. what is the name of the connective tissue that surrounds the fascicle -endomysium -epimysium -perimysium -hypodermis -adipose

-perimysium each cell of the fascicle is covered by a thin layer of connective tissue called the endomysium. the fascicles are grouped together to form the belly or fleshy midsection of the muscle, and it is surrounded by a slyer of connective tissue called the epimysium. the hypodermic is a deep layer of the skin that is made up of adipose or fat tissue.

the outermost covering of a bone that is connected to tendons and ligaments, and attaches the bone to muscles and other bones is called the -endosteum -epiphysis -epiphyseal plate -periosteum -diaphysis

-periosteum

proteins that are anchored by the cytoskeleton and attached to the surface of the phospholipid bilayer are called -transmembrane proteins -ligands -channel proteins -peripheral proteins -none of the above

-peripheral proteins receptor proteins on the surface of the plasma membrane recognize specific molecules called ligands that bind to the cell. transmembrane proteins cross or span the membrane. channel proteins are generally transmembrane proteins.

the series of rhythmic muscle contractions that propel food through digestive tract is called -capillary action -segmented movement -peristalsis -mechanical digestion -none of the above

-peristalsis segmented movement is similar where smooth muscle cells in the small intestine contract and relax creating back and forth or oscillating movements. these motions mix the contents of the intestine and force it against the walls so that nutrients can be absorbed. mechanical digestion describes the physical breaking down of the food that takes place in the oral cavity and in the stomach. capillary action is the movement of a liquid against gravity, by rising in a narrow space.

natural selection acts on the ______ of the species in order to allow traits that are more advantageous to survive -genotype -phenotype -offspring -A & B -none of the above

-phenotype

in the C4 pathway of photosynthesis, CO2 combines with -phosphoenol pyruvate -oxaloacetic acid -RuDP -phosphoglyceric acid -none of the above

-phosphoenol pyruvate in the C4 pathway, the initial product is oxaloacetic acid. here, CO2 is accepted by phosphoenol pyruvate which is present in the chloroplast of mesophyll cells of C4 plants. C4 plants produce oxaloacetate. the fixation of CO2 occurs in the presence of the enzyme phosphoenolpyruvate carboxylase.

Phenylketonuria is a condition in which a genetic defect of a single locus causes multiple disorders such as mental retardation and improper skin pigmentation. Phenylketonuria is an example of a ___________ disease. -epistatic -pleiotropic -polygenic -multifactorial -Mendelian

-pleiotropic in epistatic, one gene's activity is modified by another gene's expression in polygenic inheritance, multiple genes control a single trait in multifactorial, multiple factors such as genetics, behavior, and environment cause the disease. in Mendelian, genetics teaches that one gene controls one trait. ***in pleiotropy a single gene controls many phenotypic traits in the individuals

Tay Sachs disease is a genetic disease in humans. Individuals who are carriers are not affected by the gene that causes Tay Sachs. Although the mutation that causes Tay Sachs disease only affects one nucleotide in a single gene, Tay Sachs disease affects the nervous system and causes paralysis, difficulty swallowing, and blindness, which of the following describes Tay Sachs disease -autosomal dominant -pleiotropic -multifactorial -polygenic -sex-linked

-pleiotropic not autosomal dominant since carriers are not affected. multifactorial are diseases that are influenced by multiple factors such as genes, diet, environment etc and Tay Sachs is described as a strictly genetic disease. polygenic diseases are caused by multiple genes. there is no mention of the disease affecting one sex more than the other so sex-linked is not it. it is pleiotropic because it is caused by a mutation in a single gene and affects multiple systems in the body.

the fluid inside the cavities of the membranes that lines the lungs is called -surfactant -aqueous fluid -vitreous fluid -peritoneal fluid -pleural fluid

-pleural fluid

embryonic cells that have the potential to differentiate into the cells that form the three germ layers of the body are classified as: -unipotent -senescent -multipotent -pluripotent -somatic

-pluripotent

the condition in which an individual is born with more than two complete sets of chromosomes is referred to as -monosomy -diploidy -trisomy -nondisjunction -polyploidy

-polyploidy

the aggregate of process that determine the size and composition of any population is known as -population dynamics -population dispersal -population explosion -population density -none of the above

-population dynamics all populations are dynamic. they change in size and composition over time and the whole process that determines the size and composition is known as population dynamics. population dispersion is the spatial distribution of individuals within the population population explosion is very fast population growth population density is the number of individuals per unit of area or volume

for the exponential model of population growth, which of the following is correct? -as the population size increase the rate of immigration decreases -at first population growth increases then decreases -population remains constant for certain time then increases -population growth stops at the carrying capacity -population growth continues indefinitely

-population growth continues indefinitely when the birth rate is more than the death rate, population grows. if the growth is slow at first and is very fast afterwards, the population growth is exponential. if we draw a graph taking 'time' along the horizontal axis and the 'population size' along the vertical axis, we will get a J-shaped curve. so in this model, population grows indefinitely.

glucose molecules are broken down during cellular respiration to produce chemical energy in the form of ATP, the type of energy present in the glucose molecule is referred to as -potential energy -kinetic energy -thermal energy -mechanical energy -power

-potential energy

the enzyme that adds a short RNA sequence to the growing DNA strand at the onset of DNA replication is referred to as -helicase -DNA polymerase -RNA polymerase -ligase -primase

-primase during DNA replication, a short RNA sequence that is called a primer is needed to initiate the replication process. this primer is added to the growing DNA strand by primase. DNA polymerase is the enzyme that adds nucleotides to the growing DNA strand it is unable to begin working without the primer. RNA polymerase is involved in RNA synthesis during the process of transcription. helices unwinds the double stranded DNA molecule at the onset of replication. ligase seals the newly formed DNA molecules together at the end.

which of the following hormones is called the "pregnancy hormone" because of its extensive role in gestation? -estrogen -progesterone -oxytocin -LH -FSH

-progesterone FSH triggers the development of follicles in the ovary. LH triggers the release of eggs from the ovary. estrogen slows down both FSH and LH, and prepares the uterus for the egg. if the egg is fertilized, gestation begins during which the embryo develops into a fetus. progesterone maintains gestation. oxytocin stimulates the contractions of the uterus during child birth.

the first type of cell that formed on earth was a(n) -amoeba -prokaryote -eukaryote -plant cell -none of the above

-prokaryote

during transcription, the DNA site at which RNA polymerase binds to, is called -regulator -enhancer -promoter -receptor -acceptor

-promoter initiator site or promoter end

the disappearance of the nuclear envelope occurs during what stage of mitosis -interphase -prophase -metaphase -cytokinesis -telophase

-prophase

the spindle apparatus appears during what stage of mitosis? -prophase -metaphase -anaphase -telophase -cytokinesis

-prophase order: prophase metaphase anaphase telophase cytokinesis

which of the following is correct about the 'fluid mosaic mode' -there is a phospholipid layer outside the protein layer -there is a protein layer outside the phospholipid layer -there is a phospholipid layer between two protein layers -protein molecules are embedded in the phospholipid layer -none of the above

-protein molecules are embedded in the phospholipid layer according to this model, a cell membrane consists of a phospholipid bilayer where many different types of proteins and cholesterol molecules are embedded and thus the name 'mosaic' is spawned.

during blood clotting, vitamin K is necessary for the synthesis of -fibrinogen -thromboplastin -prothrombin -factor XIII -proaccelerin or factor V

-prothrombin prothrombin is a plasma protein produced in the liver in the presence of vitamin K (and converted into thrombin during the clotting of blood) fibrinogen is a soluble plasma glycoprotein synthesized in the liver by hepatocytes and megakaryocytes thromboplastin is a protein present in subendothelial tissue, platelets and leukocytes. factor XIII is an enzyme of a blood coagulation system circulating in plasma proaccerlin or factor V is a protein circulating in plasma which is required in the conversion of prothrombin to thrombin

diatoms are -fungi -plantae -protista -protozoans -none of the above

-protista diatoms are freshwater or marine unicellular golden-brown algae with a cell wall consisting of two silica impregnated valves. they are included in the kingdom protista.

the Ras protein is normally involved in signal transduction pathways in the cell that result in cell growth and differentiation. Forty percent of human cancers involve a mutation in the Ras protein. The gene that codes for the Ras protein is an example of a(n) -proto-oncogene -growth factor -tumor suppressor gene -inhibitor -activator

-proto-oncogene genes that are normally involved in cell-growth, and are susceptible to mutations that cause cancer are called photo-oncogenes. since a gain of function in the Ras protein leads to excessive cell growth, ras is a proto-oncogene. tumor suppressor genes create tumor suppressor proteins, which are responsible for preventing the growth of tumors and maintaining the integrity of DNA in the cell. these proteins detect and correct mutations in DNA, and a loss of function would also lead to mutations in cellular DNA. inhibitors are molecules that turn off gene expression. activators turn on gene expression. growth factor is a protein hormone that primates cellular growth.

in a crocodile-bird association, a bird enters the mouth of a crocodile and feeds on parasitic leeches. the bird gets food and the crocodile gets rid of blood sucking leeches. both the partners can live independently also. such an association is called -mutualism -commensalisms -amenalism -protocooperation -none of the above

-protocooperation protocooperation is a type of relationship which is beneficial to both partners, but is not obligatory for their survival. this relationship has been also termed as non-obligatory mutualism.

members of kingdom Animalia in which the mouth forms before the anus during embryonic development are called -deuterostomes -protostomes -coelomates -acoelomates -zygotes

-protostomes deuterostomes, the anus forms first. coelomates are animals that have a complete body cavity. acoelomates are animals that do not have any body cavities. a zygote is a fertilized egg.

the role of placenta is to -provide nutrition to developing embryo -protect embryo from shock -act as a storage organ -convey nerve impulses -all of the above

-provide nutrition to developing embryo in the uterus, the embryo develops by collecting nutrients and oxygen from the mother's blood through placenta. also, the waste products and CO2 are transferred back to the mother's blood from the fetus through placenta.

when two genetic loci produce identical phenotypes in cis and trans position, they are considered to be -different genes -pseudoalleles -multiple alleles -parts of same gene -none of the above

-pseudoalleles the pseudoalleles are two genetic loci producing identical phenotypes in both the cis and trans position.

animals that have a fluid-filled body cavity that is partially lined with mesodermal tissue are called -coelomates -pseudocoelomates -acoelomates -invertebrates -echinoderms

-pseudocoelomates animals that have a body cavity that is completely surrounded by mesodermal (connective) tissue are called coelomates. those that contain a fluid-filled body cavity that is partially lined with mesodermal tissue are called pseudocoelomates. acoelomates are animals that do not have any body cavities. invertebrates are animals that do not have a vertebral column. echinoderms are marine invertebrates that have radial symmetry, such as sea stars (star fish) and sea urchins.

blood travels through the _________ to the lungs, where it becomes re-oxygenated and turns bright red -aorta -pulmonary arteries -pulmonary veins -coronary arteries -coronary veins

-pulmonary arteries in the circulatory system blood is oxygenated in the lungs, and deoxygenated in the tissues. generally, the blood vessels that carry blood away from the heart are called arteries. while those that return blood to the heart are called veins. arteries generally carry oxygenated blood, while veins carry deoxygenated blood. there is one pair of exceptions. the pulmonary arteries carry deoxygneated blood to the lungs, where it receives fresh oxygen and turns bright red. likewise, the pulmonary veins carry oxygenated blood back to the left side of the heart. the blood then travels through the aorta which is the largest artery in the body to return to the tissues of the body. the coronary arteries carry oxygenated blood to the muscles of the heart, and the coronary veins carry deoxygenated blood from the muscles of the heart back to the right atrium of the heart.

j-receptors are found primarily in the -pons and medulla -trachea -pulmonary microvessels -pharynx -none of the above

-pulmonary microvessels J-receptor, or juxtapulmonary-capillary receptors, are located in or near the walls of pulmonary microvessels. they are normally dormant but are estimated by an increase in lung interstitial pressure caused by the collection of fluid in the interstitium. this results in rapid shallow breathing and a dry cough.

the manner in which species are distributed relative to each other throughout a community, defines what feature of the community -temporal structure -spatial structure -biodiversity -biogeography -geological distribution

-spatial structure

which of the following does NOT open into right atrium? -pulmonary vein -coronary sinus -thebesian vein -superior vena cava -none of the above

-pulmonary vein the four pulmonary veins carry oxygen-rich blood from the lungs to the left atrium. coronary sinus receives blood from the myocardium of the heart and drains directly into the right atrium. the superior vena cava carries deoxygenated blood from the upper half of the body to the right atrium. the thebesian veins are small venous tributaries that directly drain into the cardiac chambers. they primarily exist in the right atrium and right ventricle.

the macromolecule that provides both structure and enzymatic activity for ribosomes is -mRNA -rRNA -tRNA -ATP -GTP

-rRNA

the bone in the forearm that is located on the same side of the arms as the thumb is the -ulna -carpals -tibia -radius -fibula

-radius

cartilaginous joints are NOT found in which of the following? -talus of the foot -radius-ulna joint -pubic symphysis -epiphysial plates -intervertebral discs

-radius-ulna joint radius-ulna joint is connected by dense connective tissue consisting mainly of collagen; therefore, it is a fibrous joint wile the others are connected by cartilage.

which of the following does not decrease the genetic diversity of the individuals in a population? -inbreeding -bottleneck -recombination -founder effect -stabilizing selection

-recombination recombination makes it easier for natural selection to target individual genes while avoiding the potentially disadvantageous effect of simultaneously reducing diversity at neighboring genes

the hormones released by placenta during its formation are -testosterone and progestorone -FSH and LH -relaxin and estrogen -only relaxin -follicle stimulating hormone and aldosterone

-relaxin and estrogen relaxin and estrogen hormones are released by placenta. relaxin helps in the enlargement of the birth canal by the relaxation of pelvic ligaments and growth of the vagina. estrogen helps in the maintenance of the corpus luteum during pregnancy.

which of the following anatomical structures in the kidney is the site of ultrafiltration -renal pelvis -renal pyramid -renal capsule -renal cortex -renal medulla

-renal cortex the renal medulla is the innermost part of the kidney, and it splits into sections called renal pyramids. functionally, the medulla and pyramids regulate salt and water balance in the blood. urine flows through the renal pelvis on its way to ureter. the renal capsule is the region of the kidney that is composed of fatty, fibrous layers of tissue. the capsule surrounds the kidney and protects it from trauma and physical damage.

which of the following hormones is not produced by the adrenal gland and regulates the blood pressure in the body? -aldosterone -cortisol -renin -cortisone -epinephrine

-renin the adrenal gland produces multiple hormones that are involved in multiple processes in the body. aldosterone, cortisol, cortisone, and epinephrine are all produced by the adrenal gland, which sits on top of the kidney. renin is produced by specialized cells of the kidney, and works with angiotensin from the liver to regulate blood pressure. aldosterone triggers the absorption of sodium in the kidneys and raises the blood pressure. cortisol is the stress hormone that promotes the release of sugar into the blood stream. cortisone is a similar hormone to cortisol and it also responds to stress. epinephrine is also called adrenaline, and it is release in the body in response to stress.

the active transport mechanism is "against the concentration gradient" and -requires ATP -does not require ATP -does not require ATP -does not require protein -requires neither ATP nor protein

-requires ATP with the help of ATP, the movement of molecules across a membrane, against concentration gradient, is called active transport.

which of the following cuts DNA from specific places? -restriction endonuclease -ligase -exonuclease -phosphatase -none of the above

-restriction endonuclease a group of enzymes known as restriction endonucleases can cause a cut at a specific site in the DNA segment. this cut results in sticky ends at either side of the DNA molecules.

manipulation of DNA in genetic engineering became possible due to the discovery of -DNA ligase -primase -transcriptase -restriction endonuclease -none of the above

-restriction endonuclease genetic engineering means manipulation of gene and it depends upon recombinant DNA technology. the cornerstone of recombinant DNA technology is a class of enzymes called restriction endonuclease. it is used for cutting DNA in genetic engineering.

in genetic engineering, which of the following are used to cut DNA strands at specific sequences before inserting them into host cells? -DNA ligase -restriction enzyme -vectors -primase -DNA polymerase

-restriction enzyme in genetic engineering, genes undergo "cut and paste" procedures for insertion into new genomes. the DNA is cut by restriction enzymes, which are endonuclease that cut DNA at specific internal nucleotides. DNA ligase is used to seal these DNA fragments together. vectors are vehicles that bring foreign DNA into a cell, examples include plasmids and viruses. DNA polymerase is the enzyme that builds new DNA molecules. primase provides the primer (RNA sequence) that initiates DNA synthesis.

which of the following is used to cut DNA molecules at specific locations? -restriction enzymes -cloning vectors -polymerase chain reaction -gel electrophoresis process -none of the above

-restriction enzymes to cut DNA molecules bacterial proteins are used. these are called restriction enzymes. these enzymes recognize specific short DNA sequences and cut the DNA in or near the sequence. polymerase chain reaction can produce copies of DNA fragments. these DNA fragments are sorted by size by a technique called gel electrophoresis. DNA fragments are clones by inserting them into cloning vectors.

variations that occur among individuals at a single position in their genomes are called -short tandem repeats -long tandem repeats -single base extension -single nuclear polymorphisms -none of the above

-single nuclear polymorphisms polymorphism means different genotypes at the same position in a DNA sequence. when that polymorphism is caused by a variation in a single base pair, it is referred to as a single nuclear polymorphism. tandem repeats are patterns of adjacent repeated nucleotides in a DNA sequence. they can be both short and long. a single base extension involves the addition of a single base pair to a DNA strand.

in the cardiovascular system, the heartbeat originates in the -atrioventricular nodes -Purkinje fibers -sinoatrial nodes -bundle of His -septum

-sinoatrial nodes

a pair of chromosomes that contain identical genes at identical loci are referred to as -sister chromatids -homologous chromosomes -recombinants -alleles -diploids

-sister chromatids if the pair of chromosomes are genetically identical, they are referred to as sister chromatids. a chromatid is one of a pair of identical copies of a chromosome that appears during cell division. homologous chromosomes are the same size, carry the same genes, and each of a pair was received from one parent. therefore, they are not genetically identical. alleles are different forms of a gene. diploids are organisms that have two sets of chromosomes in their genomes. recombinants are cells or organisms that have different combination of genes from their parents.

oxygen-rich muscle fibers that give the muscle its characteristic red color are called ______ fibers -slow twitch -fast twitch -white muscle -B & C -none of the above

-slow twitch there are two general groups of skeletal muscle fibers: -slow twitch -fast twitch slow twitch (oxidative fiber) are oxygen rich and well suited for intense aerobic activity. the vascular supply to these muscles is very dense, and this gives the muscle its red color. fast twitch are non-oxidative. they carry less oxygen, and appear white. some fast twitch fibers are anaerobic and use glycolysis to produce energy.

which is incorrect about the smooth muscle cell? -smooth muscle cells have single nuclei -z-line is absent -troponin is absent -smooth muscle contains more protein than skeletal muscle -none of the above

-smooth muscle contains more protein than skeletal muscle smooth muscle cells, in general, have a single nucleus. smooth muscle cells contain myofibril but z-line is absent in them. also, the protein troponin is absent in smooth muscle. instead there are two other proteins, caldesmon and calponin. the protein content in smooth muscle is 110 mg/g muscle and that in skeletal muscle is 200 mg/g muscle.

genetic mutations that are not passed on to an individual's offspring are referred to as ________ mutations -silent -missense -somatic mutation -nonsense -germ cell

-somatic mutation most genetic mutations are acquired in the somatic cells of the body. the somatic cells include all of the cells in the body except for the sex or germ cells that produce gametes (egg and sperm). therefore, mutations that are not passed on hereditarily are referred to as somatic mutations. mutations that are passed on through hereditary are found in the germ cells. if a mutation is silent the change results in the production of the same gene product and no differences are noticed in the individual. if the mutation results in the production of another gene product, a missense mutation has occurred. if the mutation results in a shortened or non-functional gene product, a nonsense mutation has occurred.

the chemical messenger which inhibits HCl secretion in the stomach is -histamine -acetylcholine -gastrin -somatostatin -none of the above

-somatostatin all four chemical messengers regulate the HCl secretion. during a meal, the rate of HCl secretion increases. histamine, acetylcholine, and gastrin regulate the insertion of H+/K+-ATPases into the parietal cell membrane of the stomach and thus are responsible for the acidification of the stomach contents. when the concentration of HCl increases, the H+ ions stimulates the release of somatostatin. somatostatin is important because not only does it inhibit HCl secretion but it also inhibits the release of gastrin and histamine.

a sperm becomes mature from the immature stage through a process called -spermiogenesis -spermatogenesis -spermatocytosis -gametogenesis -none of the above

-spermiogenesis spermatogenesis is the process by which male spermatogonia develops into a mature spermatozoa. spermatogenesis is broken down into 3 stages: 1. spermatocytogenesis/spermatocytosis - includes the first gamete mitosis and meiosis I, producing two diploid primary spermatocytes and haploid secondary spermatocytes. 2. spermatogenesis - short-lived meiosis II producing four haploid 3. spermiogenesis - maturation of spermatocyte gametogenesis - the female form of spermatogenesis.

which of the following organelles, or parts of a cell, has a single membrane? -nucleus -mitochondria -cell wall -spherosome -all of the above

-spherosome spherosome is a small spherical organelle which is covered by a single membrane. spherosome synthesizes and stores fat.

Francisco Redi performed a groundbreaking experiment, in which he placed meat in three jars. The first jar was unsealed, the second was sealed, and the third was covered with gauze. Redi noticed that the unsealed jar had maggots on the meat, the sealed jar had no maggots on the meat, and the jar covered with gauze had a few maggots on the gauze but none on the meat. What theory did Redi's experiment disprove -biogenesis -spontaneous generation -alchemy -germ theory of disease -cell theory

-spontaneous generation

a mother goose will roll an egg that has been displaced from her nest back into the nest in the same manner every time. this is an example of -foraging -memory -stereotype -cognition -altruism

-stereotype behaviors that an animal always repeats are called action patterns or stereotypes. since the goose always performs the same routine, this is an example of a stereotype. memory is the ability to recall past events. cognition refers to mental processes that deal with information analysis and retrieval. in altruism animals behave in the interstate's of other community members while risking their own survival. foraging describes how an animal searches for food.

a detectable change in an animal's external or internal environment is referred to as a(n) -action pattern -stereotype -stimulus -kinesis -taxis

-stimulus action patterns or stereotypes are behaviors that an animal always repeats. kinesis is a change in behavior in response to a stimulus. taxis is a movement of the animal towards or away from a stimulus.

cellular division occurs in which of the following layers of the skin? -dermis -hypodermis -stratum basale -stratum lucidum -stratum corneum

-stratum basale stratum lucidum is a layer of clear dead cells in the epidermis stratum corneum is a layer of dead cells on the surface of the skin

which of the following represents the sublayer of skin containing a melanocyte cell? -stratum corneum -stratum lucidum -stratum granulosum -stratum spinosum -stratum basali

-stratum basali the outer layer epidermis of the skin is made of five sublayers. the sublayer stratum basali is made of melanocyte cells which produce melanin pigments. this pigment forms the color of the skin.

the layer of the skin that is only found on the palms and soles of the hands and feet, is referred to as the -dermis -stratum lucidum -stratum basale -stratum corneum -stratum granulosum

-stratum lucidum the stratum granulosum is the layer of the skin from which hair and fingernails grow. the stratum corneum is the layer of dead cells on the surface of the skin.

which of the following is a salivary gland? -adrenal -pituitary -Brunner -lacrimal -sublingual

-sublingual three pairs of salivary glands: -parotid -submaxillary -sublingual

during glycolysis, the oxidation of a glyceraldehyde 3-phosphate (PGAL) occurs by the removal of electrons accompanied by hydrogens, which are picked up and NAD+ to form NADH. at the same time, ADP is converted to ATP by the transfer of phosphate group from PGAL, this is an example of -oxidative phosphorylation -substrate level phosphorylation -catalysis -A & C -none of the above

-substrate level phosphorylation

a genetic cross between an individual of unknown genotype with an individual of known genotype is referred to as a -monohybrid cross -pedigree -dihybrid cross -test cross -validation cross

-test cross

in Krebs' cycle, the FAD participate as electron acceptor during the conversion of -succinic acid into fumaric acid -succinyl Co-A into succinic acid -fumaric acid into malic acid -alpha-ketoglutarate into succinyl Co-A -none of the above

-succinic acid into fumaric acid in the Krebs' cycle, during the conversion of succinct acid into fumaric acid, the hydrogen of NADH+ is accepted by FAD. as a result of this, FAD is reduced to FADH+. Krebs' Cycle: Acetyl CoA + oxaloacetate + H2O <-> citrate citrate <-> aconitate + H2O aconitase +H2O <-> isocitrate isocitrate <-> a-Ketoglutarate + NADH + CO2 a-Ketogluterate + NADH+ <-> succinyl CoA + NADH + CO2 succinyl CoA +GDP <-> succinate + GTP succinate + FAD+ <-> fumarate + FADH2 fumarate + H2O <-> malate malate + NAD+ <-> oxaloacetate + NADH _____________________________________________________ Acetyl CoA + 3NAD + FAD+ = GDP + 2H2O <-> oxaloacetate + CoA + 3NADH + FADH2 + GTP + 2CO2

which of the following is NOT correct -superficial nephrons constitute 85% of total nephrons where as juxtamedullary nephrons constitute 15% -superficial nephrons lie at the center, juxtamedullary nephrons lie at the junction of cortex and medulla of kidney -superficial nephrons produce more concentrated urine than juxtamedullary nephrons -superficial nephrons are smaller, juxtamedullary nephrons are larger -none of the above

-superficial nephrons produce more concentrated urine than juxtamedullary nephrons according to position, there are two types of nephrons. superficial nephrons are present in the outer part of the cortex. they constitute 85% of the total nephrons. they are smaller and act in normal conditions and produce less concentrated urine. juxtamedullary nephrons are present in the inner part at the junction of cortex and medulla. they constitute 15% of the total nephrons. they are larger. they work under the conditions of stress and produce concentrated urine.

as the size of a cell increases, the -surface area to volume ratio decreases -surface area to volume ratio increases -surface area to volume ratio stays the same -surface area to volume ratio doubles -none of the above

-surface area to volume ratio decreases

the space between two neurons in which they can release neurotransmitters to communicate with each other is referred to as the: -synapse -gap junction -active site -voltage-gated channel -active zone

-synapse

the most movable articulation in the body are known as -fibrous joints -cartilaginous joints -synovial joints -sutures -none of the above

-synovial joints

birds have a special sound producing organ called the -larynx -glottis -esophagus -syrinx -none of the above

-syrinx at the posterior end, or base of the trachea, or at its junction with the bronchi there is a special structure called syrinx or voice box. it helps to produce sound in birds.

during protein synthesis amino acids are brought over by -DNA -tRNA -rRNA -mRNA -nucleotides

-tRNA forming proteins is based on DNA information, which is then carried out by RNA. this process is called protein synthesis. each type of RNA plays a different role in protein synthesis. transfer RNA transfers amino acids to the ribosome to make a protein.

normally, DNA molecules have A-T and G-C pairing. However, these bases can exist in an alternative valency status, owing to rearrangements due to -point mutation -analog substitution -frame-shift mutation -tautomerisational mutation -none of the above

-tautomerisational mutation in a normal molecule of DNA, the adenine is linked to the thymine by two H-bonds. if these bases exist in alternate states (which is rare), then these are called tautonomers and occur due to tautomerisational mutation. this situation, known as copy error, is not stable because at the next replication tautomers returns to their common state.

in which phase of mitosis does the nucleolus reappear? -interphase -prophase -metaphase -anaphase -telophase

-telophase telophase is the reverse process of prophase. a nuclear envelope reforms around each chromosome set. the spindle the disappears, the nucleolus reforms and cell division by mitosis is complete at this point.

many animals and plants that are found in deserts are dormant most of the year. the plants germinate in response to seasonal rains. what property of a community does this describe? -spatial structure -trophic structure -species diversity -species richness -temporal structure

-temporal structure the appearance and timing of a species' activities in a community is referred to as the temporal structure of the community. the spatial structure describes the way in which species in a community are distributed relative to each other. the species richness is the number of species in the community. the species diversity is the number of different types of species in the community. the trophic structure describes the feeding hierarchy of the community.

to be biologically active, proteins must adopt specific folded ______ structures -primary -secondary -tertiary -quaternary -amino acid

-tertiary the primary structure is the string of amino acids. the secondary structure involves folds or arrangements of the protein called alpha helices and beta sheets. the protein becomes biologically active with its tertiary structure. the tertiary structure is the functional organization of the protein's secondary structure.

the males in a species express all of their sex-linked genes. this occurs because -the Y chromosome is smaller than the X chromosome -the X chromosome is smaller than the Y chromosome -there are more genes on the X chromosome than there are on the Y chromosome -there are more genes on the Y chromosome than there are on the X chromosome -the Y chromosome does not have complementary alleles for some of the genes on the X chromosome

-the Y chromosome does not have complementary alleles for some of the genes on the X chromosome although the Y chromosome is smaller than the X chromosome, this does not explain why males express all of their sex-linked genes. likewise, the X chromosome has more genes, but this is not an explanation for the observation. most genes are controlled by two alleles, and individuals receive one copy of each gene from each parents. females always receive an X chromosome from their mothers, and one from their fathers. males receive their X chromosome from their mothers. therefore, if the mother's X chromosome has an affected allele or gene, the trait will be expressed in her male offspring because he received only one copy of the X chromosome. the Y chromosome does not have any complementary alleles for many of the genes on the X chromosome. in addition, if the sex-linked disease is carried on the Y chromosome, only male can be affected by it.

prokaryotic cells are characterized by -the absence of nuclear membrane -the absence of genetic material -the presence of distinct chromosomes -the presence of distinct nuclear membrane -the presence of ribosome

-the absence of nuclear membrane prokaryotes lock a membrane bound nucleus and membrane-bound organelles, though their genetic information, which is in the form of DNA, is often concentrated in a part of the cell called the nucleoid.

which one is NOT correct regarding nemathelminthes? -the alimentary canal is incomplete -all the members show bilateral symmetry -circulatory and respiratory systems are absent -sexes are separate and sexual reproduction occurs -none of the above

-the alimentary canal is incomplete it is complete and straight with a mouth and anus at opposite ends

in a hypotonic solution -fluid flows out of the cell -the concentration of the solutes is greater outside the cell -the concentration of the solutes is greater inside the cell -the concentration of the solutes is equal on both sides of the cell -A & C

-the concentration of the solutes is greater inside the cell

point mutation occurs due to -the interchange of a pair of bases -the interchange of two pairs of bases -the interchange of more than two pairs of bases -the deletion of a single nucleotide -the addition of extra nucleotides

-the deletion of a single nucleotide when a single base is substituted by another, point mutation occurs. generally the substitution, addition or removal of a single nucleotide is a point mutation, a change that occurs within a single gene or other segment of DNA on a chromosome.

the electron transport system (ETS) occurs in -the matrix of mitochondria -the inner membrane of mitochondria -the outer membrane of mitochondria -the head and stalk of F1 particle -none of the above

-the inner membrane of mitochondria electron transport system or electron transport chain occurs in the inner membrane of mitochondria by the respiratory assemblies like NADH, FADH2. electrons from NADH, FADH2 are passed through an electron transport chain to oxygen, which is reduced to water.

hair pin loop is formed during -the last stage of transcription -the first stage of transcription -the last stage of translation -the first stage of translation -none of the above

-the last stage of transcription transcription stops when the newly synthesized RNA molecules forms a G-C rich hairpin loop, followed by a run of U's, which makes it detach from the DNA template.

the bicuspid valve regulates the flow of blood from -the left ventricle to the SA node -the right auricle to the right ventricle -post caval to the right auricle -the left auricle to the left ventricle -left ventricle to aorta

-the left auricle to the left ventricle the bicuspid valve consists of two membranous flaps or cusps situated between the left auricle and left ventricle of mammals. it regulates the flow of blood from the left auricle to the left ventricle in the heart.

what property of detergents, such as sodium dodecyl sulfate (SDS) makes them useful for disrupting the plasma membrane during experiments -they are amphiphilic -they are fully hydrophobic -they are acidic -B & C -all of the above

-they are amphiphilic SDS has both hydrophilic and partially hydrophobic properties. detergents are alkaline substances so they cannot be acidic.

invertebrates: vitelline envelope :: mammals: ________ -zona pellucida -cumulus -corona radiata -cortex -plasma membrane

-zona pellucida vitelline envelope in invertebrates and zona pellucida in mammals play the same role during fertilization: it binds the sperm, and it initiates the acrosomal reaction after the sperm is bound.

in photosynthesis, NADPH and ATP are produced during -the light-dependent reaction -the Calvin cycle -electron transport chain -glycolysis -cellular respiration

-the light-dependent reaction photosynthesis is divided into the light dependent and dark reactions. the light dependent reaction takes place and involves the use of sunlight to create chemical energy in the plant cell. the molecules that store this chemical energy are NADPH and ATP. the dark reaction is also called the Calvin Cycle and used the energy in NADPH and ATP to produce more stable compounds such as glucose. glycolysis is the process by which glucose is broken down to yield chemical energy in the form of ATP, but it is not a part of photosynthesis. cellular respiration is a process that takes place in the mitochondria of the cell in which the end products of glycolysis are used to produce a considerable amount of energy in the form of ATP. the electron transport chain (ETC) is a component of cellular respiration.

the carrying capacity of a species is determined by -the growth rate -the birth rate -the death rate -the limitation of the wealth -all of the above

-the limitation of the wealth the number of individuals the environment can support over a long period of tine is the carrying capacity of the population. below the carrying capacity, population increases as the birth rate is more than the death rate and above it, population decreases. the carrying capacity of an environment may change over time due to a variety of factors including food availability, water supply, environmental conditions and living space.

during telophase, in mitosis, which of the following occurs -chromosome condense -the nuclear envelope reappears -chromosomes are pulled to opposite sides of the cells -A & B -A & C

-the nuclear envelope reappears telophase is the fourth and final stage of mitosis, which is the process of nuclear division. during telophase the chromosomes de-condense. the nuclear envelope reappears, and cytokinesis follow immediately. chromosomes condense during prophase, which is the first step of mitosis. this is followed by the lining up of the chromosomes across the center of the cell, during metaphase. the chromosomes are pulled apart during anaphase.

a population cannot achieve Hardy-Weinberg equilibrium when -mating is random -the population is small -there is no gene flow -no mutations occur -there is no natural selection

-the population is small the Hardy-Weinberg equilibrium or principle describes the distribution of alleles for a specific trait in a population.

during transcription, RNA polymerase is added to -the promoter of DNA -the operator of DNA -the enhancer of DNA -the regulator of DNA -none of the above

-the promoter of DNA the promoter region occurs "upstream" (toward the 5' region) of the DNA coding region and provides a binding site for the RNA polymerase, which is necessary to initiate transcription. the operator region of DNA is classically defined as the region between the promotor and the coding region significant for repressor/activator binding. the enhancer region can occur "upstream" or "downstream" from the DNA coding region, binding transcription factors that stabilize and enhance gene transcription. regulatory regions of DNA generally occur a short distance "upstream" from coding regions and bind transcription factors to regulate transcription. this term can encompass sequences within the promotor such as the TATA box

the glucose molecule is very stable; however it is regularly broken down by the cell to produce energy and smaller molecules such as CO2 and H2O. which law of thermodynamics explains this phenomenon -the zero law -the first law -the second law -the third law -the fourth law

-the second law second law is the law of entropy, so it's moving towards disorder.

which of the following is not a mode of passive transport in the cell -diffusion of gases into the cell -osmosis of water out of the cell -facilitated diffusion of substances out of the cell -facilitated transport of substances into the cell -the sodium potassium pump

-the sodium potassium pump

"normal parents can have a color blind child" - which of the following is correct? -the statement is true -the statement is false -50% of the child have a chance to be color blind -only the girl child will be color blind -insufficient data

-the statement is true if the mother is a carrier of the X-linked recessive gene for color blindness, the parents can have a color blind child. in this case, one male child out of four children has a chance to be color blind.

which of the following is a denitrifying bacteria -rhizobium -chlorobium -thiobacillus -nitrobactor -none of the above

-thiobacillus denitrifying bacteria turn nitrogenous compounds back into nitrogen gas or nitrous oxide. thiobacillus is a denitrifying bacteria.

sympathetic nerves in mammals arise from -cervical region -thoraco-lumbar region -sacral region -3rd, 7th, 9th, 10th cranial nerves -none of the above

-thoraco-lumbar region the sympathetic nervous system is one of the two divisions of the autonomic nervous system which supplies motor nerves to the smooth muscles of internal organs and to the heart muscles. sympathetic nervous fibers arise via spinal nerves in the thoracic and lumbar regions.

which of the following endocrine glands store its secretion in the extracellular space before discharging it into the blood? -testis -adrenal -thyroid -pancreas -none of the above

-thyroid the thyroid gland stores its hormonal secretion thyroglobulin in follicles called extracellular space before discharging it into the blood.

the function of the surfactant is -to increase the surface tension of alveoli -to decrease the surface tension of alveoli -to protect pleura -to increase immunity power -none of the above

-to decrease the surface tension of alveoli the secretion of surfactant starts at week 28 of human gestation and becomes physiologically useful at about week 34. the alveolar cells secretes a surfactant, consisting phospholipids, into the fluid bathing the lungs. the surfactant increases the compliance of the lungs by decreasing the surface tension of alveoli and thus enables the alveolar cells to touch each other without sticking together.

the function of p factor is -to start transcription -to end transcription -to control the spreading of phase of transcription -to end translocation -none of the above

-to end transcription p factor is a protein found in prokaryotes, generally in E. coli. it functions solely at the end of transcription.

if a human egg is fertilized in vitro, what types of cells are formed -germ -gametes -totipotent -pluripotent -multipotent

-totipotent in vivo - in a living system in vitro - outside of a living system totipotent cells give rise to all cell types including embryonic cells.

which of the following is not a method of cellular fractionation -gel electrophoresis -centrifugation -transfection -A & B -none of the above

-transfection fractionation is the separation of cellular components in the cell. transfection is the introduction of a new gene into a cell.

the insertion of a new gene into an organisms genome is accomplished using which of the following techniques -cloning -transgenics -amplification -forward genetics -reverse genetics

-transgenics there are numerous molecular genetics technique that involve changing an organisms genome. cloning - a new organism is created that is genetically identical to its parent. with this technique, new genes are not introduced into the new offspring. transgenics - a new gene is incorporated into an organism's genome. examples of transgenics includes the use of plasmid DNA as vectors to incorporate insulin producing genes into bacteria. amplification - the process by which a gene or DNA sequence is copied many times. forward genetics - mutations are introduced into an organisms genome using a mutagen to identify gene function. reverse genetics - "knockout" organisms are created in which genes have been deleted from their genomes. this allows researchers to investigate the phenotype associated with the deleted gene.

ecotone is characterized by -terrestrial ecosystems -forest ecosystems -fresh water ecosystems -transition zones between two vegetation -transition zone between water and land

-transition zones between two vegetation ecotone is a boundary between two plant communities of major rank, or a transition zone between two adjacent biomes containing some organism from adjacent biomes and some characteristic ones, restricted to the zone itself.

the cellular process in which nucleotides are converted into a string of amino acids is called -replication -transcription -translation -synthesis -transformation

-translation

in the plasma membrane, proteins that span the entire length of the cell membrane are called -receptors -cell adhesion molecules -transmembrane proteins -peripheral proteins -extracellular matrix

-transmembrane proteins

genes that have the ability to move themselves across the genome to new positions are called -operons -transposons -introns -exons -inducers

-transposons operons are regulatory units or a set of genes that work under the same promoter. introns are the regions of DNA that contain non-coding regions. exons are coding regions in the DNA inducers are molecules that start or induce gene expression

DNA elements, which can switch their position, are called -transposons -introns -exons -cistrons -none of the above

-transposons the transposons are genetic elements which were originally discovered in the maize plant by B. McClintock. it is responsible for turning the expression of a gene on or off.

which is the most important characteristic of mollusca -have a hard shell made of calcium carbonate covering the body -have a muscular foot for locomotion -visceral organs are covered with mantle -do not have segmented bodies -have radula

-visceral organs are covered with mantle all mollusks have visceral mass which contains heart and the organ of digestion, excretion and reproduction. a layer of epidermis, called the mantle, covers the visceral mass. octopus and some other animals of cephalopoda class do not have hard shell. the animals of the class monoplacophora have segmentation in internal parts of the body. the animals of bivalvia class have a pair of labial pulp instead of radula. caudofoveata do not have a muscular foot.

human beings are -ovoviviparous -oviparous -viviparous -parthenogenetic -none of the above

-viviparous in viviparous animals, the embryo develops inside the body of the mother. oviparous animals lay eggs and they have little or no other embryonic development within the mother. in the case of ovoviviparous animals, the eggs are hatched inside the mother's body. the females of parthenogenetic animals produce eggs which develop without fertilization.

in humans, the gonadal rudiments remains sexually indifferent until -week 1 -week 4 -week 7 -week 13 -week 21

-week 7 the gonadal rudiment can develop into either an ovary or testes. the path of differentiation taken by this rudiment determines the future sexual development of the organism. before this decision is made, the human gonad first develops through an indifferent stage, during which time it has neither female nor male characteristics. thus, rudiment remains sexually indifferent until week 7. week 5 - neural development, notochord, spinal groove, primitive heart formation week 6 - ears, arms, visceral organs week 7 - eyes, hand and feet buds week 8 - digits forming, gonad differentiation, lungs week 9 - all essential organs, hair follicles

if for a reaction delta G > 0, then it -will proceed easily -will not proceed -may proceed but needs an enzyme -generates water -none of the above

-will not proceed a reaction cannot proceed if delta G is greater than 0. enzymes are catalysts and can change the rate of the reaction but they cannot start it.

the word 'prokaryote' means a cell - -with one nucleus -with more than one nucleus -with diffused nucleus -without nucleus -without chloroplast

-without nucleus the word prokaryote means before nucleus. prokaryotes are organism which do not have a membrane-bound nucleus or membrane-bound organelles. the DNA and plasmids of a prokaryotic cell are concentrated in a part of the cell called the nucleoid

in a developing embryo, the first blood cell originates at which of the following sites? -amniotic sac -yolk sac -allantois -chorion sac -none of the above

-yolk sac four membranes are formed surrounding the developing embryo. the first membrane is the fluid-filled amniotic sac. next is the yolk sac where the first blood cell originates. the allantois forms near the yolk sac, while the chorion surrounds all the other membranes.

which of the following extra embryonic membranes is involved in the formation of red blood cells in the human embryo -amnion -chorion -allantois -yolk sac -none of the above

-yolk sac like the tissues of the body, the extra-embryonic membranes are derived from the three germ layers: endoderm, mesoderm, and ectoderm. they are not considered part of the embryo, because they are lost at birth. there are four such membranes in land vertebrates: amnion, chorion, allantois, and yolk sac. the amnion surrounds and protects the embryo. the chorion transfers nutrients from the mother to the fetus. the allantois is involved in gas exchange. the yolk sac helps in formation of red blood cells and its also present in humans.

the structure in the early two-layered embryo that gives rise to the digestive tube, but does not provide the embryo with nutrients is the -epiblast -amnion -allantois -yolk sac -chorion

-yolk sac the amnion surrounds and protects the embryo. the chorion transfers nutrients from the mother to the fetus. the allantois is involved in gas exchange. none of these give rise to the digestive tube. the digestive tube evolves from the yolk sac. the yolk sac is the first gestational sac observed during human pregnancy and it gives rise to the digestive tube. the yolk sac also helps in the formation of red blood cells. the epiblast is the early embryonic tissue that gives rise to the three early germ layers during gastrulation.

the umbilical cord is formed from -yolk sac and amnion -yolk sac and chorion -yolk sac and allantois -amnion and chorion -chorion and allantois

-yolk sac and allantois during gastrulation, at one side of the chorion a small fingerlike projection is formed, called chorionic villi. this chorionic villi and a portion of uterine lining form placenta. the umbilical cord develops from and contains remnants of the yolk sac and allantois.

which of the following is not a coenzyme? -vitamin C -zinc -biotin -vitamin B12 -none of the above

-zinc cofactors and coenzymes are molecules that enzymes need to carry out their reactions. cofactors are generally inorganic compounds such as metal ions, while coenzymes are organic compounds. by definition, organic compounds are those that contain carbons. vitamins are organic compounds so A and D are valid coenzymes. biotin is an abundant coenzyme in the body. since zinc is an inorganic compound, it is a cofactor not a coenzyme.

match the following: POPULATION 1. growing 2. stable 3. diminishing SHAPE OF AGE PYRAMID a. triangular b. uru-shaped c. bell-shaped

1. a 2. c 3. b an age pyramid is a graphical representation of different age groups found in a population. the pre-reproductive groups are at the base, reproductive ones in the middle and post-reproductive groups at the top. it is of three types: triangular - when population is growing. bell-shaped - when population remains stable uru-shaped - when population diminishes in size.

match the following: 1. bacilli 2. cocci 3. staphylococcic 4. streptococcid a. rod-shaped b. occurs in chains c. sphere-shaped d. grapelike structure

1. a 2. c 3. d 4. b bacteria occur in many shapes and sizes. most bacteria have one of three basic shapes. rod-shaped bacteria are called bacilli. sphere-shaped are called cocci. spiral-shaped are call spirilla. streptococci occur in chains while staphylococci are like clusters of grapes.

match the following: 1. autonomous specification 2. conditional specification 3. syncytial specification a. characteristic of most insect classes b. characteristic of most invertebrates c. characteristic of all vertebrates

1. b 2. c 3. a in autonomous specification, if a particular blastomere is removed from an embryo early in its development, the isolated blastomere will produce the same cells that it would have made if it were still part of the embryo. it is the characteristic of most invertebrates. in conditional specification, if a blastomere is removed from an early embryo, the remaining embryonic cells alter their fate so that the roles of the missing cells can be taken over. it is the characteristic of all vertebrates and a few invertebrates. most insects use cyclical specification where interactions occur not between cells, but between parts of one cell.

match the following: 1. chordamesoderm 2. intermediate mesoderm 3. lateral plate mesoderm a. urogenital system b. notochord c. heart

1. b 2. a 3. c the mesoderm of a neural stage embryo is divided into five regions - chordamesoderm, paraxial mesoderm, intermediate mesoderm, lateral plate mesoderm and head mesenchyme. the tissue of the chordamesoderm forms the notochord. intermediate mesoderm forms urogenital system. the lateral plate mesoderm gives rise to the heart, blood vessels, the blood cells of the circulatory system and to the lining of the body cavities.

match the following: 1. zygomycota 2. basidiomycota 3. ascomycota a. small club-like structures are formed during sexual reproduction b. spores form in a saclike compartment c. conjugation

1. c 2. a 3. b sexual reproduction in the phylum zygomycota is called conjugation which occurs when the hyphae of two compatible mating types meet. during sexual reproduction, basidiomycota produce small club-like reproductive strictures called basidia and hence are often called club fungi. ascomycota are called sac fungi as a saclike compartment develops during sexual reproduction and sexually produced spores form in it.

match the following: 1. commensalism 2. symbiosis 3. mimicry 4. parasitism a. rhizobium - leguminous plants b. rafflesia c. sea anemone - hermit crab d. stick insect - dry branches

1. c 2. a 3. d 4. b commensalism is the association of two species in which one lives attached to the other, but does not harm it. sea anemone's live attached to the body of a hermit crab. symbiosis is the association of two species such that one benefits from and over the other. rhizobium stays in the root nodules of leguminous plants. some animals protect themselves by camouflaging their body shape and color to suit the environment. this is called mimicry. stick insects mimic thin dry branches. parasites derive food from their host. malaria parasites, tape worms etc. are examples of animal parasites. rafflesia, mistletoe etc. are parasitic plants.

the following changes occurring during human gestation, match the following: BY THIS TIME... 1. 3rd week 2. 5th week 3. 6th week 4. 8th week THESE BEGIN TO DEVELOP... a. eyes and ears b. tooth buds c. nervous system d. fingers and toes e. bones

1. c 2. a 3. d 4. b, e during the first trimester all these begin to form. in the third week the brain, spinal cord and nervous system begin to form. by the fifth week arms, legs, eyes and ears begin to form and at the sixth week fingers and toes develop. by the eighth week the mouth develops tooth buds that will become baby teeth and also the bones begin to develop.

the name of the structures in the cleidoic egg of reptiles and birds and their functions are given below. match the correct structure with its function 1. competitive behavior 2. communication 3. social behavior 4. cyclic behavior a. altruism b. hibernation c. territory d. mimicry

1. c 2. d 3. a 4. b an animal establishes an area, its territory, and prevents other members of the same species to occupy the area. this is called the competitive behavior of animal. some animals protect themselves by looking like a dangerous animal. this strategy is called mimicry and is an example of communication. sometimes animals act in such a way that other members of the group benefit from their actions, although that individual may be at a risk. this is known as altruism, a type of social behavior. some animals go into a period of inactivity, called hibernation, during the winter. this vent is an annual biological cycle and a kind of cyclic behavior.

the name of the structures in the cleidoic egg of reptiles and birds and their functions are given below. match the correct structure with its function 1. yolk sac 2. allantois 3. amnion 4. chorion 5. shell a. compartment acting as a watery cushion b. brittle cover protecting the inside c. supplies food for the developing embryo d. a chamber for gas exchange and the elimination of nitrogenous wastes e. projects embryo

1. c 2. d 3. a 4. e 5. b reptiles or bird egg's are enclosed (cleidoic) eggs consisting of five parts. inside the egg the embryo absorbs yolk and develops within a protective bath of amniotic fluid, which is well protected by a leathery or brittle shell. the yolk sac supplies food rich in high-caloric fat for the developing embryo. allantois is the chamber for gas exchange and the elimination of nitrogenous wastes. amnion acts as a watery fusion that saves the embryo from mechanical shock. chorion is the outer membrane surrounding and protecting the embryo.

which of the following represents the best match of the different epithelial cells and their location 1. simple squamous epithelium 2. simple cuboidal epithelium 3. simple columnar epithelium 4. stratified cuboidal epithelium 5. stratified squamous epithelium a. gall bladder, stomach, intestine b. epidermis of the skin c. blood vessels, lymph vessel d. collecting duct of kidney, bronchioles e. sweat glands

1. c 2. d 3. a 4. e 5. b squamous cells are flat cells. while simple squamous epitheliums are single layers and are found in the blood and lymph vessels, major body cavities and are associated with diffusion of water, electrolytes etc., stratified squamous epitheliums are multi-layers and are found in epidermis of the skin, anus, vagina etc. cuboidal cells are cubelike. while single layer simple cuboidal epitheliums are found in the smallest duct glands and many kidney tubules, stratified cuboidal epitheliums are found in the ducts of sweat glands. simple columnar cells are taller and also are single layered. they are found in stomach, large and small intestine, gall bladder etc.

match the following cell organelles and their functions: 1. ribosome 2. lysosome 3. smooth ER 4. Golgi apparatus 5. peroxisomes a. modifies protein after is has been synthesized b. is responsible for the conversion of H2O2 into water c. helps in protein synthesis d. stores and releases calcium ions e. contains digestive enzymes

1. c 2. e 3. d 4. a 5. b ribosomes are responsible for building proteins. lysosome contains digestive enzymes that break down large molecules like proteins, phospholipids, nucleic acid, carbohydrates, etc. an important role of smooth ER is to store the sudden release of Calcium ion. these are pumped into the lumen of ER from the cytosol where the concentration is a 100 times more than that in cytosol. Calcium ions are released back into cytosol when needed. Golgi apparatus modifies protein such that they are directed to the various parts of the cell. H2O2 is formed in the mitochondria and is converted to water by peroxisomes.

which two area in the brain are known as the speech centers

44 and 45 (Broca's area) of frontal lobe are called speech center, the center of articulate speech

the correct order of stages in the process of spermiogenesis is given by -cap phase -Golgi phase -acrosomal phase -maturation phase

Gogli phase -> cap phase -> acrosomal phase -> maturation phase the process of spermiogenesis is traditionally divided into four stages: the Golgi phase, the cap phase, the acrosomal phase, the maturation phase. in the Golgi phase, the head forms at one end of the spermatid and at the other end axoneme is formed. the Golgi apparatus creates enzymes that will form acrosome. in the cap phase, the Golgi apparatus surrounds the now condensed nucleus, becoming the acrosomal cap. in the acrosomal phase, the tail of the sperm is formed by the centriole. in the maturation phase, the excess cytoplasm is phagocytosed by surrounding Sertoli cells in the testes.

which represents the correct order of muscle contraction -secretion of neurotransmitter -activation of voltage-dependent calcium channels on the axon -formation of actin myosin crossbridge

activation of voltage-dependent calcium channels on the axon --> secretion of neurotransmitter --> formation of actin myosin crossbridge action potential comes to the neuron and activates voltage-dependent calcium channels on the axon. calcium causes vesicles containing the neurotransmitter acetylcholine to fuse with the plasma membrane, releasing acetylcholine into the synoptic cleft between the motor neuron terminal and the motor end plate of the skeletal muscle fiber. during the last step actin myosin cross bridge is formed. calcium binds to the troponin C present on the actin-containing thin filaments of the myofibrils and this makes the site available for myosin binding. myosin is then bound to actin in a string binding state.

neurofibromatosis is an autosomal dominant disease. if one parent is affected by the disease and is homozygous for the gene, what is the probability of having an offspring who is NOT affected by the disease? -0% -10% -25% -50% -100%

-0%

the total yield of ATP molecules that is produced during cellular respiration in prokaryotes is -30 -32 -34 -36 -38

-38

in PCR, DNA annealing occurs at -50-65 degrees Celsius -72 degrees Celsius -94-96 degrees Celsius -98 degrees Celsius -none of the above

-50-65 degrees Celsius the first step is initialization that occurs at 95 degrees celsius next step is DNA denaturation or melting, which occurs at 98 degrees celsius the next step is annealing (the RNA primers bind to the DNA template) and this occurs at 50-65 degrees celsius the last step is extension (nucleotides are added to the growing strand of DNA) and this occurs at 72 degrees celsius

which of the hormones below stimulates the maturation and release of an egg from the ovaries in the female reproductive system -LH -FSH -testosterone -A & B -all of the above

-A & B the maturation and please of eggs in the female reproductive system is called ovulation, and it is controlled by multiple hormones of the endocrine system. eggs are released from the follicles inside the ovaries. the eggs are released when the follicles rupture and release an egg after stimulation by follicle stimulating hormone (FSH) and luteinizing hormone (LH). FSH and LH are produced by the anterior pituitary gland in the endocrine system. testosteron is not involved in ovulation.

scientists believe ribosomes were the first organelles that formed on earth because they function as -catalysts -proteases -enzymes -A & C -all of the above

-A & C

the ability to perform photosynthesis is present in which of the following kingdoms -Monera -Protista -Plantae -A & C -All of the above

-All of the above cyanobacteria and proteobacteria belong to kingdom Monera and are both capable of photosynthesis. kingdom plantae are almost exclusively photosynthetic. some members of protista such as algae are also photosynthetic

multicellular bodies, movement, and heterotrophy are characteristics of which of the following kingdoms? -monera -protista -animalia -plantae -B & C

-B & C

during RNA replication, _________ adds new nucleotides to the growing DNA strand that are complementary to the parent strand -DNA polymerase -RNA polymerase -helicase -topoisomerase -ligase

-DNA polymerase helicase appears early in the process of DNA replication and unwinds the double stranded DNA molecule. topoisomerase reduces the tension due to supercoiling in the DNA molecules. this relaxes the DNA strands and permits replication. ligase appears near the end of of the process and glues the two newly formed DNA strands together. RNA polymerase also adds nucleotides, however this enzyme is involved in RNA synthesis.

of the following kingdoms, which does not contain members that are classified as photoautotrophs? -Monera -Protista -Fungi -A & B -A & C

-Fungi

the organelle that packages secretory vesicles for export from the cell is the -proteasome -ribosome -ER -smooth ER -Golgi apparatus

-Golgi apparatus

the immunoglobulin that is found in the bodily secretions such as tears, breast milk, and saliva is -IgA -IgB -IgD -IgM -IgE

-IgA IgB is not one of the 5 IgD and IgM are found on the surfaces of the B-cells that produce immunoglobulins. IgE is involved in allergic reactions and binds to parasitic worms that enter the body IgG (not listed) is the dominant immunoglobulin during the immune response against invading pathogens. IgG also crosses the placenta to provide immunity to the developing fetus

all of the following are required to perform a polymerase chain reaction except: -polymerase -RNA template -nucleotides -primers -none of the above

-RNA template

the region of the spermatozoa that contains digestive enzymes that break down the outer membrane of the egg during fertilization is the -acrosome -centriole -axisome -axial filament -flagellum

-acrosome

the movement of substance against their concentration gradient across the cell membrane is called -kinesis -passive transport -active transport -diffusion -facilitated diffusion

-active transport

which of the following is not a pyrimidine base found in nucleic acids? -thymine -adenine -uracil -cytosine -none of the above

-adenine purines in DNA and RNA - adenine, guanine pyrimidines in DNA - thymine, cytosine pyrimidines in RNA - uracil

the ______ division of the nervous system collects sensory information about the environment and transmits it to the central nervous system -motor -somatic -efferent -afferent -parasympathetic

-afferent efferent division handles the response to glands and tissues the motor response is also part of the efferent division the parasympathetic branch of the nervous system contains both afferent and efferent components that it uses to calm and relax the body when necessary. somatic means body, so somatic branch of the nervous system deal exclusively with the regions of the body that are under voluntary control.

in the coordinated muscle movement, the ______ is the muscle group that produces most of the force required to move -agonist -antagonist -fixator -synergist -origin

-agonist

movement is achieved in kingdom Protista by which of the following structures -cilia -flagella -pseudopodia -B & C -all of the above

-all of the above

the skeletal system -produces blood cells -stores minerals and lipids -protects and supports the tissues of the body -A & C -all of the above

-all of the above

histones regulate the expression of genes inside the nucleus. histones modify regions of the DNA using which of the following processes? -acetylation -deacetylation -methylation -A & C -all of the above

-all of the above gene expression is regulated by histones. histones are proteins that ind tightly to the DNA molecules inside the cell nucleus. histones can turn genes on and off by chemically modifying the DNA. the two chemical processes that have been observed are acetylation and methylation. in addition, the histones must remove these chemical groups after the DNA has been expressed through the reverse processes of deacetylation and demethylation.

the second law of thermodynamics states that -energy can neither be created nor destroyed -if two systems are in equilibrium with the third system, then they are also in equilibrium with each other -all system have a tendency to move towards disorder -it is impossible to cool a system to absolute zero -None of the above

-all systems have a tendency to move towards disorder 1st law - energy can neither be created nor destroyed 2nd law - all systems have a tendency to move toward disorder (called the law of entropy) 3rd law - no system can reach an absolute zero temperature 0th law - if two systems are both in thermal equilibrium with a third system, then they are in thermal equilibrium with each other

___________ enzymes have multiple binding sites -non-competitive -competitive -regulatory -catalytic -allosteric

-allosteric

the process by which animals increase the fitness of other members of their population, while lowering their own chances of survival is referred to as -imprinting -egotism -inclusive fitness -altruism -kinship

-altruism

unicellular organisms are found in all of the following kingdoms except -monera -protista -fungi -animalia -none of the above

-animalia

some damaged cells in the body undergo a process of programmed cell death that is called -blastulation -cell shrinkage -expansion -apoptosis -restriction

-apoptosis cell shrinkage is observed when cells are dehydrated but they can recover, and expansion refers to an exponential increase in the number of dividing cells. restriction is not a term specific to cell biology and can describe multiple biological or cellular processes.

in the central nervous system, there are more supportive cells than there are conducting cells. what type of supportive cell is most abundant in the central nervous system -neurons -oligodendrocytes -astrocytes -microglia -none of the above

-astrocytes the CNS is composed of the brain and the spinal cord. the primary cells in the nervous system are the neurons and glial cells. neurons transmit the info in the brain using electrical current that flows from neuron to neuron. the glial cells are supportive, and they nourish and support the neurons in the brain. the most abundant type of glial cells are the astrocytes. astrocytes are star shaped glial cells that maintain the blood-brain barrier. oligodendrocytes are glial cells that secrete myelin. myelin is the fatty substance that is wrapped around the axons of the neurons in the brain. it increases the speed at which the signals are conducted by the neurons. microglia are white blood cells that live in the brain, but they are not the most abundant.

the process in which the newly formed zygote undergoes cleavage is called -blastulation -gastrulation -metamorphosis -differentiation -induction

-blastulation

during photosynthesis the light-dependent reactions take place in the -plasma membrane -nucleus -chloroplast -inner mitochondrial matrix -plasmodesmata

-chloroplast light dependent reactions that occur during photosynthesis take place in the thylakoid membranes of the chloroplasts. the nucleus stores DNA and although it contains the genes that control all photosynthetic reactions, it is not the site of the light reactions. the plasma membrane functions as the protective barrier for the cell. inner mitochondrial matrix is involved in energy production via cellular respiration, however that is independent of photosynthesis. plasmodesmata are channels that connect individual plant cells allowing them to share their cytoplasm.

compact structures of DNA that are non-dividing are referred to as -chromosome -Barr bodies -chromatin -sister chromatids -karyotype

-chromatin

the ABO blood system in humans has three alleles that produce four phenotypes. in the AB blood group the A and B antigens are produced in equal amounts. this is an example of: -incomplete dominance -codominance -semi-dominance -masking -polygenic inheritance

-codominance incomplete dominance (also known as semi dominance) is when two genes create an intermediate phenotype such as pink flowers from red and white parents. masking refers to the ability of dominant alleles to hide or mask the expression of the recessive alleles in the organism. when multiple genes control a single trait the process is referred to as polygenic inheritance.

during protein translation, mRNA ________ bind to ________ on the tRNA -introns; exons -exons; introns -codons; anticodons -anticodons; codons -bases; triplets

-codons; anticodons introns and exons are found in DNA.

unlike normal cells in the body, cancer cells have the ability to form tumors. to accomplish this, what property of normal cells have cancers lost? -senescence -contact inhibition -differentiation -metastasis -none of the above

-contact inhibition senescence refers to a cell's inability to divide after is has reached the maximum number of divisions for its life span metastasis is not a property of normal cells, it is the ability of cancer cells to migrate to different tissues in the body. contact inhibition is the ability of a cell to detect its boundaries for growth. this prevents cells from piling on top of each other, and growing into neighboring tissues to form tumors. this is absent in cancer cells and allows them to form tumors.

the integumentary system -produces Vitamin A -contains thermoreceptors for sweating -consists of an alkaline environment -A & B -A & C

-contains thermoreceptors for sweating exposure of the skin to sunlight causes the production of vitamin D not vitamin A. the surface of the skin is a salty, acidic environment that prevents the growth of several types of pathogens. the receptors for sweating are in the skin

all of the following are members of kingdom Protista except: -green algae -amoeba -cyanobacteria -all of the above -none of the above

-cyanobacteria

the part of the long bone that is composed of compact bone and surrounds the marrow cavity is called the -epiphysis -periosteum -growth plate -diaphysis -none of the above

-diaphysis the epiphysis is found at the ends of the bone the periosteum is the outer lining of the bone the growth plate is near the epiphysis at the ends of the long bone the diaphysis is the midsection of the bine and it surrounds the marrow cavity

alleles are -identical copies of the same gene -different copies of the same gene -the recessive form of a gene -the dominant form of a gene -present in three copies

-different copies of the same gene

At the end of mitosis the daughter cells are _______ -haploid -diploid -sister chromatids -homologous -recombinants

-diploid

in a type III survivorship curve, mortality occurs -early in life -late in life -in middle-aged organisms -both early and late in life -at a constant rate and is independent of age

-early in life in type I most individuals survive early in life and die in the latest stages of life in type II mortality rates are constant regardless of the ages type III most individuals experience death early in life C and D do not describe any curves

the separation of DNA fragments based on their sizes is called -sequencing -cloning -electrophoresis -labeling -profiling

-electrophoresis sequencing is the elucidation of the DNA sequence of a gene, chromosome, or organism by using sequencing technologies. cloning is the engineering of genetically identical organisms. labeling is a general biological technique that is used to mark molecules of interest using fluorescent, radioactive, or protein compounds. profiling is the use of DNA technology to identify an individual based on the uniqueness of each individual's genome.

The first law of thermodynamics states that -energy can neither be created nor destroyed -if two systems are in equilibrium with a third system, then they are also in equilibrium with each other -all system have a tendency to move towards disorder -it is impossible to cool a system to absolute zero -none of the above

-energy can neither be created nor destroyed

the layer of the skin in which the Langerhan cells are found is the -cuticle -dermis -endoderm -epidermis -hypodermis

-epidermis

the layer of the skin in which melanin producing cells are found is the -dermis -endoderm -mesoderm -epidermis -hypodermis

-epidermis the dermis is located below the epidermis and the hypodermic is below the dermis. endoderm and mesoderm are germ tissues that are present during early embryological development.

in the male reproductive system, sperm is stored in which of the following structures before it is ready for release from the body? -epididymis -urethra -vas deferens -prostate gland -testes

-epididymis the sperm is produced in the testes and then sent to the epididymis (a system of tubes that stores it until it is ready for release from the body). the vas deferens are the ducts that transport the sperm from the epididymis to the ejaculatory duct. the prostate gland produces a slippery fluid that combines with the sperm to form semen. the urethra is the opening through which the sperm leaves the body.

in the three domain classification of life, members of domain archaea have more biochemical similarities with which of the following -bacteria -eukaryotes -cyanobacteria -A & C -None of the above

-eukaryotes there are three domains of life: archaea, bacteria, eukaryotes archaea - single celled organisms that resemble bacteria which are also single celled. although they resemble bacteria structurally, biochemically and genetically they are more closely related to eukaryotes which include protists, fungi, plants, and animals.

in movement, the muscle group that prevents the movement of bones is called the -origin -agonist -antagonist -fixator -insertion

-fixator

saprotrophic organisms that exist in multicellular forms belong to which of the following kingdoms? -monera -protista -animalia -plantae -fungi

-fungi members of kingdom Monera are all unicellular Protists are generally unicellular as well (the exception being algae and autotrophs that make their own food) Animalia and Plantae are all multicellular but they are not saprotrophic, rather they digest food internally, or produce it through photosynthesis.

the process in which the early embryo forms a three-layered structure is referred to as -blastulation -gastrulation -morulation -organogenesis -schizocoely

-gastrulation morulation is the formation of the 32-celled morula following symmetrical divisions that begin with the fertilized egg. following morulation is blastulation in which a hollow sphere of cells is formed. next is the gastrula which contains the three germ layers: endoderm, mesoderm, and ectoderm that will form the tissues of the body. organogenesis is the development of organs later during development. schizocoely occurs when a body cavity is split into two, and it is absent in deuterostomes such as humans.

according to the principle of _________, speciation is a slow process that occurs in a uniform progression over time -punctuated equilibrium -gradualism -natural selection -mass extinction -adaptation

-gradualism

a pair of chromosomes that contain genes at the same loci, have the same length, and have the same centromere positions are referred to as -alleles -sister chromosomes -sister chromatids -homologous chromosomes -autosomes

-homologous chromosomes

in the DNA molecule the base pairs are held together by -hydrophobic interaction -hydrogen bonds -ionic bonds -phosphodiester bonds -peptide bonds

-hydrogen bonds adenine-thymine and cytosine-guanine are held together by hydrogen bonds.

during the cell-cycle, cells carry out their normal functions and double their number of organelles in what stage? -prophase -metaphase -anaphase -interphase -telophase

-interphase organelle are not duplicated during mitosis so organelle are not doubled during prophase, metaphase, anaphase, and telophase

the process of bone formation in the fetus in which cartilage is absent is known as -endochondral ossification -intramembranous ossification -longitudinal growth -appositional growth -fracture repair

-intramembranous ossification during endochondral ossification, cartilage is replaced by bone. in contrast, during intramembranous ossification cartilage is not present, so that's the correct answer. longitudinal growth describes an increase in the length of a bone, while appositional growth describes an increase in bone diameter. fracture repair is a complex process that requires replacement and remodeling of a bone multiple directions and it is not usually a component of fetal development.

the third law of thermodynamics states that -energy can neither be created nor destroyed -if two systems are in equilibrium with a third system, then they are also in equilibrium with each other -all systems have a tendency to move towards disorder -it is impossible to cool a system to absolute zero -none of the above

-it is impossible to cool a system to absolute zero

which of the following is not a function of the kidneys? -renal reabsorption -renal excretion -hormone secretion -filtration -micturition

-micturition in renal reabsorption, the kidneys return solutes and water to the blood. in renal excretion the kidneys remove excess solutes and pharmacological drugs. kidneys secrete hormones such as erythropoietin, and the kidneys filter the blood before it is returned to the heart micturition is the process by which urine is intermittently released from the bladder

in eukaryotic cells, cellular respiration takes place in the -chloroplast -mitochondrial matrix -outer membrane of the mitochondria -plasma membrane of the cell -lungs

-mitochondrial matrix

the inability of chromosome to separate properly during anaphase is known as -nondisjunction -translocation -inversion -chromosome duplication -chromosome loss

-nondisjunction

the functions of the kidney are important for maintaining homeostasis in the body. which of the following is not a function of the kidney -hormone production -fluid retention -salt retention -blood filtration -none of the above

-none of the above

the plasma membrane in cells contains all of the following except: -proteins -phospholipids -cholesterol -glycolipids -none of the above

-none of the above

which of the following structures is not involved in cell to cell connections by the cell? -desmosomes -gap junctions -tight junctions -the extracellular matrix -none of the above

-none of the above

nucleotides consist of all of the following except: -phosphate group -monosaccharide -nitrogenous base -all of the above -none of the above

-none of the above all three of these are components of nucleotides

hematopoietic stem cells can be chemically induced to differentiate into all of the following except: -erythrocytes -lymphocytes -megakaryocytes -monocytes -none of the above

-none of the above hematopoietic stem cells give rise to the blood cells in the body. erythrocytes are red blood cells lymphocytes are specialized type of white blood cell monocytes are also white blood cells megakaryocytes are cells that fragment to form the platelets in the body that are responsible for forming blood clots. all of them are derived from hematopoietic stem cells

inside the cell, ribosomes are produced by which of the following -microtubules -golgi apparatus -rough ER -nucleolus -smooth ER

-nucleolus

what is the final electron acceptor in the electron transport chain? -pyruvate -oxygen -carbon dioxide -FAD -NADH

-oxygen

a symbiotic relationship between two organisms in which one partner benefits, while the other is harmed is called -commensalism -mutualism -adaptation -parasitism -predation

-parasitism when the union is harmless and one-partner benefits, it is referred to as commensalism. when both members benefit form the union they are experiencing a mutualistic relationship. if one-partner benefits, while the other is harmed then the union is considered parasitism. predation, though it involves harm is not an example of symbiosis. adaptation is the ability of organism to adjust to changes in their environment and is independent of symbiosis

the organelle inside of the cell that breaks down toxic chemicals is the -lysosome -peroxisome -ribosome -centriole -desmosome

-peroxisome lysosomes - digests cellular wastes ribosomes - responsible for protein synthesis centrioles - exist in pairs and are involved in cell division desmosomes - NOT organelles, rather they are a type of adhesion structure that joins individual cells together to form tissues. remember its PEROXisome because it breaks down chemicals such as hydrogen PEROXide

the innermost layer of the meninges is the -dura mater -epidural space -subdural space -subarachnoid space -pia mater

-pia mater dura mater is the outermost arachnoid later is the middle pia mater is the innermost

in humans, eye color is controlled by three independent genes. this is an example of -codominance -blending -pleiotropy -polygenic inheritance -recessiveness

-polygenic inheritance one gene can control many traits and one trait can be controlled by many genes the basis of eye color is one trait being controlled by many genes. pleiotropy is the opposite, where one gene controls many traits. blending is another term for incomplete dominance (a pink flower from white and red parents)

inside the cell proteins are synthesized by what organelles? -nucleus -mitochondria -lysosomes -ribosomes -smooth ER

-ribosomes nucleus stores DNA and controls the cell's activities mitochondria are the sites of energy production lysosomes digest cellular waste smooth ER is the site of lipid production

heterotrophic animals that are predators of herbivores are classified as -primary producers -primary consumers -secondary consumers -tertiary consumers -C & D

-secondary consumers heterotrophs are animals that cannot produce their own food and must "feed" to survive. unlike heterotrophs, producers make their own food usually through photosynthesis. primary producers include algae and plants. primary consumers include herbivores secondary consumers include feed on herbivores. tertiary consumers are carnivores that feed on primary and secondary consumers.

the region of the heart that initiates the heartbeat is the -aorta -left ventricle -atrioventricular node -sinoatrial node -Bundle of His

-sinoatrial node the heartbeat is initiated by the pacemaker cells, collectively called the sinoatrial node located in the right atrium of the heart. atrioventricular node sets the rhythm of the heartbeat.

in the eukaryotic cell, the structure that carries out lipid synthesis is the -liposome -lysosome -rough ER -smooth ER -Golgi apparatus

-smooth ER liposomes are not organelles. they are synthetic vesicles that are made from the phospholipids that form the plasma membrane in the cell. the lysosome is an organelle but it is involved in digestion of cellular debris. the golgi apparatus packages and ships vesicles, but it does not produce lipids. the rough ER produces and processes proteins while the smooth ER produces lipids.

involuntary muscles that do not contain striations are called -skeletal muscles -cardiac muscles -smooth muscle -autonomic muscle -cerebral muscle

-smooth muscle there are three types of muscles. smooth muscles lack striations and are involuntarily controlled. skeletal muscles are striated and are under voluntary control. cardiac muscles are striated but involuntarily controlled.

helical, spiral shaped bacteria with long cells in the kingdom Monera are classified as -cocci -spirochetes -spirilla -vibrio -bacilli

-spirochetes cocci - bacteria that are round and spherical spirochetes - bacteria that have long, helical, twisted bodies spirilla - bacteria that have twisted bodies that resemble spirals. vibrio - bacteria that are comma shaped bacilii - bacteria that are shaped like rods

if the genetic variation decreases in a population because extreme phenotypes are no longer favored, this is referred to as -population bottleneck -disruptive selection -stabilizing selection -directional selection -heterozygote advantage

-stabilizing selection

in the digestive system the breakdown of proteins begins in which of the following digestive organs? -oral cavity -small intestine -stomach -colon -jejunum

-stomach different macromolecules begin their digestion at different location throughout the digestive tract. in the oral cavity, carbohydrates begin their digestion due to the presence of enzymes such as amylase. in the stomach, pepsin is released which breaks down proteins. in the small intestine fats, carbohydrates, and proteins are digested and nutrients are absorbed. the jejunum is a part of the small intestine. the colon is not a site of digestion, rather it is the site of reabsorption of water and other nutrients.

in photosynthesis, the Calvin cycle takes place in what part of the cell? -thylakoid membrane -stroma -vacuole -nucleus -stomata

-stroma

during respiration gas exchange takes place in the alveoli of the lungs. what is the name of the slippery substance that is secreted by the alveoli to reduce its surface tension? -mucus -serous fluid -surfactant -cerumen -sebum

-surfactant surfactants are slippery fluids made up of proteins and lipids that reduce the surface tension of the alveoli. mucus is secreted by multiple surfaces of he body, including the lungs to protect the body from infection and wash away contaminants. serous fluid fills the cavities in the body. cerumen in another term for earwax. sebum is an oily substance secreted by glands in the skin

the establishment of a new population by a small group of individuals in which genetic variation is significantly decreased is referred to as -the founder effect -a population bottleneck -inbreeding -selection -adaptive evolution

-the founder effect in a population bottle neck most members of the population are killed or prevented from reproducing. inbreeding lowers genetic variation at the familial level, but not necessarily at the population level.

which of the following is true of the digestive, respiratory and urinary systems? -they maintain fluid levels in the blood -they maintain constant body temp -they maintain oxygen levels in the blood -they maintain the balance of substances in the blood -all of the above

-they maintain the balance of substances in the blood

stem cells that have the ability to differentiate into all types of human tissues are -omnipotent -multipotent -pluripotent -totipotent -clones

-totipotent omnipotent is not a term used to describe stem cells multipotent cells give rise to closely related families of cells pluripotent cells give rise to the three germ layers during embryonic development (endoderm, mesoderm, ectoderm)

the outer layer of the blastocyst that nourishes the developing embryo is referred to as the -blastocoel -trophoblast -morula -blastula -none of the above

-trophoblast

premature male pattern baldness is caused by a gene located on the -x chromosome -y chromosome -x and y chromosome because humans need two copies of each allele -z chromosome -autosomal chromosomes

-x chromosome it's passed on by the mother so it has to be the x chromosome

during human fertilization sperm cells bind to the ______ of the egg -acrosome -zygote -zona pellucida -zona pericolosa -cortical granules

-zona pellucida acrosome is a cap like structure on the head of the sperm zygote is a fertilized egg cortical granules inside the egg digest sperm receptors on the egg's surface so that it can no longer bind additional sperm. the zona pericolosa is not a real biological structure. zona pellucida is the region of the egg that fuses with the sperm head during fertilization.


Conjuntos de estudio relacionados

Ch. 17 Med Term- Female Reproductive Sys

View Set

Chapter 30: Recognizing and Identifying the Hazards - Lecture and Quiz

View Set

Marketing Essentials Pt. 1 Ch. 1

View Set

Night By Elie Wiesel (Page 1-46)

View Set